Вы находитесь на странице: 1из 186

CAT VERSION 2

1. A client with a hemothorax has a chest tube in the fourth intercostal space
connected to suction at 20 cm H2O pressure. Four hours after insertion, which
client outcome should the nurse consider to be within normal limits for this
client?

A. No bubbling in the suction chamber of the Pleuravac


B. Serous fluid in the drainage chamber of the Pleurovac
C. Fluctuation with respiration in the water-seal chamber of the
Pleuravac
D. The dry gauze dressing over the insertion site is clean and intact

2. A client has started long-term maintenance therapy with a cardiotonic-“Toxic”


medication that has a narrow therapeutic index. Teaching the client the
signs/symptoms of which adverse effect is most important?

A. Displacement
B. Toxicity
C. Dependence
D. Tolerance

3. In caring for a client who is receiving peritoneal dialysis, the nurse should be
alert for that what complications?

A. Clear dialysate drainage and burning on urination


B. An occluded vascular access device and flank pain
C. Abdominal pain, tenderness, and rigidity (peritonitis)
D. Increased serum albumin level, decreased BUN, and increase hematocrit
4. A high fluid intake is prescribed for a client with urolithiasis. The client wishes
to know the chief purpose for this intervention. What should the nurse tell the
client about this prescription?(esto ocurre en renal calciu)(stone—piedres)

A. This action is designed to decrease the uric acid in the urine


B. The purpose is to increase the hydrostatic pressure behind the stone
to assist in its downward passage( ayudar a la piedra salir con
precion)
C. The intent is to increase the specific gravity of the urine, thereby
increasing the probability of passing the stone
D. The fluids will increase bilirubin excretion, thereby assisting to resolve
jaundice associated with stone formation

5. Normal saline 0.9% is prescribed for a client with fluid volume deficit at a rate
of 100 ml/hour. Before starting the infusion, the nurse observes that the client’s
urine is dark amber in color. What action should the nurse take?

A. Start the IV at a keep-open rate until the assessment finding is reported o


the healthcare provider
B. Insert a saline lock, but do not start any IV fluid until contacting the
healthcare provider
C. Review the list of PRN medications to see if a diuretic can be
administered
D. Administer the normal saline at the prescribed rate of 100 ml/hour

6. Which explanation of autonomic cardiac regulation mediated by sympathetic


innervations is correct?

A. Sympathetic activation boosts K+ efflux and increases the inotropic


effect
B. Increased Ca+ influx with sympathetic stimulation raises the heart
rate ( increased Na + tambien)
C. Sympathetic activation decreases dromotrophy by lowering conduction
speed
D. Increased Na+ influx with sympathetic stimulation reduces pacemaker
firing

7. The nurse learns that a newly admitted adult client has a six month history of
recurring somatic pain. During the admission interview, it is most important for
the nurse to question the client about what problem ?problema cronico
A. Periods of restlessness
B. Episodes of tremors
C. Feelings of depression
D. Nausea and vomiting

8. A pregnant client begins to cry when the UAP tries to assist her in donning a
hospital gown, and she refuses to remove an undergarment that is worn in her
culture to preserve modesty. What should the charge nurse do first?

A. Incorporate individualized cultural care into the nursing plan of care


B. Discuss the importance of respecting cultural beliefs with the UAP
C. Determine if continued wearing of the garment will compromise care
D. Talk with the client to determine alternate means to preserve modesty

9. The nurse is preparing to insert an IV in an adult male client. Which client’s lab
value is most important for the nurse to consider prior to inserting the (IV? para
evitar prolongado sangramiento)

A. Serum sodium of 130 mEq/L


B. WBC of 12,000/mm
C. Hemoglobin of 12 g/dl
D. Platelet count of 60,000/mm
10. A 12-year-old boy who is 54 inches tall is scheduled for x-rays of his hands
and wrist to determine growth patterns. The mother asks the nurse why these x-
rays are being taken. What explanation is best for the nurse to provide this
mother?(en el Rx se ve la epiphysis del hueso que es la covectura de cartilage
que cubre la cabeza del hueso)

A. If the growth areas of the bone are closed, then growth hormone therapy
can open them
B. Hormonal influences on the bone at this age can be determined by x-
ray
C. Wrist and hand fractures are common among children of small stature
D. X-ray therapy is helpful in promoting the effectiveness of growth
hormone therapy

11. The nurse is reviewing laboratory results for a client with adrenal insufficiency.
Which finding should the nurse report to the healthcare provider?

A. Calcium 12 mg/dl (esta alto)


B. Sodium 138 mEq/L
C. Glucose 110 mg/dl
D. Potassium 4.0 mEq/L

12. At 0700 the nurse receives report for a client with chronic intractable pain
“who needs morphine every 4 hours during the day shift to control pain.” After
reviewing the client’s record, what action should the nurse implement?

A. Request a change in the prescribed dose of fentanyl (Duragesic)


transdermal patch
B. Scheduled the PRN doses of morphine and codeine at the same time
every 4 hours
C. Correct the shift summary to be consistent with the medication
administration record
D. Administer a PRN dose of morphine immediately at the IV rate of 1
mg/minute
13. After a client experiences spontaneous rupture of the membranes during labor,
the nurse notes a visible prolapse of the umbilical cord. What intervention
should the nurse implement immediately?

A. Push the presenting part off the cord


B. Turn the client to a supine position
C. Administer oxygen by face mask at 6L/min
D. Prepare the client for a cesarean delivery

14. The nurse is preparing a teaching plan for a client receiving magnesium-based
antacids for treatment of gastro-esophageal reflux disease (GERD). Which
instruction should the nurse plan to include?

A. “Increase fiber and fluids in your diet to prevent constipation”


B. “Avoid taking any other drugs 1 to 2 hours before and after taking
the antacid”
C. “Swallow the antacid with a glass of low-fat milk to help coat the
stomach lining”
D. “Take the antacids on an as-needed basis whenever you feel bloating or
heartburn”

15. The nurse is caring for a young adult male client with facial injuries resulting
from a motor vehicle collision. Which client statement indicative of the highest
priority for nursing intervention?

A. “I am not taking any more medications because they make my mouth


dry”
B. “I don’t want my family and friends to see me looking like this”
C. “My biggest fear is that this injury will cause me to lose my job”
D. “I can’t sleep through the night because I awaken with pain when I
move”

16. What is the most important primary preventative measure the nurse can
emphasize as a means of reducing the risk of developing acute
glomerulonephritis in the general population?
A. Teach all females to seek medical attention for urinary tract infections
B. Encourage all persons to have a yearly physical with a urinalysis
C. Use good hand washing techniques to prevent throat and skin
infections
D. Eat a low salt diet and monitor the blood pressure frequently

17. The mother of a child with cerebral palsy (CP) asks the nurse if her child’s
impaired movements will worsen as the child grows. Which response provides
the best explanation?

A. The outcome depends on the continued development of the brain lesion


B. The course of CP is variable but the brain damage is not progressive
C. The most common permanent physical disability of childhood is CP
D. The classification of CP determines the severity of motor dysfunction

18. Three days postoperative, a client’s wound drainage changes in appearance


from sanguineous to serous. Based on this finding, what nursing intervention
should the nurse implement?

A. Monitor the client’s vital signs


B. Apply pressure to the wound
C. Continue to monitor the wound…..serous “good” (clear liquid)
D. Obtain a wound culture

19. Following the administration of total parenteral nutrition (TPN) via a central
line to a client diagnosed with inflammatory bowel disease (IBD), the nurse
should expect what outcome?

A. A negative nitrogen balance during TPN administration


B. A weight loss of 6 pounds within two weeks
C. Afebrile with no purulent drainage from catheter site
D. Hydration as evidenced by tented skin turgor

20. Based on the principles of asepsis, the nurse should consider which
circumstance to be sterile?
A. An open sterile Foley catheter kit set up on a table at the nurse’s
waist level
B. A sterile glove the nurse thinks might have touched her hair
C. A one-inch border around the edges of a sterile field set up in the
operating room
D. A wrapped, unopened sterile 4x4 gauze pad placed on a damp table top
21. The nurse is preparing to administer medications to a client who was admitted
to the hospital with a diagnosis of deep vein thrombosis (DVT). Which action
should the nurse implement?

A. Prepare to give a one time dose of Vitamin K 2.5 mg PO


B. Give the next oral dose of Coumadin 2 mg
C. Administer the next dose of Coumadin 5 mg
D. Report the laboratory findings to the healthcare provider

22. The nurse-researcher determines that a strong correlation exists between an


increased likelihood of infection with Crystosporidium parvum and clients
infected with HIV. Which factor of the epidemiologic triangle plays a primary
role in this correlation?

A. Environment
B. Host(anfrintion)
C. Agent
D. Social

23. A woman is brought to the labor and delivery unit after delivering a term infant
and the placenta in the hospital parking lot 10 minutes ago. What action should
the nurse perform first?
A. Draw blood for hemoglobin and hematocrit
B. Inspect the perineum for lacerations
C. Obtain a complete obstetrical history
D. Perform a fundal massage

24. A nurse is interviewing a client with a history of COPD, who is dyspneic and
has a respiratory rate of 36 breaths/minute. Which nursing diagnosis has the
highest priority?
A. Knowledge deficit
B. Impaired verbal communication
C. Ineffective individual coping
D. Alteration in body image

25. A nurse with 15 years experience working in the emergency room is reassigned
to the perinatal unit to work 8 hour shift. Which client is best to assign to this
nurse?

A. A postpartum client with an infected “episiotomy”


B. A client at 36-weeks gestation who is possibly in labor
C. A client in labor who is dilated to 3 cm
D. A mother and baby who just delivered

26.Which statement indicates to the nurse that a client understands medication


teaching about alendronate (Fosamax)? ….”F” as in fluid

A. “I will take the medication with a full glass of water”


B. “I will lie down for a half hour after taking the medication”
C. “I will take this medication with my breakfast”
D. “I will avoid milk and yogurt while I am taking the medication”

27. The nurse is preparing to remove the staples from a client’s abdominal incision
and observes that the wound edges are fully approximated. What action should
the nurse implement?
A. Use a staple remover to release the staples from the incision
B. Remove every other staple and apply adhesive skin closures
C. Cover the wound with a sterile gauze and contact the surgeon
D. Assess the length and depth of tunneling around the wound

28.A client at 32-weeks gestation reports to the clinic nurse that she has a new
onset of bright red, painless vaginal bleeding. Which intervention should the
nurse implement?

A. Teach that bleeding is normal due to Braxton-Hicks contractions


B. Position the client in a side-lying position
C. Obtain a urine specimen, and assess for white blood cells
D. Assess for signs of preeclampsia

29. In assessing a client who has just undergone a lung biopsy, the nurse is unable
to auscultate breath sounds on the biopsied side and observes that the client is
dyspneic and has slight hemoptysis. While contacting the healthcare provider to
report these findings, what intervention should the nurse implement?

A. Prepare the client for a chest x-ray….makes sense! to see what’s


going on in the lungs
B. Assess for jugular vein distention
C. Position the client with feet elevated
D. Obtain a sputum specimen

30.A client who was in a house fire is brought to the emergency department.
Which assessment finding should the nurse respond to first?

A. The client’s voice is hoarse and nasal hair is singed….”nasal,


meaning potential breathing problem”
B. 18% of the body surface has partial thickness (2nd degree) burns
C. The bum site pain is rated as a “10” on a scale of 0 to 10
D. Vital signs are BP 180/70 mm Hg, pulse 100 beats/minute, respirations
24 breaths/minute
31. The nurse is conducting assessments at the beginning of the shift. Which client
is most likely to have an increased blood pressure since the last set of vital signs
was recorded four hours ago?

A. An adolescent who is receiving azathioprine (Imuran) following a cardiac


transplant
B. A young female with increased urinary output following administration
of IV furosemide (Lasix)
C. A middle-aged male receiving prazosin hydrochloride (Minipress)
D. An elderly male who received two units of packed red blood cells
(RBCs)

32. A male client had a thyroidectomy 24 hours ago, and now complains of
cramping in the hand of the arm where his blood pressure is being taken. The
nurse notes that his hand is twitching. What intervention should the nurse
implement first?

A. Notify the healthcare provider immediately


B. Review the client’s serum calcium level
C. Take the blood pressure in the other arm
D. Administer a PRN analgesic

33.A primigravida who is Rh-negative spontaneously delivers a full term infant at


home. Two days later, the client and infant arrive at the clinic, and the
healthcare prescribes Rho (D) immune globulin (RhoGAM) because the infant
is Rh-positive. What action should the nurse implement prior to administering
the RhoGAM?

A. Assess the infant’s conjunctivas for jaundice


B. Ask a second nurse to verify the dosage and client
C. Complete a RhoGAM identification card
D. Use a filtered needle to administer the RhoGAM
34. A client with depression remains in bed most of the day, declines activities,
and refuses meals. Which nursing diagnosis has the greatest priority for this
client?

A. Self-care deficit
B. Self-care disturbance
C. Social isolation
D. Impaired social interactions

35. An older female client with cirrhosis of the liver related to alcohol abuse
reports to the nurse that her stools look like “black tar”. What action should the
nurse take?

A. Remind the client that years of alcohol abuse have caused her current
health problems
B. Tell the client to report to the emergency room immediately for
further assessment(adicional)
C. Instruct the client to call the clinic if she notices bright red blood in her
stools
D. Tell the client that age-related changes in the bowel often result in dark
stools

36. The nurse is developing a teaching plan for a client with varicose veins. What
instruction should be included in this plan?

A. Walk several minutes every hour


B. Cross legs at the thighs only
C. Use elevators, instead of stairs
D. Soak feet in warm water when fatigued

37. A female client who had a kidney transplant 5 hours ago is receiving
replacement IV fluids to match urine output. She has had 950 ml urine output
over the last hour and has a weak, irregular pulse. The electrocardiogram
indicates occasional preventricular contractions (PVC). What is the highest
priority nursing action?
A. Document urine output
B. Monitor for rejection
C. Assess serum electrolytes …..esp. potassium
D. Stop intravenous fluids

38. A client with peptic ulcer disease (PUD) is admitted to the emergency room
complaining of sudden severe upper abdominal pain. Assessment indicates an
extremely tender and rigid abdomen, B/P of 90/60 mm Hg, and pulse of 110
beats/minute. The emergency department nurse should anticipate
implementation of which intervention?

A. Administering an iced saline lavage


B. Infusing the proton pump inhibitor Protonix intravenously
C. Preparing the client for emergency abdominal surgery
D. Inserting a nasogastric tube to decompress the bowel

39.Pain medication was administered one hour ago to a 3-year-old child who had a
short arm cast applied to the left arm three hours ago. The child continues to
cry, the fingers are cold and dusky, and the capillary refill is five seconds.
Which intervention should the nurse implement?

A. Administer additional pain medication


B. Assess the child’s fingers hourly
C. Prepare to bivalve the cast
D. Request a portable STA x-ray

40. The nurse is planning care for a client diagnosed with end-stage cirrhosis of the
liver secondary to alcoholism. When assigning care for this client to a practical
nurse (PN), what information is accurate for the charge nurse to provide the
PN?

A. Higher amounts of narcotics are often needed for pain control


B. Drug doses are often reduced for clients with liver failure
C. Creatinine and BUN blood levels should be monitored daily
D. Decreasing ammonia levels may increase susceptibility for infection

41. A male college student returns to the student health clinic one week after
receiving a positive mono spot test for mononucleosis and requests a
prescription for amoxicillin (Amoxil, Polymox). He is afebrile and complains of
fatigue, a sore throat, dysphagia, and extremely swollen glands. What response
should the nurse provide?

A. Inform the healthcare provider of the client’s request for the prescription
B. Emphasize the need to avoid contact sports for at least two weeks
C. Clarify that these symptoms will not respond to antibiotic therapy
D. Explain that no effective treatment is available for these symptoms

42. A nurse developed an educational program on healthy eating for high schools
students. The program consisted of a series of four classes. What finding is
indicative of a program outcome?

A. 90% of the students rated the instructor as superior


B. Students showed interest in the food models used during the second class
C. Overweight students lost an average often pounds by the program’s
end….dope
D. Student surveys showed that they enjoyed the classes

43. While the female psychiatric nurse is on the phone, a male client, diagnosed
with an antisocial personality, interrupts the nurse and tells her that he needs to
talk to her about something very important. Which action should the nurse
implement?

A. Hang up the phone and explain the consequences of his behavior


B. Ask another nurse to talk with the client until she gets off the phone
C. Allow the client to explain what is wrong then finish the phone
conversation
D. Tell the client she is busy and will talk to him after getting off the
phone

44. A client in the first trimester of pregnancy calls the nurse to report she has
symptoms of a cold and wants to know if it is safe for her to take the herb
Echinacea. Which instruction should the nurse provide this client?

A. Check the label regarding use of Echinacea during pregnancy


B. Take Echinacea until the cold symptoms are relieved
C. Use an herbal tea to relieve the cold symptoms
D. Avoid using herbs during the first trimester of pregnancy

45. A female client is admitted to the psychiatric department on an emergency


commitment. The client’s husband asks the nurse, “What is going to happen to
my wife? Can I take her home now?” Which information should the nurse
provide?

A. A psychiatric evaluations is required for continued hospitalizations


B. Emergency commitment extends to a maximum of 90 days
C. Discharge can be completes after arrangements with the business office
D. Hospitalization is mandated until a mental health court hearing is
held

46. Which technique should the nurse use to assess for manifestations of erythema
infectiosum (fifth disease) in a 4-year-old?

A. Palpate lymph nodes


B. Auscultate breath sounds
C. Visualize oropharynx
D. Observe physical appearance…..Fiscal appearance! “Fifth disease”

47. A male client is scheduled for a cardiac catheterization in the morning. Which
interventions should the nurse plan to implement prior to this procedure?

A. Explain to the client that he will be asleep during the procedure and will
not experience any discomfort
B.
C. Offer a clear liquid diet prior to the procedure and hold all medications
the morning of the procedure
D. Inform the client that he may experience a flushed feeling
throughout his body when the dye is injected
E. Explain to the client that the procedure will last about 30 minutes and
will be done in the x-ray department

48. A male client, who is in end stage renal disease and has been on a waiting list
for a transplant for over one year, s told his condition is now terminal. He tells
the nurse that he found a Web site with a kidney for sale, and asks the nurse
where he can obtain a tissue match analysis. What is the nurse’s ethical
responsibility?

A. Suggest a support group for renal transplant recipients and their families
B. Inform the client that it is a criminal offense to purchase organs in
the United States
C. Provide the client with a scheduled for the tissue bank mobile unit for his
local area
D. Report the client’s desires to the healthcare provider and recommend a
psychiatric consultation

49. A client is receiving a nitroglycerin infusion at 10 mcg/min. The pharmacy


dispenses an IV solution of nitroglycerin 50 mg in 250 ml of DW. The nurse
should program the infusion pump to deliver how many ml/hr?
Answer: 3
50. A young adult male client is admitted to the emergency room with a bleeding
abdominal wound following a motor vehicle collision. He is crying out with
pain. His friends report that he often uses cocaine. What nursing diagnosis has
the greatest priority?

A. Pain related to injuries


B. Anxiety related to trauma of motor vehicle accident
C. High risk for injury related to cocaine withdrawal
D. High risk for injury related to hemorrhage

51. A client receiving oxygen at 2 L/minute per nasal cannula has a change in
oxygen saturation from 92% at 0800 to 88% at 1200, but there is no change in
respiratory rate during this same time period. What action should the nurse take
first?

A. Document the saturation level in the medical record


B. Increase the oxygen flow from 2 to 3 L/minute…..U need an order
C. Remove the nasal cannula and apply a face mask
D. Apply the pulse oximeter to a different finger

52. A post-term primipara is admitted to labor and delivery for scheduled induction
of labor. What finding should the nurse report to the healthcare provider before
initiating the infusion of oxytocin (Pitocin)?

A. Early labor evidenced by regular contractions every 15 minutes


B. Leopold maneuvers indicating a breech presentation
C. Sterile vaginal exam revealing 3 cm dilatation
D. Biophysical profile results showing oligohydramnios

53. Which member of the interdisciplinary team on a skilled nursing care unit can
legally prescribe medications?
A. Geriatric nurse practitioner
B. Gerontology-certified staff RN
C. Certified medication aide
D. Licensed physical therapist

54. The parent of a teenage boy who has been admitted to a treatment center
because of drug and alcohol abuse tells the nurse, “Sometimes I feel like I hate
my own son.” Which response would be best for the nurse to provide?

A. “You may hate him now, but treatments has helped many drug and
alcohol abusers become wonderful individuals”
B. “There is nothing wrong with the way you feel. It is what you do with
these feelings that is important”
C. “Hating your own son will only make you more unhappy and result in
making you feel guilty”
D. “I don’t blame you. I would feel the same way if my son had destroyed
his life with drugs and alcohol”

55. The nurse observes the unlicensed assistive personnel (UAP) giving a bed bath
to a client who is unconscious. The bed is elevated to a high position and the
bed’s opposite side rail is raised. Which intervention should the nurse
implement?

A. Encourage the UAP to use a Hoyer lift to move the client


B. Tell the UAP to request assistance giving the bed bath
C. Take no action since the UAP is using proper technique to give a bed
bath
D. Demonstrate the correct way to give a bath to a client who is unconscious

56. A client with an exacerbation of systemic lupus erythematosus (SLE) is


admitted for parenteral corticosteroid therapy. What factor is most important for
the nurse to consider when assigning a room for this client? The client

A. Should not share a room with a client who has an infection…


B. Should share a room with a client with a similar cultural heritage
C. Should not share a room with a client who is immunocompromised
D. Needs a stimulating environment with sufficient lighting
57. A 2-year-old child with celiac disease experiences a relapse of symptoms. In
developing a teaching plan for the child’s family, which topic should be the
nurse’s primary focus?

A. Perianal skin care


B. Disease complications
C. Dietary management……”celiac” gluten free!!!
D. Chronic disease adaptation

58. The nurse is preparing to teach the parents of a child who had a surgical repair
of a myelomeningocele how to change an occlusive dressing on the child’s
back. Which statement by the parents indicates that they understand this
procedure?

A. “The purpose of the dressing is to protect the incision from fecal


contamination”
B. “The dressing will help dry the sutures for ease of removal”
C. “We should rapidly remove the tape from the edges of the dressing
when changing it”
D. “The dressing should be wetted periodically to keep the skin incision
moist”

59.The nurse is teaching a male client the self-care skills needed to deal with his
newly diagnosed chronic disease, hypertension. Which strategy is most likely to
promote the client’s commitment to needed lifestyle changes?

A. Emphasize the risks associated with noncompliance to the treatment


regimen
B. Provide clearly written and easily understandable materials to reinforce
the teaching session
C. Schedule multiple teaching sessions for the client to demonstrate his
psychomotor skills
D. Help the client identify ways in which these skills can benefit his
quality of life

60. An autopsy is needed based on what pathologic finding that supports the
diagnosis of Alzheimer’s disease?

A. Cerebral cortex micro-hemorrhages and infarcts destroy motor and


sensory functions
B. Amyloid B peptide neurofibrils in the neurons of the hippocampus
transmissions to the cortex
C. Thiamine deficiency alters short-term memory by short-circuiting neuron
transmission in the cortex
D. An intracranial shift occurs due to the accumulation of venous blood
below the dura mater

61. The nurse is reviewing a client’s record. What change in the client’s serum
laboratory values indicates an increased risk for impaired drug excretion?

A. Increased creatinine=kidney
B. Increased glucose
C. Decreased potassium
D. Decreased WBC count

62. Which foods are best for the nurse to offer a bipolar client who is in an acute
manic phase and is pacing in the hallway?

A. Chicken soup, lettuce salad, doughnut


B. Bologna sandwich, ear corn, candy bar
C. Steak, baked potato, apple pie
D. Hot dog, potato salad, white cake

63. An elderly male client reports to the clinic nurse that he is experiencing
increasing nocturia with difficulty initiating his urine stream. He reports a weak
urine flow and frequent dribbling after voiding. Which nursing action should be
implemented?

A. Instruct the client in effective techniques to cleanse the glans penis


B. Obtain a urine specimen for culture and sensitivity
C. Encourage the client to schedule a digital rectal exam
D. Advise the client to maintain a voiding diary for one week

64. The mother of an 8-month-old with a medical diagnosis of rotavirus tells the
nurse that her child has had watery diarrhea for about 36 hours. Which
assessment data supports the history provided by the mother?

A. Urine is dark amber in color, scanty amount


B. Crackles heard bilaterally in all lung field
C. Apical pulse rate of 120 beats/minute
D. Infant cries vigorously when picked up by the nurse

65. When planning care for a 56-year-old male client who is in respiratory distress,
the nurse knows that the standard treatment protocol is to administer oxygen at
4 L/minute. However, the nurse notes that a prescription for only 2 L/minute is
provided for this client. What action should the nurse take first?

A. Call the healthcare provider to verify the prescription


B. Administer oxygen by protocol at 4 L/minute
C. Check to see if the client has a history of COPD
D. Call the client’s family to see if the client is hypersensitive to oxygen

66. A one-year child with neuroblastoma is crying continuously and is curled into a
fetal position. What action is most important for the nurse to implement?

A. Ask the parent to rock the child


B. Administer diazepam (Valium)
C. Give a prescribed analgesic
D. Reduce environmental stimulation

67. A school-aged child who is recovering after an appendectomy is working a


crossword puzzle. According to Erikson’s theory of psychosocial development,
which stage is the child in at this time?

A. Initiative vs. guilt(3-6)


B. Autonomy vs. shame and doubt(1-3)
C. Identity vs. role confusion(ADOLECXENTE)
D. Industry vs. inferiority (6A 10) I&I

68. A nurse seeks to alter a provision of a state’s Nurse Practice regarding nurse-
client ratios, which the nurse believes to be unsafe. What action is most likely
to impact a rulling by the state’s Board of Nursing?

A. Send documentation of the problem to the American Nurse’s


Association
B. File a grievance at the medical center where the nurse is employed
C. Send a anonymous letter of concern to the local newspaper
D. Meet with the nurse’s representative to the state legislature
69. A client is receiving morphine sulfate 1 mg q10 minutes with a lockout dose of
24 mg per 4 hours via an intravenous patient-controlled analgesia (PCA) pump.
Which record, found on the PCA pump history, indicates that the client has
used the PCA pump effectively during the previous hour?
A. 12 attempts: 6 mg administered
B. 1 attempt: 10 mg administered
C. 5 attempts: 5 mg administered …..5-5
D. 4 attempts: 24 mg administered

70.A nurse-manager at a long-term care facility is concerned about the health of


the nursing staff. Which program should the nurse-manager institute first?

A. Place dental floss in the staff restrooms for employee use


B. Encourage staff to walk during breaks
C. Provide lift devices for immobile clients….Ha! our backs!!!
D. Arrange to start a Weight Watcher program at the facility

71. What explanation is best for the nurse to provide a preoperative client about the
purpose of an incentive spirometer?

A. Prevents collapse of the air sacs in the lungs


B. Helps reinflate the lungs after anesthesia
C. Promotes the removal of anesthesia from the lungs
D. Improves blood and oxygen supply to the tissues
72. A client is receiving an IV infusion of regular insulin, 50 units in 100 ml of
normal saline at 4 units/hour. The nurse should program the infusion pump to
deliver how many ml/hour?
DEVIDE 4 UNITS BY 50 THEN MULTIPLY BY 100 = 8
Answer: 8

73. A client is admitted to the rehabilitation center after having a stroke involving
the Broca’s area of the left cerebral cortex. Based on the location of this stroke,
which limitation should the nurse anticipate this client will have?

A. Problems with coordination and balance


B. Loss in ability to read and write
C. Difficulty in speech articulation….Bruh, Speak!!!
D. Problems with language comprehension

74.An infant is receiving penicillin G procaine (Wycillin) 180,000 units IM. The
drug is supplied as 600,000 units/ml. What volume in ml should the nurse
administer?

Answer: 0.3

75. Six hours after coronary artery bypass (CABG) surgery, the client has a blood
pressure of 90/60 mm Hg, pulse rate of 120 beats/minute, and urinary output of
100 ml since surgery. The nurse recognizes that this client is exhibiting
symptoms of which condition?
(Shock----BP lowers, HR decreases)
A. Acute pulmonary edema
B. Congestive heart failure
C. Mitral insufficiency
D. Cardiogenic shock
76. The nurse is assessing a 3-month-old infant who had a pulmonary yesterday.
This child should be medicated for pain on which findings? (Select all that
apply)

A. Increased respiratory rate


B. Restlessness
C. Knees drawn to abdomen
D. Increased pulse rate
E. Increased temperature
F. Peripheral pallor of the skin

77. In the differential diagnosis of delirium versus dementia, which assessment


finding supports a diagnosis of delirium?

A. The confusion started after admission to the hospital


B. The client’s attention span has not been adversely affected
C. The client’s memory has been markedly impaired
D. The confusion has been slowly developing over several months

78. A 65-year-old male client is admitted to the emergency department. He is


nonresponsive with dry mucous membranes and rapid breathing. Laboratory
results confirm diabetic ketoacidosis, and the nurse plans to administer
intravenous insulin. Which IV solution provides the best dilution for regular
insulin?

A. Normal saline ….0.9 “hypotonic”


B. Dextrose 5% in ½ normal saline
C. Lactated Ringer’s
D. Dextrose 5% in water

79. The nurse is feeding a client with Alzheimer’s disease when the client pushes
the food away and states, “Don’t do that! You’re making me mad.” What action
should the nurse implement?

A. Explain the importance of adequate nutrition to the client


B. Ask the healthcare provider about initiating enteral feedings
C. Encourage the client to eat only small portions of the meal
D. Return in 30 minutes to assist the client with the meal

80. A new mother tells the nurse that she does not want her newborn to receive any
immunizations. It is the hospital’s policy to routinely administer immunizations
to all newborns. What intervention should the nurse implement?

A. Administer the immunization after first explaining the hospital policy to


the mother
B. Screen the baby for immunization sensitivity before administering the
immunizations
C. Advise the mother to sign out of the hospitals AMA if wishing to refuse
the immunizations
D. Do not administer the immunizations and document that the mother
has refused permission

81. A client’s history indicates a subjective report of diminished sensory function.


Which assessment finding by the nurse supports that report?

A. Glasgow coma scale (GCS) score of 15


B. Negative 2 point tactile discrimination
C. Negative Babinski reflex
D. Patellar DRT (deep tendon reflex) +4

82. The mother of a 3-year-old asks the nurse to clarify the healthcare provider’s
diagnosis of acute otitis media. What is the most accurate explanation? “It is an
inflammation of the

A. Inner and outer ear”


B. Inner ear”
C. Middle ear”
D. Middle and outer ear”
83. The nurse is taking a health history a health history of a 46-year-old male client
who has smoked cigarettes for 30 years. He has had chronic bronchitis for the
past 6 months. What statement best describes the rationale for obtaining
information from the family as well from the client?

A. Including the family helps to ensure that the client will comply with the
treatment regime
B. Family members are usually more anxious than the client to get the
physical problem resolved
C. Poor oxygenation inhibits the client’s memory and renders information
unreliable
D. Clients tend to grow accustomed to their cough and underestimate
their nicotine use

84. The charge nurse is developing the nursing guidelines for a mental health care
unit. Which reference is likely to be the most useful in developing these
guidelines?

A. The Health Insurance Portability and Accountability Act


B. The American Nurse’s Association’s Standards of Practice
C. The Americans with Disability Act of 1990
D. The Patient’s Bill of Rights of 1990

85. The nurse is preparing a community education program and plans to provide
information about the importance of testicular self-examination for males. What
description of testicular cancer should the nurse include in the teaching plan?
This disease

A. Occurs in men of all ages, and available treatments have a low success
rate
B. Affects young adult males and needs to be treated promptly !!!
C. Usually occurs in middle-aged men and is slow growing
D. Usually occurs in men over 50 years of age and is associated with
prostate cancer
86. During a home visit, the nurse determines that a male client is experiencing
symptoms that should be controlled by his prescribed medication. The client
states that he forgot when he was supposed to take his medications. What is the
priority nursing diagnosis when the nurse develops the plan of care for this
client?

A. Family coping ineffective, related to medication regimen


B. Self-esteem disturbance related to physical symptoms of illness
C. Altered health maintenance related to lack of knowledge cos he forgt
D. Noncompliance related to medication administration

87.Furosemide (Lasix) is prescribed for a 4-year-old child who has a ventricular


septal defect. Which outcome indicates to the nurse that this pharmacological
intervention was effective?

A. Daily weight decreased from 47 pounds to 45 pounds


B. Serum BUN increased from 9 mg/dl to 14 mg/dl
C. Urine specific gravity increased from 1.02 to 1.03
D. Urinary output decreased from 25 ml/hr to 20 ml/hr

88. While changing a client’s postoperative dressing, the nurse observes purulent
drainage at the site. Before reporting this finding to the healthcare provider, the
nurse should note which of the client’s laboratory values?

A. Serum electrolytes
B. Platelet count
C. White blood cell count ….cos of purulent
D. Hemoglobin and hematocrit

89.A client with end-stage renal disease (ESRD) is experiencing systemic pruritis.
Which metabolic conditions are the main causes of the development of this
symptom?

A. Contact dermatitis and hypoparathyroidism


B. Metabolic alkalosis and hypophosphatemia
C. Uremic dermatitis and azotemia
D. Seborrheic dermatitis and hyperalbuminemia

90. Following a thoracentesis, what assessment finding indicates to the nurse that
the client is experiencing a complication of this procedure?

A. Asymmetry of respiratory movement


B. Increased pulse rate and blood pressure
C. Clear, watery mucus when coughing
D. Inability to demonstrate the cough reflex

91. The nurse anticipates the prescription of a reduced dosage of a nephrotoxic


medication for the client with which problem?

A. Subjective reports of dysuria with burning pain and cloudy amber urine
B. Diminished creatinine clearance found after 24-hour urine collection
C. Observable hematuria following a renal biopsy procedure
D. Documented presence of a kidney cyst found via ultrasound
92.A female client with fibromyalgia asks the nurse to arrange for hospice care to
help her manage the severe, chronic pain. Which interdisciplinary team member
should the nurse consult to assist the client?

A. Psychologist
B. Hospice nurse
C. Pharmacist
D. Pain specialist

93. When providing care for a group of clients, which client should the nurse
closely monitor for development of acute renal failure (ARF)? The client with

A. An anxiety disorder being treated with lorazepam (Ativan)


B. Peptic ulcer disease being treated with sucralfate (Carafate)
C. A resistant staphylococcus infection being treated with vancomycin
HCI (Vancocin)
D. Atrial fibrillation being treated with digoxin (Lanoxin)
94. A practical nurse (PN) and a registered nurse (RN) are the only two staff
members working the night shift on a small medical unit. The RN notes that the
call light of a client assigned to the PN is on and finds the PN asleep in the
break room. What action is best for the RN take?

A. Report the incident to the nursing supervisor


B. Awaken the PN to answer the client’s call light
C. Inform the PN that sleeping will not be tolerated
D. Advise the nurse to sleep more during the day

95. During a newborn home visit, the nurse observes cracked paint on the walls of
an older home. Siblings living in the home include a 1-year-old, a 2-year-old,
and a 4-year-old. Besides assessing the newborn, what other action should the
nurse take?

A. Perform an oral assessment on each child’s teeth and mouth


B. Obtain a blood sample to screen for lead poisoning in the older
children
C. Notify the Occupational Safety and Health Administration about the
paint
D. Determine the need for a referral to Child Protective Services

96.A client is hemiplegic following a cerebrovascular accident. To prevent this


client from experiencing a painful shoulder, what intervention should the nurse
include in the plan of care?

A. Position the affected arm on pillows while the client is seated in a


chair
B. Keep the client’s affected arm elevated above the level of the heart
C. Avoid range of motion exercises on the affected shoulder until pain in the
shoulder has passed
D. Exercise the affected shoulder by using it when assisting the client out of
bed
97. The nurse is planning care for a male client with a diagnosed personality
disorder. To effectively use the milleu for this client, which interventions
should the nurse include in this client’s plan of care? (Select all that apply)

A. Provide a structured daily routine


B. Ask the client what goals are reasonable for him to achieve while
hospitalized
C. Reinforce adaptive changes in his behavior
D. Determine what sports activities the client prefers
E. Clarify the consequence of his actions

98.The nurse is providing routine tracheostomy care for a client who has been
admitted with pneumonia. Place the following steps of the procedure in the
correct order of implementation.

1. Put on clean gloves


2. Discard soiled dressing
3. Prepare sterile supplies
4. Put on sterile gloves
5. Cleanse inner cannula with H2O2 sterile water
6. Replace twill tape and clean dressing

99. A mother calls the nurse to report that at 0900 she administered a PO dose of
digoxin (Lanoxin) to her 4-month-old infant, but at 0920 the baby vomited the
medicine. What instruction should the nurse provide to this mother?

A. Give half dose now and half in one hour


B. Give the infant another dose of Lanoxin
C. Mix the next dose with food to make it easier to take
D. Skip this dose and give the next dose on time

100. A client admitted to the hospital is suspected of having meningitis. The


nurse should plan to prepare the client for which diagnostic test?
A. Cervical x-rays
B. Synovial fluid analysis
C. Lumbar puncture
D. Electroencephalogram (EEG)

101. The nurse should question a prescription for docusate sodium (Colace) for a
client with which problem?

A. Two days following a knee replacement


B. History of liver disease
C. First day post myocardial infarction
D. Abdominal pain of unknown etiology

102. Following vaginal delivery in a birthing suite, the nurse assess a newborn
male and finds that his respirations are 58 breaths per minute and his hands and
feet are cyanotic. What action should the nurse take?

A. Transfer the infant to the nursery to determine his oxygen saturation rate
B. Record the findings and continue to observe the infant
C. Notify the pediatrician immediately
D. Administer oxygen at 5 L/minute

103. The nurse is providing intermittent gavage feedings for a 32-week


gestational age newborn. The nurse positions the newborn in a right side-lying
position with the head slightly elevated and passes the feeding tube through the
mouth. Prior to administering the bolus feeding, it is most important for the
nurse to obtain which assessment?

A. Gag reflex and vomiting


B. Stomach residual volume
C. Sucking on gavage tube
D. Nasal breathing obstruction
104. A postmenopausal client, who smokes a pack of cigarettes a day, is taking
estrogen (Premarin) daily. What instruction should the nurse provide to this
client?

A. Observe for swelling and calf pain


B. Do not stop the drug abruptly
C. Drink 8 glass of fluid daily
D. Stop the drug if a beta-blocker or thiazide diuretic is prescribed

105. The nurse is performing a routine examination of a 6-month-old girl at a


community health clinic. Records indicate that the child weighed 6.5 lbs at
birth. The clinic uses kg to describe weight. When assessing this child,
approximately what weight, in kg, should the nurse consider to be within
normal range for this child?

A. 6 to 7.5 kg
B. 15 to 18 kg
C. 12 to 15 kg
D. 9 to 11.5 kg
106. Two days following surgery, a bedfast male client demonstrates leg
exercises by tightening his thigh and pressing the back of his knee against the
mattress. What instruction should the nurse provide?

A. These range of motion exercises will help maintain joint function


B. This activity can break a blood clot loose and should not be performed
C. Do not perform this exercise while wearing antiembolism stockings
D. Continue to perform these exercise frequently while on bedrest

107. The nurse is conducting health assessments. Which assessment finding


increases a 56-year-old woman’s risk for developing osteoporosis?

A. Use of birth control pills until age 45


B. Family history of coronary heart disease
C. Obesity
D. Cigarette smoking
108. A male client psychosis explains that he was having thoughts that people are
trying to read his mind. He does not want to go back to his apartment because
he believes that someone is waiting there to kill him. What initial response is
best for the nurse to provide this client?

A. “These thoughts will go away if you take your medications”


B. “Have you been taking your medications every day as prescribed?”
C. “It must be frightening to feel that someone wants to hurt you”
D. Tell me about the people who were trying to hurt you”

109. The pharmacist enters the wrong dose of a medication when transcribing
prescriptions to a client’s medication administration record (MAR). Which
action should the nurse take to prevent a medication error from occurring?

A. Verify the room number on the medication administration record (MAR)


B. Compare the medication label with the medication administration record
(MAR)
C. Check the client’s identification bracelet prior to administering the
medication
D. Compare the medication administration record (MAR) to the
prescription

110. The nurse in the new newborn nursery admits a baby from labor and
delivery who is suspected of having a congenital heart disease. Which finding
helps to confirm this diagnosis?

A. Skin cracked, parchment-like, with desquamation


B. Tachypnea with centralized cyanosis when crying
C. Hands and feet cyanotic with lips and tongue pink
D. Heart rate of 160 and respiration rate of 48

111. Which finding would the nurse anticipate when assessing a client with
osteomalacia?
A. Flexion contractures
B. Fever
C. Joint tenderness
D. Pain on weight-bearing

112. The mother of an 11-year-old boy who has juvenile arthritis tells the nurse,
“I really don’t want my son to become dependent on pain medication, so I only
allow him to take it when he is really hurting”. Which information is most
important for the nurse to provide this mother?

A. Giving pain medication around the clock helps control the pain
B. Moist heat to the affected areas negates the need for pain medication
C. Encourage quiet activities such as watching TV to prevent pain
D. The child should be encouraged to rest when he experiences pain

113. A client with acute coronary syndrome (ACS) who is sleeping has been in
sinus rhythm with occasional premature ventricular contractions for the past 24
hours. What action should the nurse take when the monitor suddenly alarms and
shows irregular, wide, and erratic complexes?

A. Bring crash cart to the bedside


B. Initiate cardiopulmonary resuscitation
C. Attach automatic external defibrillator
D. Check placement of electrodes….cos client was sleping??

114. The nurse is caring for a client who is in the terminal stage of lung cancer
with metastasis to the pancreas. Which issue is most important for the nurse to
address when planning care for this client?

A. Client’s relationships with significant others


B. Adequate relief of pain and discomfort
C. Ability to ingest both liquid and solid foods
D. Skin care following palliative radiation therapy
115. The healthcare provider prescribes a low dose heparin protocol at 12
units/kg/hr for a client with a possible pulmonary embolism. This client weighs
144 pounds. The heparin solution contains 25,000 units in 250 ml DW. The
nurse should program the pump to deliver how many ml/hr?
Answer: 8

116. A male client calls the crisis center and tells the nurse that he wants to die
and is planning to commit suicide. What means of suicide should the nurse
determine is most lethal if in the client’s possession?

A. A loaded gun
B. A garden hose
C. Two bottles of Prozac
D. A bottle of an alcoholic beverage

117. When the nurse is preparing a client for surgery, what has the highest
priority in assessing a client’s readiness to receive a preoperative medication?
A. A family member is present
B. Dentures/prostheses are removed
C. Vital signs are documented
D. Surgical consent is signed

118. An 89-year-old male client complains to the nurse that people are
whispering behind his back and mumbling when they talk to him. What age-
related condition is likely to be occurring with this client?

A. Presbycusis(NO ESCUCHA BIEN )


B. Cerebral dysfunction
C. Presbyopia(NO VE BIEN )
D. Delirium

119. While obtaining a GI history on a frail elderly female client, the nurse learns
that she has dentures, lives alone, no longer drives, and is on fixed income. This
client has the highest risk for which problem?
A. Decreased ability to perform ADLs
B. Injury in the home
C. Nutritional deficit
D. Constipation

120. A public health nurse teaching a class on diabetes plan to discuss risk factors
for developing Type 2 diabetes. Which individuals has the greatest risk for
developing Type 2 diabetes?

A. A 72-year-old African-American who has a history of hypertension


B. A 24-year-old Caucasian male whose father is a Type 2 diabetic
C. A 48-year-old Hispanic female who is 5’2” and weighs 230 lbs
D. A 36-year-old Italian male who eats a diet high in carbohydrates

121. A female client reports feeling nervous and having a headache. When the
nurse assesses her blood pressure (BP) using an automatic blood pressure
apparatus, it fails to register because the BP is too high. What action should the
nurse take first?

A. Retake blood pressure in 30 minutes


B. Assess rate and rhythm of the client’s pulse
C. Take blood pressure on the other arm…..yeahhh smhhhh
D. Report symptoms to the healthcare provider

122. An adult client has been treated for hypovolemic shock for 3 hours. Which
findings indicate that the client is positively responding to treatment?

A. Cool, pale skin and SaO of 93%


B. Increased heart and respiratory rates
C. Increasing SVR and decreasing serum sodium
D. Decreasing lactate and increasing arterial pH

123. Elastic stockings have been prescribed for a client who is recovering from a
myocardial infarction. What is the best time to apply the stockings?
A. Mid-afternoon
B. Before bedtime
C. Noon time
D. Early morning

124. The nurse notices a reddened area on the coccyx of a wheelchair-bound


client. Which intervention should the nurse implement?

A. Carefully rewash the site and apply a DuoDerm patch


B. Ask the team leader to document the assessment findings
C. Provide a donut-shaped cushion for the client to use
D. Encourage the client to shift weight while sitting

125. Which method of anchoring an intravenous infusion catheter demonstrates


sound nursing judgment?

A. Secure the catheter and place a sterile, transparent dressing over the
skin insertion site
B. Wrap a strip of tape around the entire circumference of the arm for the
length of the catheter inserted
C. Elevate the hub of the catheter with a 2x2 gauze sponge, then tape the
catheter and tube securely on the top of the extremity
D. Use one strip of tape to secure the hub of the catheter and one strip of
tape to secure the tubing

126. An unresponsive female victim of a motor vehicle collision is brought to the


emergency department where it is determined that immediate surgery is
required to save her life. The client is accompanied by a close friend, but no
family members are available. What action should the nurse take?

A. Notify the unit manager that an emergency court order is needed to allow
the surgery
B. Continue to prepare the client for the surgery without a signed
informed consent
C. Ask the woman’s friend to sign the informed consent since the client is
unresponsive
D. Maintain continuous monitoring of the client until a family member can
be located

127. In developing a plan of care for a child with acute lymphocytic leukemia, the
nurse identifies the nursing diagnosis of, “Potential for injury related to
brushing and bleeding.” What laboratory finding provides supporting data for
this diagnosis?

A. Thrombocytopenia
B. Anemia
C. Neutropenia
D. Leucopenia

128. A female client complains that she cannot sleep, cries much of the day, and
is unable to work. Her healthcare provider diagnosis her as depressed and
prescribes monoamine oxidase (MAO) inhibitors. In preparing a teaching plan,
what foods should the nurse instruct this client to eliminate from her diet?

A. Carbonated beverages, eggs, and alcohol


B. Salty foods such as chips, and chocolate
C. Fruits with a high acidity such as grapefruit and oranges
D. Cheese, beer, and avocados

129. Which nursing diagnosis is best to formulate for a 76-year-old client who is
exhibiting an external locus of control?

A. Powerlessness
B. Hopelessness
C. Social isolation
D. Personal identity disturbance
130. A 10-month-old girl is admitted with a diagnosis of possible cystic fibrosis.
What question should the nurse ask the parents of this child in the diagnosis of
cystic fibrosis?

A. “Is she ever constipated”


B. “Does her urine have a musty odor?”
C. “Does she taste salty when you kiss her?”
D. “What is her daily intake of milk?”

131. An emergency room nurse is caring for a client with a possible abdominal
injury. Grey turner’s sign (ecchymosis in the flank area) is noted. What should
the nurse suspect from this finding?

A. Retroperitoneal bleeding
B. Early disseminated intravascular coagulation
C. Abdominal mesenteric artery occlusion
D. A femoral vein thrombosis

132. When assessing a client the first postpartum day, the nurse finds a moderate
amount of lochia rubra, with the uterus firm, dextroverted, and three
fingerbreadths above the umbilicus. What action should the nurse take first?

A. Check the hemoglobin to determine uterine hemorrhage


B. Massage the uterus to decrease atony
C. Assess the bladder for distension
D. Provide a stool softener for constipation

133. The nurse is assessing the nutritional status of several infants. Based on date
obtained while taking a history, which infant’s family will need additional
nutritional guidance?

A. A 12-month-old whose mother is giving finger foods


B. A 6-month-old whose diet includes rice cereal, fruit and breast milk
C. An 8-month-old whose mother is starting to introduce formula in a cup
D. A 10-month-old who takes 40 ounces of formula
134. A 14-year-old girl with asthma complains of feeling nervous and jittery after
a respiratory therapy bronchodilator treatment. What explanation is best for the
nurse to provide to this adolescent?

A. The nervousness is due to the asthma and hypoxia, but should disappear
after several bronchodilator treatments
B. Rapid dilation of the bronchioles and increased heart rate may cause
nervousness and jitteriness
C. The bronchodilator treatment contained albuterol, which can cause a
fast heart rate and jitteriness
D. Bronchodilators may produce excessive coughing, which can contribute
to tachypnea and anxiety

135. When the nurse prepares to administer a pain medication to a child, the
mother states that she does not want her child to have any more narcotics. What
action should the nurse implement first?
A. Document the mother’s refusal of the medication in the medical record
B. Ask the mother to clarify what she understands about the medication
C. Explain that regularly administered analgesics help improve pain control
D. Notify healthcare provider of the mother’s refusal of the medication

136. Which technique should the nurse use to assess a client’s eyes for
nystagmus?(POCO MOVIMIENTO DE LOS OJOS )CANCER

A. Use an ophthalmoscope to examine the retinal structures


B. Inspect the bulbar and palpebral portions of the conjunctiva
C. Ask the client to hold a brief gaze in specified positions
D. Compare size, shape, and reaction to light of both pupils

137. The nurse notes a new prescription for linezolid (Zyvox) IV for a client with
nosocomial pneumonia due to methicillin resistant staphylococcus aureus
(MRSA), as reported by the findings of the sputum culture and sensitivity. The
nurse also notes that the client is allergic to cephalosporins. What action should
the nurse implement?
A. Consult with the pharmacist regarding the prescription
B. Prepare to administer the medication as prescribed…yea cos is not
cillin
C. Notify the healthcare provider of the client’s allergy
D. Review the culture report with the healthcare provider

138. The charge nurse in a critical care unit is reviewing client’s conditions to
determine who is stable enough to be transferred. Which client status report
indicates readiness for transfer from the critical care unit to a medical unit?

A. Pulmonary embolus with an intravenous heparin infusion and new onset


hematuria
B. Myocardial infarction with sinus bradycardia and multiple ectopic beats
C. Adult respiratory distress syndrome with pulse oximetry of 88%
saturation
D. End-stage renal failure with creatinine of 2.5 mg/dl and urinary
output of 10 ml/hr

139. The nurse includes the diagnosis, “Impaired mobility related to weakness
and fear of falling” in the plan of care of a postoperative client. Which goal
should be added to the care plan o address this diagnosis? The client will

A. Be instructed in safety measures


B. Not fall during the hospital stay
C. Report any weakness to the nurse
D. Demonstrate increased mobility

140. Following a CVA, the nurse assesses that a client has developed dysphagia,
hypoactive bowel sounds and a firm, distended abdomen. Which prescription
for the client should the nurse question?

A. Metoclopramide (Reglan) intermittent IV piggyback


B. Continuous tube feeding at 65 ml/hour via gastrostomy…..Continous
tube feeding with a distended abdomen??????...ridiculous!
C. Total parenteral nutrition to be infused at 125 ml/hour
D. Nasogastric tube connected to low intermittent suction

141. In caring for a client with laryngitis, the nurse observes that the client has a
frequent, dry cough while conversing with family members. The client also
reports experiencing dysphagia due to pain. What action should the nurse
implement?

A. Advise the client to restrict intake of oral liquids


B. Apply a cold compress to the client’s throat
C. Instruct the client to restrict conversations
D. Encourage the client to use the incentive spirometer

142. A client is diagnosed with an anxiety disorder. According to behavioral


therapy, which cognitive restructuring intervention should the nurse recommend
when the client is addressing anxiety-producing situations?

A. Take an anti-anxiety medication prophylactically


B. Call a friend to discuss the anxiety provoking situation
C. Try to avoid situations that cause the anxiety
D. Recite a favorite poem when feeling anxious

143. A client is undergoing intracranial surgery. What intervention to decrease


periorbital edema should the nurse include in this client’s plan of care?

A. Put the client in a trendelenburg position


B. Apply moist heat over the eyes
C. Patch both eyes for 36 hours
D. Apply light, cold compresses over the eyes

144. What intervention should the nurse implement during the administration of a
vesicant chemotherapeutic agent via an IV site in the client’s arm?

A. Administer a topical anesthetic to reduce pain and burning at the site


B. Monitor radial pulse distal to the IV site regularly
C. Instruct the client to expect temporary burning at the IV site
D. Assess the IV site regularly for signs of infiltration

145. When caring for a client who has a pulmonary artery catheter in place,
which observation warrants immediate intervention by the nurse?

A. The wave form indicates the catheter is in the right ventricle


B. The client is experiencing isolated unifocal PVCs
C. The pulmonary capillary wedge pressure (PCWP) is 8
D. The pulmonary artery pressure (PAP) is 20/10

146. The nurse is planning care for a client who admits having suicidal thoughts.
Which client behavior indicates the highest risk for the client acting on these
suicidal thoughts?

A. Begins to show signs of improvement


B. Describes being very depressed
C. Has a appetite and neglects personal hygiene
D. Is not interested in the activities of family and friends

147. After administering an 8 ounce can of nutritional supplement and two


teaspoons of medication, the nurse should document the client’s fluid intake as
how many ml?
Answer: 250

148. The nurse identifies, “Altered sleep patterns related to hot flashes” as a
priority diagnosis for a female client during perimenopause. After
implementing the plan of care, which documentation indicates a successful
outcome?

A. Room temperature reduced to manage symptoms


B. Client slept through the night
C. Client maintains normal body temperature
D. Hypnotic medication administered at bedtime
149. The nurse is teaching a course on care of the elderly to unlicensed assistive
personnel (UAP)s. In teaching about confusion in the elderly, which 80-year-
old client is at highest risk for the onset of acute delirium? A client who

A. Was discharged home following unilateral cataract surgery


B. Was just transferred to a rehabilitation unit one week after hip
arthroplasty
C. Has a spouse just admitted to a critical care unit
D. Is one day postoperative an emergency colon resection….patients are
usually confused after one day post op. Is why we usually sit them

150. Low molecular weight heparin therapy is prescribed for a client following a
thrombolytic stroke. What precaution should the nurse take during
anticoagulation therapy?

A. Monitor daily international normalized ratio (INR) values


B. Administer the medication in a large muscle group
C. Keep protamine sulfate available as a reversal agent
D. Teach the client to limit intake of foods rich in vitamin K

151. The nursing staffs of a medical unit are asked to make recommendations
regarding the installation of computer workstations on the unit. Which factors
should the staff consider as a priority to ensure effective ergonomics?

A. Height of the countertop and available lighting in the area


B. Distance to client’s rooms and the number of chairs available
C. Availability of the chart rack and a dictation area for healthcare providers
D. Location of the elevators relative to the entrance of the nursing station

152. Repeated from Version 1

153. A male client with diabetes mellitus reports that he has had trouble
following his diet, and the result of his fasting blood glucose test is 90 mg/dl.
What action should the nurse implement first?
A. Obtain a urine specimen from the client to test for ketonuria
B. Assure the client that his diabetes control is within normal limits
C. Review the findings of his glycosylated hemoglobin test
D. Scheduled the client to attend classes about diet management

154. Initial assessment by the nurse of a client who is admitted to the Emergency
Center following a boating accident indicates that the client has chest wall
bruising with crepitus, shortness of breath, and a respiratory rate of 40
breaths/minute. Which assessment finding requires the most immediate
intervention by the healthcare provider?
A. Distension of bilateral neck veins
B. Tracheal deviation to the left of the midline
C. Paradoxical movement of the chest wall
D. Diminished breath sounds over the right lung field

155. After receiving a telephone prescription for a medication to be administered


today, in what sequence should the nurse perform these tasks? (Arrange from
first to last)

1. Write down the prescription as stated


2. Confirm the accuracy of the prescription
3. Administer the medication as prescribed
4. Ensure the prescription is signed by the prescriber

156. The client in which situation requires the most immediate nursing
intervention?

A. History of multiple sclerosis, experiencing an acute relapse


B. Recent onset of migraines, experiencing vomiting and tinnitus
C. Previously diagnosed with epilepsy, experiencing status epilepticus
D. Diagnosed with Parkinson’s disease, with new onset dyskinesia

157. The nurse observes nonverbal cues that indicate a preoperative client does
not have sufficient knowledge about the impending surgery. What action should
the nurse take?
A. Notify the surgeon that the client needs further teaching
B. Wait the client to verbalize any questions and concerns
C. Determine if the client has signed the informed consent form

158. The nurse is evaluating the effectiveness of a client’s plan of care prior to a
client’s discharge. Which action has the highest priority?

A. Determine which interventions were effective


B. Review the effect of medical treatment on the care plan
C. Measure the length of time needed to complete the plan of care
D. Establish whether the goal was achieved

159. Which technique should be used to obtain a sterile urine specimen using a
straight catheter?

A. Use a sterile syringe to obtain the specimen


B. Drain the urine from the collection bag into a sterile container
C. Discard the first specimen, clamp the catheter, then collect the next
specimen
D. Drain the urine from the catheter into a sterile container

160. Thirty minutes after a teen-age girl is transferred to the unit following the
delivery of a stillborn infant, the nurse finds the teen joking and laughing with
her boyfriend and other friends. How should the nurse respond?

A. Accept that laughter may be this young woman’s method of coping


B. Determine if the joyful behavior is related to ending an unwanted
pregnancy
C. Ask the friends to leave so that the couple may experience their grief
D. Tell the group of teenagers that now is the time to talk about their loss

Cat Version 4
1- A 59-year-old male client is brought to the emergency room where he is assessed to have a
Glasgow Coma Scale of 3. Based on this assessment, how should the nurse characterize
the client’s condition?
a. The client has increased intracranial pressure
b. He has a good prognosis for recovery
c. This client is conscious, but is not oriented to time and place
d. He is in a coma, and has a very poor prognosis

Correct D

2- At a community health fair, a 50-year-old woman tells the nurse that she has an annual
physical exam that includes a clinical breast exam and an annual mammogram. How should the
nurse respond?

a. Encourage the woman to explore her fears about breast cancer.


b. Ask the woman if she also performs monthly breast self- exams.
c. Commend the woman for adhering to the recommended cancer detection guidelines.
d. Advise the woman that mammograms are only needed every two years at her age.

Correct B

3- Which assessment finding should indicate to the nurse that a client with arterial hypertension
is experiencing a cardiac complication?
a. Complaints of an occipital headache
b. A palpable dorsal is pedis pulse bilaterally
c. Complaints of shortness of breath on exertion
d. A blood pressure of 160/90

Correct C

4- A college student who is diagnosed with a vaginal infection and vulva irritation describes
the vaginal discharge as having a “cottage cheese” appearance. Which prescription should
the nurse implement first?
a. Cleanse perineum with warm soapy water 3 times per day

b. Instill the first dose of nystatin (Mycostatin) vaginally per applicator

c. Perform glucose measurement using a capillary blood sample

d. Obtain a blood specimen for sexually transmitted disease (STDs)


Correct B

5- A client in acute renal failure has serum potassium of 7.5 mEq/L. Based on this finding, the
nurse should anticipate implementing which action?
a. Administer an IV of normal saline rapidly and NPH insulsubcutaneously.
b. Administer a retention enema of Kayexalate
c. Add 40 mEq of KCL (potassium chloride) to present IV solution.
d. Administer a lidocaine bolus IV push.

Correct B

6- A client who had an intraosseous (IO) cannula placed by the healthcare provider
for an emergent fluid resuscitation is complaining of severe pain and numbness below the IO sit.
The skin around the site is pale and edematous. What action should the nurse takes first?

a. Discontinue the IO infusion


b. Administer an analgesic via the IO site
c. Elevate the extremity with the IO site
d. Notify the healthcare provider

Correct A

7- The nurse-manager of a perinatal unit is notified that one client from the medical-surgical
unit needs to be transferred to male room for new admissions. Which client should the
nurse recommend for transfer to the antepartal unit?
a. A 45-year-old who has a chronic hepatitis B.
b. A 35-year-old with lupus erythematous
c. A 19-year-old who is diagnosed with rubella
d. A 25-year-old with herpes lesions of the vulva

Correct B

8- A nurse is teaching a client postoperative breathing techniques using an incentive


spirometer (IS). What should the nurse encourage this client to do to maintain sustained
maximal inspiration?
a. Breathe into the spirometer using normal breath volumes
b. Exhale forcefully into the tubing for 3 to 5 seconds
c. Inspire deeply and slowly over 3 to 5 seconds
d. Perform IS breathing exercises every 6 hours

Correct C

9- The nurse plans to educate a client about the purpose for taking the prescribed
antipsychotic medication clozapine (Clozaril). Which statement should the nurse
provide?

a. You will be able to cope with your symptoms


b. It will help you function better in the community
c. The medication will help you think more clearly”
d. It will improve your grooming and hygiene

Correct C

10 – A male client with diabetes mellitus takes Novolin 70/30 insulin before meals and
azithromycin (Zithromax) PO daily, using medication he brought from home. When the nurse
delivers his breakfast tray, the client tells the nurse that he took his insulin but forgot to take his
daily dose of the Zithromax an hour before breakfast as instructed. What action should the
nurse implement?

a. Offer to obtain a new breakfast tray in an hour so the client can take the Zithromax
b. Instruct the client to eat his breakfast and take the Zithromax two hours after
eating
c. Tell the client to skip that day’s dose and resume taking the Zithromax the next day
d. Provide a PRN dose of an antacid to take with the Zithromax right after breakfast

Correct B

11-What instruction is most important for the nurse to provide a female client who has just been
diagnosed with Trichomoniasis?

a. Avoid douching
b. Treat sexual partner (s) concurrently
c. Avoid using moist washcloths when bathing
d. Postpone becoming pregnant until the infection is treated

Correct B
12- A primigravida at term comes to the prenatal clinic and tells the nurse that she is having
contractions every 5 min. The nurse monitors the client for one hour, using an external fetal
monitor, and determines that the client’s contractions every 5 minutes. The nurse monitors the
client for one hour, using an external fetal monitor, and determines that the client’s contractions
are 7 to 15 minutes apart, lasting 20 to 30 seconds, with mild intensity by palpation. What action
should the nurse take?

a. Tell the client to go directly to the hospital for admission to labor and delivery for active
labor
b. Send the client home and instruct her to call the clinic when her contractions occur 5
minutes apart for one hour
c. Tell the client to check into the hospital within the next hour for evaluation of possible
urinary tract infection

Correct B

13- Which instruction should the nurse provide to an elderly client who is taking an ACE inhibitor
and a calcium channel blocker?

a. Wear long-sleeved clothing when outdoors


b. Report the onset of sore throat
c. Eat plenty of potassium-rich food
d. Change the position slowly

Correct D

14- Assessment finding of a 3-hour-old newborn include: axillary temperature of 97.7 F, heart
rate of 140 beats/minute with a soft murmur, and irregular respiratory rate at 42 breaths/min.
Based on these findings, what action should the nurse implement?

a. Place a pulse oximeter on the heel


b. Swaddle the infant in a warm blanket
c. Record the findings on the flow sheet
d. Check the vital signs in 15 minutes

Correct C

15- A client admitted to the hospital for depression is escorted to a private room. Prior to leaving
the room, what intervention is most important for the nurse to implement?
a. Explain the programs guidelines

b. Search all personal belongings

c. Initiate psychosocial assessment

d. Review the healthcare’s provider’s prescription

Correct B

16- An experienced nurse tells the nurse-manager that working with a new graduate is
impossible because the new graduate will not listen to suggestions. The new graduate comes to
the nurse-manager describing the senior nurse’s attitude as challenging and offensive. What
action is best for the nurse manager to take?

a. Have both nurses meet separately with the staff mental health consultant
b. Listen actively to both nurses and offer suggestions to solving dilemma
c. Ask the senior nurse to examine mentoring strategies used with the new graduate
d. Ask the nurses to meet with the nurse-manager to identify ways of working together

Correct D

17- Which nursing diagnosis has the highest priority when planning care for a client in
cardiogenic shock?

a. Risk for imbalance body temperature


b. Excess fluid volume
c. Fatigue
d. Ineffective Tissue Perfusion

Correct D

18- A client who had a cerebral vascular accident (CVA)

Is paralyzed on the left side of the body and has developed a Stage II pressure ulcer on the left
hip. Which nursing diagnosis describes this client’s current health status?
a. Risk for impaired tissue integrity related to impaired physical mobility
b. Impaired skin integrity related to altered circulation and pressure
c. Ineffective tissue perfusion related to inability to move self in bed.
d. Impaired physical mobility related to the left side paralysis

Correct B

19- The nurse offers diet teaching to a female college student who was diagnosed with iron-
deficiency anemia following her voluntary adoption of a lacto-vegetarian diet. What nutrients
should the nurse suggest this client eat to best meet her nutritional needs while allowing her to
adhere to a lacto-vegetarian diet?

a. Drink whole milk instead of skim milk to enhance the body’s production of amino acids
b. Take vitamin K 10mg PO daily to enhance production of red blood cells
c. Increase amounts of dark yellow vegetables such as carrots to fortify iron stores
d. Combine several legumes and grains such as beans and rice to form complete
proteins

Correct D

20- The nurse is triaging clients from a train wreck. A client has multiple open wounds, a blood
pressure of 90/56, and a pulse of 112 beats/minute. Which triage tag color should the nurse
place on this client?

a. Black
b. Yellow
c. Green
d. Red….open wound! Wound is red.

Correct D

21- Which action should the nurse include in the plan of care a client who is receiving acyclovir
(Zovirax) IV for treatment of herpes zoster (shingles)?

a. Initiate cardiac telemetry monitoring


b. Maintain continuously pulse oximetry
c. Perform capillary glucose measurements
d. Monitor serum creatinine levels
Correct D

22- A client receiving amlodipine (Norvasc), a calcium channel blocker, develops 1+ pitting
edema around the ankles. It is most important for the nurse to obtain what additional client
data?

a. Bladder distention
b. Serum albumin level
c. Abdominal girth
d. Breath sounds

Correct D

23- A male adult client is transferred to a psychiatric facility following release from the hospital
for treatment of a self-inflicted gunshot wound. In attempting to develop a therapeutic
relationship with this client, which information is most important for the nurse to determine?

a. The family’s reaction to this situation


b. The nurse’s feeling about this client
c. What losses the client recently experienced
d. Why the client attempted to kill himself

Correct B

24-Which client requires careful nursing assessment for signs and symptoms of
hypomagnesemia?

a. A young adult client with intractable vomiting from food poisoning


b. A client who developed hyperparathyroidism in late adolescence
c. A middle-age male client in renal failure following an unsuccessful kidney transplant
d. A female client who is overzealous with her intake of simple carbohydrates

Correct C

25- While assessing a client who is experiencing Cheyne-Stokes respirations, the nurse
observes periods of apnea. What action should the nurse implement?

a. Elevate the head of the client’s head


b. Auscultate the client’s breath sound
c. Measure the length of the apneic periods(longitude)
d. Suction the client’s oropharynx

Correct C

26- The nurse is preparing to administer the 0800 dose of 20 units of Humulin R to an 8-year-
old girl diagnosed with Type 1 diabetes. The mother comments that her daughter is a very picky
eater and many times does not eat meals. Which intervention should the nurse implement first?

a. Administer the 20 Units of Humulin R subcutaneously as prescribed


b. Ask the girl if she will be eating her breakfast this morning
c. Discuss changing the insulin prescription to Lispro with the healthcare provider
d. Explain to the mother the importance of eating the schedule meals

Correct B

27- The nurse working in an emergency center collects physical evidence 6 hours following a
reported sexual assault. After placing the samples in sealed containers, which action is most
important for the nurse to implement?

a. Maintain possession of the evidence collection kit at all times until submitted to law
enforcement
b. Provide discharge instruction for prophylactic antibiotic, pregnancy, and HIV prevention
medication
c. Document the characteristics of the various sites of sample collection
d. Assist the client with toileting, hygiene, and dressing with clean clothes.

Correct A

28- The nurse is caring for a 10-year-old who is diagnosed with acute glomerulonephritis. Which
outcome is the priority of this child?

a. Activity tolerance as evidenced by performing appropriate age-level-activities


b. No signs of skin breakdown as evidenced by intact skin and no redness
c. Adequate nutritional status as evidenced by no weight gain or loss
d. Fluid balance maintained as evidenced by a urine output of 1 to 2 ml/kg/hr

Correct D(repetida)
29- A 20-year-old male client is diagnosed with Ewing’s sarcoma following examination for a
knee injury. Which instruction is most important for the nurse to provide the client?

a. Take analgesics regularly to reduce the pain


b. Notify the healthcare provider if the swelling worsens
c. Avoid weight-bearing until the injury heals
d. Seek treatment for the sarcoma immediately

Correct D

30- The nurse in the newborn nursery admits a baby from labor and delivery who is suspected
of having a congenital heart disease. Which finds helps to confirm this diagnosis?

a. Pink lips and tongue with cyanotic hands and feet


b. Respiration rate of 40 and heart rate of 144
c. Centralized cyanosis and tachycardia when crying
d. Desquamation from areas of cracked, parchment-like skin

Correct C

31- A postoperative client returns to the nursing unit following a ureter lithotomy via a flank
incision. Which potential nursing problem has the highest priority when planning nursing care for
this client?

a. Ineffective airway clearance…..Airway


b. Altered nutrition less than body requirements
c. Fluid volume excess
d. Activity intolerance

Correct A

32- A client who is bleeding after a vaginal delivery receives a prescription for methylergonovine
(Methergine) 0.4 mg IM every 2 hours, not to exceed 5 doses. The medication is available in
ampules containing 0.2 mg/ml. what is the maximum dosage in mg that the nurse should
administer to this client? (Enter numeric value only)

Ans: 2

0.4 mg/dose = x mg/5 doses


X= 2 mg

33- A male client admitted three days ago with respiratory failure is intubated and with 40%
oxygen per facemask is initiated. Currently his temperature is 99 F, capillary refill is less than 4
seconds, and respiratory effort is within normal limits. What outcome should the nurse evaluate
to measure for successful estuation?

a. Exhibits adequate tissue perfusion


b. Remains free of injury
c. Remains free of infection
d. Maintains effective breathing pattern…..priority

Correct D

34- When the nurse enters the room to change the dressing of a male client with cancer, he
asks, “Have you ever been with someone when they died?” What is the nurse’s best response
to him?

a. “Yes I have. Do you have some questions about dying?”


b. “Several times. Now, let’s get your dressing changed”
c. “A few times. It was peaceful and there was no pain”
d. “Yes, but you’re doing great. Are you concerned about dying?”

Correct A

35- A client with a C-6 spinal injury changes to a breathing pattern of shallow respirations and
dyspnea twelve hours after the causative incident. The nurse should notify the healthcare
provider and implement which intervention?

a. Put the client in reverse Trendelenburg position


b. Prepare for intubation with an endotracheal tube
c. Administer a pain medication to the client
d. Instruct the client on deep breathing exercises

Correct B

36- When caring for a laboring client whose contractions are occurring every 2 to 3 min, the
nurse should document that the pump is infusing how many ml/hr? (Enter numeric value only. If
rounding is required, round to the nearest whole number. Click on each chart tab for additional
information. Please be sure to scroll to the bottom right corner of each tab to view all information
contained in the client’s medical record.

Ans: 42

37- When caring for a laboring client whose contractions are occurring every 2-3 minutes, the
nurse should document that the pump is infusing how many ml/hour? (Enter numeric value only.
If rounding is required, round to the nearest whole number. Click on each chart tab for additional
information. Please be sure to scroll o the bottom right corner of each tab to view all information
contained in the client’s medical record.

ANS 5

38- The nurse in a community health clinic is interviewing a female client who has tree children.
The client tells the nurse that she has a new man in her life, with whom she is having a sexual
relationship, and that they both smoke cigarettes. Which information is most important for the
nurse to provide this client?

a. Oral contraceptives should be started to prevent an unwanted pregnancy


b. Children have more upper respiratory infections if exposed to smoke at home
c. Cigarette smoking increases the risk for peptic ulcers and emphysema
d. A diaphragm and condom provide effective contraception when used together

Correct D

39- An adult male is admitted to the psychiatric unit from the emergency department because
he is in the manic disorder. He has lost 10 pounds in the last two weeks and has not bathed in a
week because he has been “trying to start a new business” and is “too busy to eat.” He is alert
and oriented to time, place and person, but not situation. Which nursing diagnosis has the
greatest priority?

a. Self-care deficit
b. Disturbed sleep pattern
c. Disturbed thought processes
d. Imbalanced nutrition

Correct D
40- A 9 year-old received a short arm cast for a right radius. To relieve itching under the child’s
cast, which instructions should the nurse provide to the parents?

a. Blow cool air from a hair dyer under the cast


b. Twist the cast back and forth
c. Shake powder into the cast
d. Push a pencil under the cast edge

Correct A

41- A client in the oliguric phase of acute renal failure (ARF) has a 24-hour urine output of 400
ml. The nurse should allow this client to have how much oral intake during the next 24 hours?

a. Encourage oral fluids as tolerated


b. Decrease oral intake to 200 ml
c. Allow the client to have exactly 400 ml oral intake
d. Limit oral intake to 900 to 1,000 ml

Correct D

42- A female client on the mental health unit tells the nurse that her roommate is sitting on the
bathroom floor with superficial cuts on her wrists. The nurse cleans and assesses the client’s
wrists and asks what happened. She doesn’t respond. What should the nurse do next?

a. Find supplies to put a dressing on the client’s wrist


b. Take the client to a room for supervision by staff
c. Call the healthcare provider to report the client’s behavior
d. Go find a staff to stay in the room with the client

Correct B

43- What assessment technique should the nurse use to monitor a client for a common
untoward effect of phenytoin (Dilantin)?

a. Bladder palpation
b. Inspection of the mouth
c. Blood glucose monitoring
d. Auscultation of breath sounds

Correct B
44- The nurse is assessing on the first postoperative day following thyroid surgery. Which
laboratory value is most important for the nurse to monitor?

a) Calcium….because T for twitching so


b) Sodium
c) Chloride
d) Potassium

Correct A

45- The nurse is preparing to administer an IM dose of vitamin B1 (Thiamine) to a male client
experiencing acute alcohol withdrawal and peripheral neuritis. The client belligerently states,
“What do you think you’re doing?” How should the nurse respond?

a.” I cannot give you this medication until you calm down”

b. “This shot will help relieve the pain in your feet.”

c. “Would you prefer to learn to administer your own shot?”

d. “You will feel calmer and less jittery after this shot”

Correct B

46- When administering an intramuscular injection containing 3 ml of a painful medication,


which intervention should the nurse implement?

a. Instill the medication quickly


b. Insert the needle slowly
c. Select a large, deep muscle mass….yeahhhh
d. Use a short small gauge needle

Correct C

47- Several clients on a telemetry unit are schedule for discharge in the morning, but a
telemetry-monitored bed is needed immediately. The charge nurse should make arrangements
to transfer which client to another medical unit? The client who is
a. Learning to self-administer insulin injections after being diagnosed with diabetes
mellitus….. I meannn
b. Ambulatory following coronary artery bypass graft surgery performed six days ago.
c. Wearing a sling immobilizer following permanent pacemaker insertion earlier that day
d. Experiencing syncopal episodes resulting from the dehydration caused by severe
diarrhea

Correct A

48- The nurse preceptor is orienting a new graduate nurse to the critical care unit. The
preceptor asks the new graduate to state symptoms that most likely indicate the beginning of a
shock state in a critically ill client. What findings should the new graduate nurse identify?

a. Tachycardia, mental status change, and low urine output


b. Warm skin, hypertension, and constricted pupils
c. Bradycardia, hypotension, and respiratory acidosis
d. Mottled skin, tachypnea and hyperactive bowel sounds

Correct A

49- The nurse is making assignments for a new graduate from a practical nursing program that
is orienting to the unit. Because the unit is particularly busy this day, there will be little time to
provide supervision of this new employee. Which client is the best for the nurse to assign this
newly graduate practical nurse? A client

a. Whose discharge has been delayed because of a postoperative infection


b. With poorly controlled type 2 diabetes who is sliding scale for insulin administration
c. Newly admitted with a head injury who requires frequent assessments
d. Who is receiving IV heparin that is regulated based on protocol

Correct A

50- Nurses working in labor and delivery are demanding a change in policy because they
believe they are required to float more often than nurses on other units. However, floating to
labor and delivery is not reciprocated because other nurses are not competent to provide highly
specialized obstetrical skills. What action is best for the nurse-manager to implement?

a. Require the other nurses to cross-train for obstetrics


b. Propose a method for self-staffing labor and delivery
c. Remind nurses that floating is an administrative policy
d. Encourage nurses to share their feelings with administration
Correct B

51- Locate the optic disk. (Click the chosen location. To change, click on the new location.)
(union de venas en el ojo)

52- The nurse has explained safety precautions and infant care to a primigravida mother and
observes the mother, as gives care to her newborn during the first two days of rooming-in.
Which action indicates the mother understand the instruction?

a. Aspirates the newborn’s nares using syringe


b. Applies a dressing on the cord after the newborn’s bath
c. Breastfeeds the infant every hour during the night
d. Positions the infant supine in the crib to sleep(cuna)

Correct D

53- The nurse teaches the mother of a 6 year-old anemic boy to give iron supplements. Which
statement indicates that the mother understands the proper administration of iron?

a. “The iron tablets will be absorbed between meals, on an empty stomach”


b. “I should give the iron tablets with his milk and cereal each morning”
c. “Iron preparations can be taken with antibiotics if he develops an infection”
d. “The iron tablets may cause him to sunburn more easily so he should wear sunscreen”

Correct A

54- “Oxygen at liters/min per nasal cannula PRN difficult breathing” is prescribed for a client with
pneumonia. Which nursing intervention is effective in preventing oxygen toxicity?

a. Avoiding the administration of oxygen at high levels for extended periods.


b. Administering a sedative at bedtime to slow the client’s respiratory rate
c. Removing the nasal cannula during the night to prevent oxygen buildup
d. Running oxygen through a hydration source prior to administration
Correct A

55- The healthcare provider prescribes amoxicillin (Amoxil) 1.5 grams PO daily, in equally
divided doses to be administer every 8 hours. The medication is available in a bottle labeled,
“Amoxicillin (Amoxil) suspension 200 mg/5 ml.” How many ml should the nurse administer every
8 hours? (Enter numeric value only. If rounding is required, round to the nearest tenth.)

Ans: 12.5

56- A 20-year-old female client tells the nurse that her menstrual periods occur about every 28
days, and her breasts are quite tender when her menstrual flow is heavy. She also states that
she performs her breast self-examination (BSE) on the first day of every mouth. What action
should the nurse implement in response to the client’s statements?

a. Remind the client that is also important to schedule an annual mammogram


b. Refer the client to a nurse practitioner for an in-depth review of the BSE procedure
c. Encourage the client to perform BSE 2 to 3 days after menstrual period ends
d. Instruct the client to continue with her regular monthly exams as she is doing

Correct C

57- A 2-year-old boy has short bowel syndrome has progressed to receiving enteral feedings
only. Today his stools are occurring more frequently and have a more liquid consistency. His
temperature is 102.2  F and he has vomited twice in the past four hours. Which assessment
finding indicates that the child is becoming dehydrated?

a. Occult blood in the stool


b. Abdominal distention
c. Elevated urine specific gravity
d. Hyperactive bowel sounds

Correct C
57- One hour after delivery the nurse is unable to palpate the uterine fundus of a client and
notes a large amount of lochia on the perineal pad. Which intervention should the nurse
implement first?

a. Empty the bladder using an indwelling urinary catheter


b. Increase the rate off the IV containing oxytocin (Pitocin)
c. Assess for shock by determining the blood pressure
d. Perform gentle massage at the level of the umbilicus

Correct D

58- A client a small bowel obstruction is experiencing frequent vomiting. Which instructions are
most important for the nurse to provide to the unlicensed assistive personnel (UAP) who is
completing morning care for this client?

a. Maintain a quiet environment


b. Ensure the linens are clean and dry
c. Place an air deodorizer in the room
d. Measure all emesis accurately

Correct D

59- A child with Leukemia is admitted for chemotherapy, and the nursing diagnosis, “altered
nutrition, less than body requirements related to anorexia, nausea, vomiting” is identified. Which
intervention should the nurse include in this child plan of care?

a. Allow the child to eat foods desired and tolerated


b. Restrict foods brought from fast food restaurants
c. Recommend eating the same foods as siblings eat at home
d. Encourage a variety of large portions of food at every meal

Correct A

60- The nurse is teaching a class on child care to new parents. Which instruction should be
included about the prevention of rotavirus infection in infants who are starting to eat foods?

a. Keep house pets away from the food preparation area


b. Avoid feeding infants fresh fruits
c. Use only lactose-free formulas
d. Wash hands before any food preparation
Correct D

61- The nurse believes that a client who frequently requests pain may have a substance abuse
problem. Which intervention reflects the nurse’s value of client autonomy over veracity?

a. Administer the prescribed analgesic when requested


b. Enroll the client a substance abuse program
c. Collaborate with the healthcare provider to provide a placebo
d. Document the frequency of medication requests

Correct A

62- An 18-year-old gravida 1, at 41-weeks gestation, is undergoing an oxytocin (Pitocin)


induction and has an epidural catheter in place for pain control. With each of the last three
contractions, the nurse notes a late deceleration. The client is repositioned and oxygen
provided, but the late decelerations continue to occur with each contraction. What action the
nurse takes first?

a. Prepare for immediate cesarean birth


b. Turn off the oxytocin (Pitocin) infusion
c. Notify the anesthesiologist that the epidural infusion needs to be disconnected
d. Apply an internal fetal monitoring device and continue to monitor carefully

Correct B

63- In preparing to administer a scheduled dose of intravenous furosemide (Lasix) to a client


with heart failure, the nurse notes that the client’s B-Type Naturetic peptide (BNP) is elevated.
What action should the nurse take?

a. Measure the client’s oxygen saturation before taking further action


b. Administer a PRN does of nitroglycerin (Nitrostat)
c. Administer the dose of furosemide as scheduled
d. Hold the dose of furosemide until contacting the healthcare provider

Correct C

64- When obtaining a urine specimen from a female infant, which intervention should the nurse
implement?

a. Place the wet diaper in a biohazard specimen bag


b. Obtain the urine sample using a straight sixe 4 French catheter
c. Collect the urinary stream in mid-air when the infant cries
d. Secure the pediatric urine collector bag to the perineum

Correct D

65- A client is leaving the hospital against medical advice (AMA) and voluntarily signs the AMA
form. Which nursing action is essential prior to the client leaving?

a) Remove the client’s peripheral IV access


b) Administer requested medication to pain relief
c) Obtain the client’s neurological vital signs
d) Provide the client with the hospital’s phone number

Correct A

66- A client with an electrical burn on the forearm asks the nurse why there is no feeling of pain
from the burn. During the dressing change, the nurse determines that the burn is dry, waxy, and
white. What information should the nurse provide this client?

a. The depth of tissue destructions is minor


b. Pain is interrupted due to nerve compression
c. The full thickness burn has destroyed the nerves….wowww
d. Second degree burns are not usually painful

Correct C

67- The nurse observes a client in a wheelchair with a vest restraint in place. What nursing
intervention is most important for the nurse to implement?

a. Assess the need for continued restraint


b. Check the client for urinary incontinence
c. Determine skin integrity under the vest
d. Perform range-of-motion exercises on extremities

Correct A
68- A 14-year-old girl with asthma complains of feeling nervous and jittery after a respiratory
therapy bronchodilator treatment. What explanation is best for the nurse to provide to this
adolescent?

a. Nervousness should disappear when hypoxia is relieved after several bronchodilator


treatments
b. Tremors result from the rapid dilation of the bronchioles and an increased heart rate
c. A fast heart rate and jitteriness are side effects of the bronchodilator treatment
containing albuterol
d. Excessive coughing, which causes tachypnea and anxiety, result from the use of
bronchodilators

Correct C

69- A client with type 2 diabetes mellitus is admitted for antibiotic treatment of a leg ulcer. Which
signs and symptoms, indicative of hyperosmolar hyperglycemic nonketotic syndrome (HHNS),
should the nurse report to the healthcare provider? (Select all that apply.)

a. Increased heart rate


b. Visual disturbances
c. Presence if uremic frost
d. Decreased mentation
e. Blood glucose 400 mg/dl

Correct A,B,D

70- The husband and adult children of a woman who abuses alcohol ask the nurse what
approach to use when her drinking behavior disrupts family plans. Which response is best for
the nurse to provide?

a. “Destroy the hidden supplies of alcohol she has at home so she has to stay sober”
b. “When she drinks, tell her how disruptive her behaviors are and the burden they inflict on
the family”
c. “Make her responsible for the consequences of her drinking behaviors”
d. “Include her as a part of family activities whether she is drinking or sober”

Correct C
71- A client whose finger stick glucose is 210 mg/dl is receiving a sliding scale dose of short-
acting insulin before breakfast. In what sequence should the nurse prepare the dose of insulin?
(Arrange from first on top to last on the bottom)determiner,cleanse,use,verify)

72- The nurse is preparing to administer an IM injection to a 6 month-old child. Which injection
site is best for the nurse to use?

a) Vastus lateralis
b) Deltoid
c) Ventrogluteal
d) Dorsogluteal

Correct A

73- The client diagnosed with a right fractured femur has skeletal traction applied to the right
femur. Which interventions should the nurse implement?

a. Perform passive range of motion to the right leg


b. Remove skeletal weights every shift to assess right leg
c. Turn frequently from prone to supine positions
d. Maintain skeletal pin sites and assess for signs of infection

Correct D

74- A male client is admitted to the mental health unit because he experiences panic attacks
when driving on the freeway. To attempt to desensitize this fear, what action should the nurse
encourage the client to implement?

a. Watch training videos of people driving in various environments


b. Begin visualizing himself driving each route to the freeway
c. Take antianxiety medication two hours before driving on freeways
d. Get in the car with a support person and drive on a freeway during rush hour.

Correct B
75- A client diagnosed with a deep vein thrombus (DVT) followed by a diagnosis of pulmonary
embolism (PE) is receiving heparin via an infusion pump at a rate of 1400 u/hour. The client tells
the nurse, “I wish this medicine would hurry up and dissolve this clot in my ling so that I can go
home”. What response is best for the nurse to provide?

a. “Heparin prevents future clot formation, but your risk of bleeding needs to be monitored
closely”
b. “You seem to be concerned about the length of time it takes for Heparin to dissolve this
clot”
c. “Let me contact your surgeon and find out Heparin IV therapy can be given to you at
home”
d. “Why are you so anxious to leave the hospital when you know you are not well enough
yet?”

Correct A

76- An angry client screams at the emergency department triage nurse, “I’ve been waiting here
for two hour! You and the staff are incompetent” What is the best response for the nurse to
make?

a. “The emergency department is very busy at this time”


b. “I’ll let you see the doctor next because you’ve waited so long”
c. “I’m doing the best I can for the sickest clients first”
d. “I understand you are frustrated with the wait time”

Correct D

77- The UAP asks if it is necessary to continue to strain the urine of a client with kidney stones
since several stones were obtained the previous day. What information should the nurse
provide the UAP?

a. UAPs do not have authority to question prescribed care


b. It is important to continue straining all the client’s urine
c. Measuring intake and output is more important than straining the urine
d. Ensuring that the client is free from pain is a higher priority care

Correct B

78- Which assessment finding is most indicative of deep vein thrombosis (DVT) in a client’s right
leg?
a. Dorsiflexes the right foot and left on command
b. A 3 by 5cm ecchymosis area covering the right calf
c. Right calf is 3 cm larger in circumference than the left
d. Bilateral lower extremity has 3+ pitting edema

Correct C

79-The nurse is caring for a laboring 22 year-old primigravida following administration of


regional anesthesia. In planning care for this client, what nursing intervention has the highest
priority?

a. Raising the side rails and placing the call bell within reach
b. Teaching the client how to push to decrease the length of the second stage of labor
c. Timing and recording uterine contractions
d. Positioning the client for proper distribution of anesthesia

Correct A

80- The nurse is caring for a client with jaundice. Which serum laboratory value is likely to be
elevated for this client?amarillo,ictericia

a. Amylase
b. Creatinine
c. Blood urea nitrogen
d. Bilirubin

Correct D

81- When evaluating the preoperative teaching of a client scheduled for arthroscopic anterior
cruciate ligament repair, which statement by the client indicates that the teaching was effective?

a. “I will use crutches to keep my weight off my knee”


b. “ I will stay home until a wheelchair is delivered”
c. “ I can use the trapeze bar and side rails on the bed to help me turn regularly”
d. “ I can put my full weight on my foot starting the day after surgery”

Correct A
82- When lactulose (Cephulac) 30 ml QID is prescribed for a male client with advances
cirrhosis, he complains that it causes diarrhea. What action the nurse takes in response to the
client’s statement?

a. Explain that diarrhea is expected, but the drug reduces ammonia levels
b. Document that the client is non-compliant with his treatment plan
c. Tell the client to be concerned about more significant side effects of this drug
d. Obtain a prescription for loperamide (Imodium) 4mg PO PRN diarrhea

Correct A

83- The nurse is obtaining the medical histories of new clients at a community-based primary
care clinic. Which individual has the highest risk for experiencing elder abuse?

a. A 69-year- old widowed female who lives alone and volunteers at a school
b. A 95-year-old ambulatory male who resides in nursing home in a small town
c. A 78-year-old female on a fixed income who lives with her relatives
d. An 81-year-old male with diabetes who lives with his wife of 52 years

Correct C

84- A 70-year-old client is admitted to the hospital after 24 hours of acute diarrhea. To
determine fluid status, which initial data is most important for the nurse to obtain?

a. Usual and current weight…..I&O


b. Color and amount of urine
c. Number and frequency of stools
d. Intake and output 24 hours prior to admission

Correct A

85- A client with a BMI of 60.2 kg/m is admitted to the intensive care unit 3 weeks after gastric
bypass with gastric rupture and impending multiple organ dysfunction syndrome (MODS). What
should the nurse prepare to implement first?

a. Mechanical ventilation
b. Platelet transfusion
c. Loop diuretic therapy
d. Cyanocobalamin administration

Correct A
86- Before administering an intramuscular injection, the nurse’s finger is stuck with the needle.
Which action should the nurse take?

a. Go to the emergency room to have blood drawn


b. Prepare the medication using a new syringe
c. Apply clean gloves before giving the medication
d. Review the medical history in the client’s chart

Correct B

87- The nurse notices that a client with diabetes mellitus type 1 has a fruity breath odor. What is
the priority nursing action?

a. Evaluate the client’s intake and output


b. Measure the client’s capillary blood glucose
c. Consult with the dietician about the client’s diet
d. Apply a pulse oximeter to the client’s finger

Correct B

88- After implementing a new fall prevention protocol on the nursing unit, which action by the
nurse-manager best evaluates the protocol’s effectiveness?

a. Compare the number of falls that occurred before and after protocol implementation
b. Analyze data that reflects the unit’s costs incurred when implementing the new protocol
c. Conduct a chart review on the unit to determine the number of clients at risk for falling
d. Consult with the physical therapist to evaluate the benefits of the new fall protocol

Correct A

89- A client receives a prescription for acetylcysteine (Mucomyst) 1.4 grams per nasogastric
tube q4 hours. Acetylcysteine is available a 10% solution (10 grams/100ml). How many ml of
the 10% solution should the nurse administer per dose? (Enter the numerical value only.)

Ans: 14
90- A client who is diagnosed with amyotrophic lateral sclerosis (ALS) is having difficulty
swallowing and articulating words. Which intervention is most important to include in this client’s
plan of care?

a. Encourage to speak slowly and articulate words


b. Sit upright and flex chin forward while swallowing(quija)
c. Position a communication board at the bedside
d. Provide feeding utensils with large grip handles

Correct B

91- When preparing the client for a thoracentesis, it is essential for the nurse to take which
action?

a. Encourage the client to cough during the procedure


b. Ask the client to void prior to the procedure
c. Have the client lie in the prone position
d. Determine if chest x-rays have been completed…..LUNGS?

Correct D

92- In deciding whether to join a nursing strike called after collective bargaining efforts have
failed, which factors is most important for the nurse to consider?

a. Nurse Practice Act of the State


b. The role of the nurses as client advocate
c. Standards of clinical nursing practice
d. Personal value system

Correct D

93- The nurse is preparing to teach the parents of a child who had surgical repair of a
myelomeningcole how to change an occlusive dressing on the child’s back. Which statement by
the parents indicates that they understand this procedure?

a. When changing the dressing, the tape should be removed rapidly from the edges.
b. To ensure easy removal of the suture, the dressing should be keep dry
c. The skin incision should be kept moist by periodically wetting the dressing
d. The incision should be protected from fecal contamination by an intact dressing

Correct D

94- While eating at a restaurant, a gravid woman begins to choke and is unable to speak. What
action should the nurse who witness the event take?

a. Cardiopulmonary resuscitation with uterine tilt


b. The Heimlich maneuver using chest thrusts
c. The Heimlich maneuver using subdiaphragmatic thirsts
d. Call 911 immediately than begin cardiopulmonary resuscitation

Correct B

95- A client with endometrial carcinoma is receiving brachytherapy and has radioactive Cesium
loaded in a vaginal applicator. What action should the nurse implement?

a. Wear a dosimeter film badge when in the client’s room


b. Spend 30 minutes at the bedside when clustering direct care
c. Change the linens every day after providing a bed bath
d. Use gloves to remove the applicator if it is dislodged in the bed

Correct A

96- A client has a history of vasovagal attacks resulting in brady-dysrythmias. Which instruction
is most important to include in the teaching plan?

a. Use stool softeners regularly…..Valsalva-vagal Brady dysrhythmia?!


b. Avoid electromagnetic fields
c. Maintain a low fat diet
d. Do not use aspirin products

Correct A
97- Which behavior is most likely to result in a breach of client confidentiality?

a. Discussing a client’s condition during a teaching conference for nursing staff caring for
the client
b. Two nurses planning a client’s care while having lunch in the hospital cafeteria
c. Nursing students on the same team discussing their assigned client’s conditions
d. A registered nurse privately sharing personal feelings about a client with another nurse
on the team.

Correct B

98- A female client tells the clinic nurse that she has doubts of binge eating but cannot make
herself vomit after meals. Which action by the nurse provides data to support the suspected
diagnosis of bulimia?

a. Ask the client to complete a food diary for the last 3 days
b. Review the client’s lab data to determine her TSH, T2 and T4 levels.
c. Interview the client about her use of laxatives and diuretics
d. Encourage the client to describe her daily exercise regimen

Correct C

99- The nurse is assessing an infant with pyloric stenosis. Which pathophysiological mechanism
is the most likely consequence of this infant’s clinical picture?

a. Metabolic acidosis
b. Metabolic alkalosis….vomiting
c. Respiratory alkalosis
d. Respiratory acidosis

Correct B

100- A high school football player comes to the clinic complaining of severe acne. The
mother reports recent behavior changes, including irritability and suspiciousness of friends. The
nurse’s assessment reveals an elevated blood pressure. Which intervention should the nurse
implement first?

a. Encourage the client to see a dermatologist


b. Refer the adolescent to a substance abuse program
c. Suggest a low-salt, low fat, and caffeine free diet
d. Inquire about a possible use of anabolic steroids

Correct D

101- Which assessment is most important for the nurse to perform before ambulating a client
with a history of syncope?(perdida de la conciensia,y cambios en la bp al cambiar la pocision)

a) Pedal pulses

b) Breath sounds

c) Oxygen saturation

d) Blood pressure…..hypotension, fall, dizzy, faint

Correct D

102- A male client with schizophrenia is jerking his neck and smacking his lips. Which finding
indicates to the nurse that he is experiencing an irreversible side effect of antipsychotic
agents?

a) Cramping muscular pain


b) Worming movements of tongue
c) Decreased tendon reflexs
d) Dry oral mucous membranes

Correct B

103- What assessment data should lead the nurse to suspect that a client has progressed from
HIV infection to AIDS?

a) Cervical lymph nodes are enlarged and tender


b) Presence of low grade fever and sore throat
c) Recent history of recurrent pneumonia
d) CD4 blood cell count of 300

Correct C
104. A community health nurse is preparing to apply for a federal grant for an educational
program about smoking cessation. In which order should the nurse prioritize the nursing
activities? Arrange the activities in the order from first on top to last on bottom

5Develop a plan of action to educate the target population

1Determine the target aggregate that will be served by the plan

3Select and train the data collection personnel

2Consider the pros and cons of different methods of data

4Review and analyze the data

105. A client is admitted with acute low back pain. What action should the nurse implement to
promote comfort?

a. Ambulate using a walker for upper body support


b. strach the back ,uscles using abdominal curls
c. Position in semi-Fowler's with the knees flexed
d. Encourage straight leg raises while lying supine.

Correct C

106. The nurse is performing a surgical hand scrub prior to entering the operating room. In what
order should the nurse perform the steps of this procedure? Top to last….R-SUC

Rinse from the fingertips to the elbow

Scrape under the nails with a nail pick

Use a soapy brush to scrub the hands

Cleanse the arm with a lathered brush

107. A male client who fell into the lake while fishing and was submerged for about 3 min was
successfully resuscitated by his friends. He was brought to the Emergency Departament for
evaluation and was admitted for a 24hr uneventful hospital stay.

a) Avoid smoke filled environments


b) Seek medical care promptly if a fever develops
c) Increase oral fluids if a cough becomes productive
d) Schedule frequent rest periods
Correct B

108. Which action should the nurse implement when assessing a client's blood pressure and
determining that there is an auscultatory gap?

a. Compare the palpated systolic blood pressure with the auscultated pressure
b. Assess the client for lightheadedness which routinely occurs upon standing
c. Determine if an automated blood pressure reading is consistent with the manual
d. Observe for a change in blood pressure when comparing lying and sitting
positions
Correct A

109. Identify the placement of the stapes footplate into the bony labyrinth. (click the chosen
location) la imagen esta en el medio donde hay que tocar

110. A 60 yr old male with type 2 diabetes mellitus tells the nurse that he is going to join a gym
and start working out. Which information is most important for the nurse to obtain?

a. Recent serum cholesterol blood levels


b. Presence of calf pain during exercise
c. Average weight gain or loss in the last year
d. Exercise tolerance test with EKG results

Correct D

111. A client is receiving an IV infusion of regular 75 unit in 100 ml of normal saline at 9 unit/hr.
The nurse shoul program the infusion pump to deliver how many ml/hr?

ANS 7

Use the formula desired dose/ available dose

9units/hour / 75 units x 100 ml = 12

112. A 14 yr old male client with a spinal cord injury (SCI) at T-10 is admitted for rehabilitation.
During the morning assessment, the nurse determines that the adolescent's face is flushed, his
forehead is sweating, his heart rate is 54 beats/min, and his blood pressure is 198/118. What
action should the nurse implement first?
a. Determine if the urinary bladder is distended
b. Irrigate the indwelling urinary catheter
c. Review the temperature graph for the last day
d. Administer an antihypertensive agent

Correct A

113. A new mother asks the nurse if the newborn infant has an infection because the healthcare
provider prescibed a blood test called the TORCH screen test. Which response should the
nurse offter to his mother's inquiry?

a. Rising liters indentify the etiology of certain neuro-sensory birth defects


b. The screen determines the risk for inherited anomalies in the newborn
c. The test indetifies the correct antibiotic to give the newborn for an infection
d. Exposure to infections that can cross the placenta cause a positive antibody liter

Correct D

114. Which techniques should the nurse use to administer an intradermal (ID) injection for a
Mantoux test to screen for tuberculosis (TB). Select all that apply

a. Observe for an interdermal bled after the antigen is injected


b. Select an ID site using the scapular region of the upper back
c. Use a 25 gauge 1/2 inch needle on a 1 ml calibrated syringe
d. Ensure that the needle is inserted into the skin with the level up
e. Insert the niddle at a 45 degree angle to the skin surface
f. Massage the are after the injection to ensure absorption

Correct D,C,A

115. Following rectal surgery, a female client seems very anxious about the pain that she may
experience during defecation. The nurse should collaborate with the healthcare provider to
administer which type of medication?

a. Bulk-forming agent
b. Antianxiety agent
c. Stool softener
d. Stimulant cathartic

Correct C
116. The mother of a teenager is told that her son has recently been found stealing from other
students at school. The mother'[s response is. " I cannot think about that today". The nurse
determines that this mother is using which defense mechanism?

a. Suppression
b. Repression
c. Sublimation
d. Undoing

Correct A

117. Which nursing entry to the client record best reflects significant data on a male client who
is admitted with complaints of chest pain?

a. Nurse will check client q1h for the presence of chest pain
b. Client has a nervous, tense personally and likely to overreact
c. Client states he will notify the nurse if chest returns
d. Client understands how to use the call button and the telephone

Correct C

118. A 37 yr old client diagnosed with chronic kidney disease (CKD) is being treated for renal
osteodystrophy. Which nursing diagnosis is most likely to be included in this client's plan of
care?

a. High risk for infection related to subclavian catheter


b. Hight risk injury related to ambulation
c. Knowledge deficit related to high-protein diet
d. Hygiene self-care deficit related to urenic frost.

Correct B

119. The nurse is performing an admission assessment on an HIV positive client with a
diagnosis of Pneumocystis carinii pheumonia (PCP). the nurse should carefully observe the
client for which symptoms?

a. Weight loss exceeding 10 percent of baseline body weight


b. Altered mental status and tachypnea
c. Creamy white patches in the oral cavity
d. Normal ABGs with wet lung sounds in all lung fields
Correct B

120. A 10 month old girl is admitted with a diagnosis of possible cystic fibrosis. What question
should the nurse ask the patient about their child to assist in the diagnosis of cystic fibrosis
(CF)?

a. When you kiss her. does she taste salty?


b. Do you notice if her urine has a musty odor?
c. Has she been drinkin cow's milk in her daily diet?
d. How ofter does she have a normal bowe mov?

Correct A

121. Parents who have one male child with sckle cell anemia are concerned about having more
children with the disease. What client teaching should the nurse provide?

a. All future children will be carries, but will not have the disease
b. There is a chanse that each future child will have the disease
c. Only male children can inherit the sickle cell disease trait
d. Only one out of four of their children will manifest the disease

Correct B

122. A client with rheumatoid arthritis reports a new onset of increasing fatigue. What
intervention should the nurse implement first?

a. Obtain a wheelchair for the client


b. Explain to the client that this is an expected symptom
c. Assess the client for pallor
d. Obtain a sedative-hynotic prescription

Correct C

123. Which laboratory finding should the nurse expect to see in a child with acute rheumatic
fever?

a. Thrombocytopenia
b. Polycythemia
c. Decreased ESR
d. Positive ASO liter

Correct D

124. A client who is sheduled to have surgery in two hr tells the nurse. " My doctor was here
and used a lot of big words about the surgery, then asked me to sign a paer." What action
should the nurse take?

a. Reassure the client that presurgery anxiety is a normal experience


b. Explain the surgery in clear terms that the client can understand
c. Call the surgen back to clarify the information with the client
d. Redirect the client;s thoughts by teaching relaxation techniques

Correct C

125. When the nurse is designing a plan of care for a client diagnosed with pheochromocytoma,
a goal statement should be prepared that relates to which topic?

a. Psychological counseling to address body image changes


b. Medication teaching concerning thyroid hormones
c. Preoperative and postoperative teaching for adrenalectomy
d. Cliwnt and family referral to a hospice

Correct C

126. A client is being treated for minor injuries following an automobile accident in which the
only other passenger was killed. The client ask the nurse. " Is my friend who was in the car with
me ok"? What response is best for the nurse to provide?

a. "I am sorry, but you friend was killed in the accident"


b. "Right now you need to concentrate on getting well"
c. "was the passenger in the car your friend?"
d. "I think you friend is going to be all right"

Correct A

127. To differentiate adventitious lung sounds associated with heart failure from those
associated with bacterial pneumonia. What information should the nurse review?
a. Sputum culture findings
b. Oxygen saturation level
c. Amount of coughing
d. Respiratory rate

Correct A

128. A two yr old boy begins to cry when his mother starts to leave. What is the nurse's best
response in this situation?

a. "Let's wave bye=bye to Mommy"


b. "Let me read this book to you"
c. "Two yr old usually stop crying the min the parent leaves"
d. "Now, now, be a big boy Mommy will be back soon"

Correct B

129. Apremature infant weighing 1.200 gr at birth receives a prescription for beractant
(Survanta) 120 mg endotracheal now and q6 hr for 24 hr. The recommended dose for beractant
is 100 mg/kg birth weight per dose. Single use vials of Survanta are labeled, 100 mg/4 ml. What
action should the nurse take?

a. Give 4.8 ml q6 hr
b. Notify the healthcare provider that the dose is too hight
c. Notify the healthcare provider that the dose is too low
d. Give 1.2 ml q6 hr

Correct A

130. Several clients on a busy antepartum unit are scheduled for procedures that require that
informed consent. Which situation should the nurse explore futher before witnessing the client's
signature on the consent form?

a. The client was medicated for pain with a narcotic analgesic IM 6hr ago
b. A 15 yr old primagravida who has been self-supporting for the pas 6 month
c. The obstetrician explained a procedure that a neurologist will perform
d. The client is illiterate but verbalize understanding and consent for the procedure

Correct C
131. An 8 yrs old child who weighs 60 pounds receives an order for polycillin (Ampicillin)
suspension 25 mg/kg/day divided in a dose every 8hr. The medication is labeled "125mg/5ml".
How many ml should the nurse administer?

ANS 9ml

132. A 3 yr old comes to the clinic for a well-child check up. Which respiratory assessment
finding should the nurse expect this child to exhibit?

a. A resting respiratory rate of 40 breaths per min


b. Brochvesicular breath sounds in the peripheral lung fields
c. Retractions in the intercostal spaces with each inspiration
d. hight-pitched whistling sounds over the brochus

Correct B

133. The charge nurse is making assignment for clients on an endocrine unit. Which client is
best to assign to a new graduate nurse?

a. A female adult with hyperthyroidism who is returning to the unit after a


thyroidectomy
b. A male adult with Cushing's syndrome who reports a headache and visual
disturbances
c. An older man with Addison's disease who is diaphoretic and has hand tremors
d. A Perimenopausal woman with GRaves' disease whi is nervous and has
exophthalmos

Correct D

134. A client is admitted with a medical diagnosis of acute pancreatitis. When taking a health
history, which client complaint should be expected?

a. A low-grade fever and left lower abdominal pain.


b. Severe headache and sweating all the time
c. Severe mid-epigastric pain after ingesting a heavy meal
d. Dull, continuos, right lower queadrant pain and nausea

Correct C
135. The nurse is planning care for a family whose children did not receive childhood
immunizations. After one of the children contracted mumps, the father is diagnosed with orchitis.
Which intervention should be included in the father's plan of care? (inflacion de los testiculos)

a. Use of bedrest with scrotal support


b. Administration of antibiotics for 10 days
c. Applying heat to promote the healing process
d. Using an ice pack to reduce scrotal pain

Correct A

136. A newborn whose mother is HIV positive is admitted to the nursery from labor and delivery.
Which action should the nurse implement first?

a. Initiate treatment with zidovudine (ZDV) syrup at 2 mg per kg


b. Bathe the infant with dilute chlorhexidine (Hibiclens) or soap…..big disease
with big name?
c. Measure and record the infant's frontal-occipital circumference
d. Administer vitamin K (AquaMEPHYTON) IM in the vastus lateralis

Correct B

a) 137. A grant is awarded to provide primary preventative health care to a community


based healthcare system. When designing the program to meet the grant objective,
which service should the nurse consider for inclusion in the program? Select all that
apply

a. Breast screening for older women


b. Rehabilitation services for stroke victims
c. Blood pressure assessments
d. Antepartal nutritional counseling
e. Ambulatory oncology treatments
f. Emergency services for traume victims

Correct A,C,D (BAB)

138. An adult male who admits to abusing IV drugs obtains the results of HIV testing. When
informed that the results are positive, he states that he does not want his wife to know. What
action should the nurse take?

b) Tell the client he is required by law to inform his sexual partners of his HIV status
c) Counsel the client about the importance of notifying his sexual partner
d) Inform the wife of her health risk related to her husband's HIV results
e) Report the client's as a sexually transmitted case to the health department

Correct B

139. Two hr after delivering a 9 pound infant, a client saturates a perineal pad every 15 min.
Although an IV containing Pitocin is infusing, her uterus remains boggy, even with massage.
The healthcare provider prescribes methylergonovine maleate (Methergine) 0.2 mg IM STAT.
Which complication should the nurse be alert to this client developing?

a. Decreased respiratory rate


b. Increased temperature
c. Tachycardia
d. Hypertension

Correct D

140. When assessing a client several hr after surgery, the nurse observes that the client
grimaces and guards the incision while moving in the bed. The client is diaphoretic, has a radial
pulse rate of 110 beats/min, and a respiratory rate of 35 breath/min. What assessment should
the nurse perform first?

a. Apical heart rate


b. IV site and fluids
c. Pain scale
d. Temperature

Correct C

141. A man calls the hospital and ask to talk with the nurse about his girlfriend who was
extremely intoxicated on admission and is receiving services for detoxification. He knows that
she is in the facility and ask the nurse about her condition. What is the nurse's best response?

a. : I can only report that the client is in satisfactory condition"


b. "Let me give you the telephone number for her room:
c. : I cannot acknowledge if a client is here or not"
d. " I will have the nurse who is working with her call you"

Correct C
142. The medical record of a child with Duchenne muscular dystrophy (MD) states a Gower sign
is present. Based on this documentation, the nurse should expect the child to exhibit which
behavior?

a. Stands from sitting on the floor by using hands to walk up legs


b. Exhibits muscular atrophy of upper and lower extremities
c. Is unable to stand because of contractures of both hips
d. Walks with an unsteady gait and slaps feet on floor

Correct A

143. Which situation is a violation of the client confidentiality, as described in the Health
Insurance Portability Accountability Act (HIPAA)?

a. Assign in sheet kept at the front desk listing client's last names and time of their
arrival
b. A nurse's handwritten notes from a telephone report discarded in the office
wastebasket
c. A computer monitor screen that is located at the nurse's station in a high traffic
area
d. Privileged Heath Information (PHI) given to an ambulance friver for transfer of a
client

Correct B

144. The nurse observes that a client who is to avoid any weight bearing on the left leg is using
a 3 point crutch gait for ambulation. What is the best action for the nurse to initiate?

a. Encourage continued use of the 3 point gait by the client


b. Encourage the client to use a wheelchair for mobility
c. Instruct the client in the use of a 4 pint crutch gait
d. Instruct the client in the use of a 2 point crutch gait

Correct A

145. The nurse is caring for a comatose client. Which assessment finding provides the greatest
indication that the client has an open airway?

a. The client has asymmetrical chest expansion


b. Percussion reveals dullness over the lung area
c. Bilateral breath sounds can be auscultated……airway and breathing
d. The client has been turned q2h

Correct C

146. The healthcare provider prescribes lidocaine (Lidoject-1) 100 mg IV push for ventricular
tachycardia (IV) for an unconscious client. What is the nurse's priority intervention?

a. Measure the client's cardiac output


b. Assess neurological status q15 min
c. Collect a blood specimen for a serum potassium
d. Infuse lidocaine (Lidoject-1) at 20 to 50 mg/min

Correct D

147. A client who sustained a pellet gun injury with a resulting comminuted skull fracture is
admitted overnight for observation. Which assessment linding obtained two hours after
admission necessitates immediate intervention?

a. The client complains of a throbbing headache rated 10 (1 to 10 scale)


b. The client repeatedly falls asleep while talking with the nurse
c. The entry site has a slow trickle of bright red blood
d. The entry site appears reddened and edematous

Correct B

148. A male client, admitted to the mental health unit for a somatoform disorder, becomes angry
because he cannot have his pain medication. He demands that the nurse call the healthcare
provider and threatens to leave the hospital. What action should the nurse take?

a. Place the client in seclusion per unit guidelines


b. Administer a PRN prescription for lorazepam (Ativan)
c. Call security to help ensure staff and client safety
d. Ask what other methods he uses to deal with pain

Correct D

149. While flushing the proximal port of a triple lumen central venous catheter with heparin
solution, The nurse meets resistance. What action should the nurse take?
a. Remove the cap and apply direct gentle pressure with the syringe
b. Contract the healthcare provider regarding the need for a chest x-ray
c. cover the cap with tape and label the port as being obstructed
d. Remove the catheter while applying gentle pressure at the insertion site

Correct C

150. A client is admitted to the hospital with a serum sodium level of 128 mEq/L, distended
neck veins, and lung crackles. What intervention should the nurse implement?

a. Increase the intake of salty foods


b. Administer NaCl supplemets
c. Restrict oral fluid intake
d. Hold the client's loop diuretic

Correct C

151. A female client presents to the emergency department in the early evening complaining of
abdominal cramping, watery diarrhea, and vomiting. She tells the nurse that she was at a picnic
and ate barbeque that afternoon. What question is most important for the triage nurse to ask
this client?

a. "Have you recently traveled outside the United States"?


b. "How high was your temperature when you returned home?"
c. "have you taken any medication to treat this problem?
d. " Is anyone else sick who was at the picnic?"

Correct D

152.A client with active tuverculosis (TB) is receiving isoniazid (INH) and rifampin (RMP) daily,
so direct observation therapy (DOT) is initiated while the client is hospitalized. Which instruction
should the nurse provide this client?

a. Describe feelings about taking daily medications


b. Take medications in the presence of the nurse
c. Notify the burse after self-medication is completed
d. keep a daily record of all medications taken

Correct B
153. The nurse should explain to a client with lung cancer that pleurodesis is performed to
achieve which expected outcome?

a. Debulk tumor to maintain patency of air passages


b. Relieve empyema after pneumonectomy
c. Prevent the formation of effusion fluid…..pleural effusion?
d. Remove fluid from the intrapleural space

Correct C

154. The nurse knows the client a mechanical valve replacement understands the discharge
teaching when the client makes which statement?

a. "I will need to take antibiotics before any type of invasive dental work"
b. " I will not have to take any more heart medication since i have a new valve"
c. " I will need to have this valve replaced in about 10 years
d. I should notify my healthcare provider if i hear a clicking sound near my heart"

Correct A

155. Following a thyroidectomy, a client experience tetany. The nurse should expect administer
what intravenous medication?

a. Sodium iodide solution


b. Levothyroxine sodium (synthroid)
c. calcium gluconate…..T for twitching and Tetany
d. Propranotol (INderal)

Correct C

156. What nursing intervention is most important to implement after a client has a completed a
myelogram?

a. Lie-sit-stand blood pressure measurement


b. Abdominal assessment for distention and bowel sounds
c. Neurovascular assessment of lower extremities…..nerve/spinal back surgery-rad
d. Assessment of skin temperature and turgor

Correct C
157. Two weeks following a Billroth II (gastrojejunostomy), a client develops nausea, diarrhea
and diaphorosis after every meal. When the nurse develops a teaching plan for this client, which
expected outcome statement is the most relevant? the client

a. describes a schedule for antacid use with other prescribed medications


b. selects a pattern of small meals alternating with fluid intake
c. agrees to participate in a variety of stress reduction techniques
d. expresses a willingness to reduce nicotine intake

Correct B

158. A client has produced the first of a series of sputum sample for cytology. what action
should the nurse implement?

a. Ensure the client remains NPO until all the samples are collected
b. Discard the initial sample and document the time it was obtained
c. Transport the sputum contains of the laboratory in a biohazard bag
d. Document the time the client las ate or drank on the laboratory slip

Correct C

159. An elderly client at an adult daycare with Type 2 diabetes becomes unresponsive verbally
with the other daycare paticipants and tells the nurse, "I just don't feel right." Which initial action
should the nurse take?

a. Evaluate deep tendon reflexes


b. Assess temperature
c. Administer glucagon 0.5 mg IM
d. Give 4 onces of apple juice

Correct D

160. The triage nurse in an emergency center must prioritize the admission of four clients from
the waiting area to a treatment room. Which client should the nurse identify as the first to
receive care?

a. An afebrile woman at 28 weeks gestation who is complaining of sinus pain and


has had a green nasal discharge for one day.
b. An 8 yr ols male with a known seizure disorder whose mother witnessed him in a
partial seizure about 50 min ao
c. A 6 month old male who is feverish has clear mucous fraining from his nose and
has been pulling on his ears for 2 days
d. A female teenager who states she took 8 of her mother's amitriotyline
hydrochloride (Elavil) pills an hr ago.

Correct D

CAT VERSION 1

1. A client is comatose upon arrival to the emergency department after


falling a roof. The client flexes with painful stimuli, and the nurse
determines the client’s Glasgow Coma Scale (GCS) is 6. Which
intervention should the nurse prepare to implement to maintain the
client’s airway?

A. An endotracheal tube
B. A nasopharyngeal tube…..duh, we breathe air through our nose
C. An oral airway
D. Tracheostomy tube insertion

2. A client is receiving a continuous half strength tube feeding at 50 ml/hr.


To prepare enough of the solution for eight hours, how many ml of full
strength feeding will the nurse need?

Answer: 200

3. The alarm of a client’s pulse oximeter sounds and the nurse notes that the
oxygen saturation rate is indicated at 85%. What action should the nurse
take first?

A. Administer oxygen by face mask


B. Notify the healthcare provider
C. Reset the alarm
D. Check the probe position……miss Tony??? awwhh

4. A client is known to have an irregular respiratory rate with periods of


apnea lasting 10 to 15 seconds. Currently, the nurse counts 22 respiratory
cycles in a 30-second interval followed by an apneic period. What
intervention should the nurse implement?
A. Reassess the respiratory rate, counting for one full minute
B. Call a code and initiate cardiopulmonary resuscitation
C. Immediately place the client in Trendelenburg position
D. Record the respiratory rate and notify the respiratory therapist

5. A retiree with depression complains of feeling “lonely and having no


purpose” in life. Based on Erikson’s developmental theory, which
questions suggest that the nurse understands the client’s most important
emotional need?

A. “Where can you go to be with others?”


B. “What about your life makes you proud?” (orgullo)…..at least let hm
mention something positive he has done/achieve in life
C. “How do you spend your days?”
D. “What time of the day do you feel lonely?”

6. Following a precipitous labor, a client has a continuous trickling of bright


red blood from her vagina. Her uterus is firm and her vital signs are
within normal limits. The nurse determines that the client’s symptoms
may indicate which condition?

A. A cervical laceration (DESGARRO)


B. A normal fourth stage of labor
C. Early postpartum hemorrhage
D. Inadequate uterine contractions

7. In preparing assignments for the shift, which client is best for the charge
nurse to assign to a practical nurse?(LPN)

A. An older client who fell yesterday and is now complaining of diplopia


B. An adult newly diagnosed with type 1 diabetes and high cholesterol
C. A client with pancreatic cancer who is experiencing intractable pain
D. An elderly client with Alzheimer’s disease complicated by dysphagia
8. The healthcare provider prescribes oxygen per nasal cannula at 2 L/min.
Which action has the highest priority when the nurse implements this
prescription?

A. Set the flow meter


B. Administer oral care
C. Pad bony prominences
D. Apply a humidifier

9. A nurse who is new to the pediatric unit is positioning a 6-month-old for


an injection of penicillin V (Pen V) in the dorsogluteal muscle. Which
action should the nurse-manager who is supervising this nurse take first?

A. Review the correct landmarks before the site is injected


B. Explain the correct procedure for giving the medication
C. Instruct the nurse to select another injection site (vastus laterali)
D. Demonstrate techniques for restraining the infant

10. After diagnosis and initial treatment of a 3-year-old child with cystic
fibrosis, the nurse provides home care instructions to the mother. Which
statement by the child’s mother indicates that she understands home care
treatment to promote pulmonary function?

A. “Cough suppressants can be used four times a day”


B. “The oxygen should be kept at 4 to 6 L/min”
C. “Chest physiotherapy should be performed at least twice a day”
D. “Activities should be planned to avoid physical exertion”

11. The nurse is preparing a community education program on osteoporosis.


Which instruction is helpful in preventing bone loss and promoting bone
formation?

A. Encourage intake of foods high in vitamin E


B. Decrease intake of foods high in fat
C. Decrease heavy lifting and bending
D. Encourage physical activity
12.The nurse reviews the signs of hypoglycemia with the parents of a child
with Type 1 diabetes mellitus. The parents correctly understands signs of
hypoglycemia if they include which symptom?

A. Sweating (sudoracion)
B. Increased urination
C. Fruity breath odor
D. Thirst

13. The nurse observes a 2 cm area nonblanchable erythema on the sacrum


of an immobile client. What documentation of this finding is best for the
nurse to enter into the client’s record?

A. Sacral area red and inflamed


B. 2 cm area reactive hyperthermia on sacrum
C. Stage 1 pressure ulcer on sacrum
D. Client at high risk for pressure ulcer

14. When assessing a 7-year-old girl, the nurse notes that she has multiple
bruises on her back and upper arms. The child’s aunt tells the nurse that
the child’s parents abuse drugs and alcohol. What intervention is most
essential for the nurse to implement?(moretones)

A. Notify child’s healthcare provider of the assessment findings


B. Determine the reliability of the aunt’s report
C. Report the child’s condition to the nursing supervisor
D. Report assessment findings to the proper legal authorities

15. A 14-year-old male client arrives at the emergency room in status


epilepticus. He was diagnosed with a seizure disorder in childhood. What
is the most likely cause of his present condition?(son reveldes )

A. Increasing intracranial pressure


B. Acute withdrawal from anticonvulsant medication
C. A closed head injury
D. A central nervous system infection

16. A Chinese-American client who just delivered a baby states that she will
not be able to take the prescribed sitz baths to help heal her episiotomy
incision because this will cause an unhealthy balance of cold and hot
forces. When planning nursing care, what nursing diagnosis has the
highest priority?

A. Knowledge deficit related to healing process


B. Noncompliance related to cultural diversity
C. Anxiety related to cultural diversity
D. Impaired tissue integrity related to episiotomy

17. A 2-year-old with sickle cell anemia has an axillary temperature of 102
F. In planning care for this child, which nursing diagnosis has the highest
priority?

A. High risk infection related to low platelet count


B. High risk for fluid volume deficit related to temperature elevation
C. Alteration in urinary elimination related to renal damage from disease
D. Potential activity intolerance related to anemia

18. During the first trimester of pregnancy, a client who was treated for
genital herpes with acyclovir (Zovirax) prior to this pregnancy tells the
nurse that she is experiencing an episode of genital herpes. Which
nursing intervention has the highest priority?
A. Identify current sexual partners so that they can be evaluated and treated
for genital herpes if necessary
B. Determine if the client has taken acyclovir (Zovirax) for this
outbreak of genital herpes
C. Instruct her to avoid sexual intercourse while active, visible lesions are
present
D. Assess her feelings about therapeutic abortions in the event the infant has
been affected

19. The nurse is obtaining a medication history for a client with a new
prescription for paroxetine (Paxil). The client reports current use of the
MAO inhibitor isocarboxazid (Marplan). What intervention is most
important for the nurse to implement?

A. Instruct the client to use good oral hygiene measures to reduce dry mouth
B. Assess the client for an increased sense of well-being once started on the
Paxil
C. Instruct the client to avoid foods high in tyamine while taking Marplan
D. Notify the healthcare provider that the client is currently taking
Marplan

20. While performing a skin inspection on a newborn, the nurse finds a


small dimple and a dark tuft of hair in the lumbosacral area of the infant’s
back. What is the most likely indication of this finding?

A. External manifestation of a spinal abnormality(yeah, hair on dark)


B. Expected finding in a newborn
C. Uncommon but normal variation in newborns
D. Variation often seen in dark-skinned infants

21. What action should the nurse implement first when delegating nursing
activities to an unlicensed assistive personnel (UAP)?

A. Consider the client’s ability to assist the UAP


B. Evaluate the experience of the UAP
C. Prioritize each assigned client’s needs
D. Determine if family is available to help

22. A female resident of a long-term care facility is being admitted to the


medical department. The client has a fractured hip and has methicillin-
resistant staphylococcus aureus (MRSA). Which room should the charge
nurse assign this client?

A. A semi-private room with a client who also has methicillin-resistant


staphylococcus aureus (MRSA)
B. A private room, and institute blood-borne standard precautions
C. A semi-private room with a client who has enterococci resistant to
vancomycin (VRE)
D. A private isolation room with a vented negative airflow system

23. The nurse is preparing a client for surgery. Which finding indicates that
the client is ready to proceed to the operating room (OR) for a scheduled
surgical procedure?
A. Hemoglobin 10.1 grams (<) (m=14-18/f=12-16)
B. Client questions which surgery is scheduled
C. Clopidogrel (Plavix) received yesterday
D. INR results of 3.1

24. The nurse is preparing to administer an intramuscular injection in the


ventrogluteal site of a client who weighs 80 kg. What size needle should
the nurse select?

A. 25-gauge, 1-inch needle


B. 27-gauge, 5/8-inch needle
C. 21-gauge, 1.5-inch needle
D. 20-gauge, 3-inch needle

25.A client at 38-weeks gestation is in active labor, and a vaginal birth after
Cesarean section (VBAC) is planned. Vaginal exam indicates that the
client is 6 cm dilated, 90% effaced, and at station 0 with intact
membranes. As the client’s contraction become stronger, the fetal heart
rate decelerates during the contractions but returns to baseline. What
action should the nurse take?

A. Prepare for an emergency Cesarean delivery


B. Set up an amniotomy tray
C. Continue to monitor the client’s labor progress (por un riesgo de
rupture uterine devido a la operacion Antigua “cesaria”)
D. Apply a fetal scalp electrode

26. An elderly client with limited mobility reports frequent episodes of


nocturia. To reduce the risk for urinary incontinence, what action should
the nurse implement?(se levanta mucho en la noche a orinar)

A. Maintain a calm and quiet environment


B. Review the client’s serum creatinine level
C. Obtain a prescription for a hypnotic at bedtime
D. Keep the call bell within the client’s reach

27. Which diagnostic tests are most important for the nurse to monitor when
providing care for a client with a bowel obstruction?
A. Serum albumin and protein
B. Serum liver enzymes
C. Serum electrolytes ( en los intestines se absorbe solo liquidos )
D. Gastric pH analysis

28. A 72-year-old male client reports that he has felt depressed since his
wife died six months ago. What question is most important for the nurse
to ask this client?

A. “Have you ever had a loved one die before?”


B. “Do you have close friends in whom you can confide?”
C. “Are you sleeping and eating well?”
D. “Have you ever felt like hurting yourself?” (lastimado)

29. After administering the first dose of newly prescribed to four clients
within a thirty minute time frame, the nurse evaluates each client for
therapeutic responses or any adverse reactions. Which medication should
the nurse evaluate first?

A. Clopidrogel (Plavix)
B. Nystatin (Mycostatin)
C. Enoxaparin (Lovenox)
D. HYdromorphone (Dilaudid) (potente opioid anargesico)dipres CNS)

30. What is the most important symptom the nurse should monitor the client
for while assisting with the insertion of a subclavian central venous
catheter?

A. Edema at the insertion site


B. Paralysis of the face and neck on the side of insertion
C. Pain, accompanied by nausea and vomiting
D. Shortness of breath….breathing always important

31. The nurse is developing a plan of care for a client who has a prescription
for the calcium channel-blocker nifedipine (Procardia) to treat angina
pectoris. What is the purpose for administration of this medication?

A. Increase heart rate and force of contraction


B. Reduce the incidence of clot formation
C. Stimulate the vagus nerve to increase heart rate
D. Decrease myocardial oxygen demands

32. While transcribing a new prescription, the nurse notes that the prescribed
dosage is much lower than the recommended dosage listed in the drug
reference guide. Which client data supports this dosage reduction?

A. Increased serum protein


B. Increased liver enzymes……metabolizes meds
C. Decreased serum creatinine
D. Prolonged prothrombin time

33. The nurse notes that a postoperative adult client’s respiratory rate is 10
breaths/minute. Which factor in the client’s history is the most likely
explanation for this finding?

A. Postoperative laboratory test results indicate that the client’s hemoglobin


is 10.1 gm/dl and hematocrit is 30.4%
B. The client has a ten-year history of chronic obstructive pulmonary
disease (COPD)
C. The PCA pump containing morphine sulfate was discontinued 15
minutes before vital signs were taken……OPIOID RESP.
D. The client smoked one pack of cigarettes/day for the past 20 years, before
quitting smoking 30 days ago

34. The charge nurse working on a postpartum unit is making assignments


for a staff consisting of a registered nurse (RN), a practical nurse (PN),
and two unlicensed assistive personnel (UAP). Which client should the
charge nurse assign to the registered nurse?

A. A primigravida who delivered an infant 6 hours ago via vaginal


delivery and is now complaining of seeing spots (VE MANCHAS)
B. A multigravida who is breastfeeding her infant and is preparing for
discharge with her infant(NO)
C. A client who delivered a 10 pound infant 8 hours ago via cesarean
section and is now complaining of pain(NO)
D. A client who had an epidural for a vaginal delivery one hour ago and now
needs assistance to the bathroom(ESTA DE PACTO)
35. A 13-year-old client with non-union of a comminuted fracture of the
tibia is admitted with osteomyelitis. The healthcare provider collects
bone aspirate specimens for culture and sensitivity and applies a cast to
the adolescent’s lower leg. What action should the nurse implement next?

A. Provide a high-calorie, high-protein diet


B. Initiate parenteral antibiotic therapy …..you give antibiotic after c/s.
C. Administer antiemetic agents
D. Encourage partial weight-bearing

36. A child with heart is receiving the diuretic furosemide (Lasix) and has a
serum potassium level 3.0 mEq/L. Which assessment is most important
for the nurse to obtain? K=(3.5-5.1)

A. Dietary intake of potassium rich foods


B. Skin turgor
C. Heart rate and cardiac rhythm
D. Urinary output

37. It is determined that a client with breast cancer has metastasis to the
liver. What is the most likely explanation for the client’s risk of
developing hemorrhagic tendencies?

A. The inability of the liver to synthesize clotting factors


B. The presence of a lowered red blood cell count
C. The loss of clotting factors resulting from chemotherapy
D. The loss of serum proteins found in edematous fluid

38. The healthcare provider performs a paracentesis on a client with ascites


and 3 liters of fluid are removed. Which assessment parameter is most
critical for the nurse to monitor following the procedure?
A. Pedal pulses
B. Breath sounds
Vital signs (to monitor BP)
C. Gag reflex
39. Which instruction should the nurse provide a client who is taking the
conventional antipsychotic medication chlorpromazine (Thorazine) for
schizophrenia?

A. Immediately report rigidity of the musculoskeletal system or a


sudden high fever ……schizo-report jay
B. For best effect, take the medication in the morning
C. Have the white blood cell count checked routinely
D. Notify the healthcare provider if a sore throat or ulcerations in the mouth
occur

40. A client is receiving a continuous infusion of normal saline at 125 ml per


hour. The nurse prepares to change the primary IV tubing and hang a new
bag of normal saline to maintain the prescription. In which sequence
should the nurse implement the procedure? (Arrange with first on top and
last on bottom)

1. Spike a new bag of normal saline 1000ml using new tubing


2. Open the clamp on the tubing to bleed all air from the line
3. Close the clamp below the pump and at the client’s venous access
4. Place the pump on hold and replace the tubing in the chamber
5. Attach the distal end of the new tubing to the client’s venous
6. Open all tubing clamps to the client and start the pump

41. A client who is wheelchair bound demonstrates a positive Thomas test


after admission to the rehabilitation unit. The healthcare provider
prescribes positioning the client prone for 30 minutes three times each
day to prevent further flexion contractures. Based on this finding, what
change in the client’s plan of care should the nurse expect?

A. The client will need surgical intervention


B. There may be a delay in the rehabilitation process (RETRASO)
C. A gel should be provided for the wheelchair seat
D. Discharge will be sooner than anticipated
42. Prostaglandin E2 (Prostin E2) is prescribed for client who had a missed
spontaneous abortion. Which finding should the nurse expect? An
increase in

A. Fetal heart tones


B. Rh antibody production
C. Uterine contractions
D. Hemoglobin (Hgb) levels

43. A client with a general anxiety disorder is pacing the hallway. The client
tells the nurse, “My heart is just racing and sometimes it feels like it’s
fluttering. I’m feeling short of breath and dizzy.” What action should the
nurse implement first?

A. Administer an anti-anxiolytic
B. Escort the client to a quiet room
C. Initiate a diversionary activity
D. Obtain the client’s signs (vital signs to determine the heart rate

44. A 22-year-old is involved in a motor vehicle collision and the spinal


cord is severed at the second cervical spine (C-2). What is the most likely
outcome of this injury?

A. Death at the scene of the accident due to respiratory arrest


B. Lifetime need to wear a cervical neck and back brace
C. Paraplegia without regaining any lower motor functions
D. Complete recovery with a chance of having a normal life

45. A nurse plans to call the healthcare provider to report an 0600 serum
potassium level of 2 mEq/L, but the charge nurse tells the nurse that the
healthcare provider does not like to receive early morning calls and will
make rounds later in the morning. What action should the nurse take?

A. Disregard the advice of the charge nurse and contact the healthcare
provider immediately to report the laboratory value
B. Ask the nurse arriving at 0700 to report the lab value to the healthcare
provider during morning rounds
C. Flag the client’s medical record so the healthcare provider will see the
results immediately upon arriving on the unit
D. Ask the charge nurse to contact the healthcare provider with the
laboratory result as soon as possible during the morning

46. The nurse should carefully assess the client with which urinary problem
for fluid volume deficit?

A. Enuresis
B. Polyuria (PO---for poatssium)
C. Dysuria
D. Frequency

47. What nursing intervention should the nurse include in the plan of care
for a client following a bone marrow aspiration?

A. Follow-up hematological laboratory studies


B. Application of warm, moist compresses to the puncture site
C. Proper positioning of the client in a prone position
D. Use of a compression dressing for firm pressure to the site

48. An outcome for treatment of peripheral vascular disease is, “The client
will have decreased venous congestion”. What client behavior would
indicate to the nurse that this outcome has been met?

A. Wears protective shoes


B. Quits smoking
C. Avoids prolonged sitting or standing
D. Avoids trauma and irritation to skin

49. When assessing a client, the nurse notices a pulsation below the
umbilicus. Upon auscultation of the area, a “swishing” sound is detected.
Based on these findings, what additional assessment should the nurse
perform?

A. Measure the blood pressure


B. Check for fecal occult blood
C. Palpate for bladder distention
D. Auscultate the breath sounds
50. An alert and oriented client requiring droplet precautions is placed in a
private room at the end of the hallway. Several days later, the nurse finds
that the client is restless and anxious. What action should the nurse
implement?

A. Encourage family members to maintain a regular visitation schedule


B. Advise unit personnel to enter the client’s room only when necessary
C. Obtain a prescription for a vest restraint from the healthcare provider
D. Transfer the client to a semi-private room closer to the nurse’s station

51. An estrogen preparation is prescribed for a client with prostate cancer.


The nurse should instruct the client to seek immediate medical attention
if what condition should develop?

A. Pain in the calves of the legs)


B. Gynecomastia
C. Testicular atrophy
D. Impotence

52. A client with carpal tunnel syndrome is in the out-patient surgical unit
after an endoscopic carpal tunnel release. What instructions should the
nurse provide the client regarding postoperative care?

A. Change the dressing after the first 12 hours


B. Sip on clear liquids for 24 hours
C. Elevate the hand above the heart…..carpal/hand
D. Keep the fingers immobilized until follow-up

53. Which client should the nurse assess first? A client with

A. Gastroesophageal reflux who reports increasing episodes of belching and


nausea
B. Inflammatory bowel syndrome who reports left lower quadrant
abdominal pain
C. Celiac disease who has developed frequent episodes of watery diarrhea
D. An inguinal hernia who has developed abdominal distention and
fever in the last 8 hours…..moms first. heernia
54. A client with a chest tube develops sepsis and dyspnea. Based on
findings in the client’s medical record, which prescription should the
nurse implement first? Click on each chart tab for additional information.
Please be sure to scroll to the bottom right corner of each tab to view all
information contained in the client’s medical record.)

A. Initiate ..
B. Give the …
C. Start IV …
D. Place ..

55. The nurse is teaching a client with COPD about health promotion
activities. What is the most important advice the nurse should give this
client?

A. “Avoid vigorous exercise”


B. “Avoid heavy meals and eat six small meals daily”
C. “If you have not already done so, quit smoking”
D. “Limit your intake of high fat foods”

56. A client at 38-weeks gestation complains of abdominal pain. The nurse


notes that her abdomen is rigid. What is the probable cause of these
findings?

A. Abruptio placenta….rigid is abruptio, like attend to it abruptly/rigidl


B. Preeclampsia
C. Progression to the transition stage of labor
D. A low-lying placenta previa

57. The nurse assess the perineum of a client who is complaining of perineal
pain 6 hours after a normal delivery, and finds that the client has small
perineal (vulvar) hematomas. Based on this assessment finding, which
treatment should the nurse implement?

A. Prepare the client for surgical excision of the hematomas


B. Instruct the client to use a warm water spray over the perineum
C. Spray a topical analgesic to the perineum
D. Apply cold packs to the perineum
58. Family members of a client who is in hospice care discuss with the nurse
their fears that their loved one’s death will be painful. Which intervention
should the nurse implement?

A. Collaborate with the hospice chaplain to respond to the family’s fears


B. Offer the family members assurance that death is not painful
C. Notify the healthcare provider of the need to increase the dose of pain
medication
D. Provide teaching about available pain control options that might be
helpful

59. Which nurse’s behavior is a breach of client confidentiality according to


the Health Insurance Portable Accountability Act (HIPAA) regulations?

A. Mails privileged health information (PHI) through the US postal service


B. Calls a client by both the first and last name in a public waiting room
C. Tells the ambulance healthcare provider about the client’s history
D. Takes home a daily report sheet with the information of the team’s
client’s

60. The nurse is assessing a client following hemodialysis. What finding


indicates that an expected outcome of dialysis was achieved?

A. Decrease in BP
B. Hemoglobin WNL
C. Increased urinary output
D. Weight gain

61. The nurse is performing a routine well-child exam on a 5-year-old.


While palpating the lymph nodes, the nurse feels several 0.5 cm nodes in
the cervical area that are round, mobile, non-tender, and non-warm to the
touch. What do these findings most likely represent?(NORMAL)

A. An abnormal finding in need of further investigation


B. A sign of acute lymphadenitis
C. An expected finding for a well child of this age
D. An indicator of early stage mumps
62. A client with acute renal failure has many complications. The nurse
recognizes what finding as a sign of an immediate life-threatening
situation?

A. An increased serum potassium concentration


B. A decreased hemoglobin
C. An increased serum sodium concentration
D. An elevated BUN

63.The nurse completes the Leopold maneuvers for a primipara who is


admitted in active labor and determines that the fetus in the right sacral
anterior (RSA) position. On which quadrants should the nurse place the
external fetal heart transducer? (Click the chosen location. To change,
click on the new location)

Answer: Right-up quadrant

64. The practical nurse (PN) reports the patterns of urinary frequency and
volume for several clients. Which finding necessitates further assessment
by the RN?

A. Voiding 300 ml clear yellow urine q4h


B. 400 ml amber urine by straight catheter q6h
C. Voiding 50 ml cloudy urine every hour
D. Total indwelling catheter output of 1800 ml in 24 hours

65.Before administering diltiazem (Cardizem SR) the nurse notes that the
client’s blood pressure is 140/94. What action should the nurse take?

A. Administer the scheduled dose of diltiazem and initiate cardiac


telemetry monitoring
B. Administer the scheduled dose of diltiazem and monitor the client’s
blood pressure
C. Administer the scheduled dose of diltiazem and contact the healthcare
provider
D. Hold the scheduled dose of diltiazem and advise the client to remain on
bedrest
66. A male client sitting in his room tells the nurse “The CIA put this
transistor right here under my left ear. They are transmitting messages.
Can’t you hear them? They’re so loud they scare me.” Which response is
best for the nurse to provide?

A. “Do you think others hear the message?”


B. “The messages scare you?”
C. “How long have you been hearing the message?”
D. “What is the message telling you”

67. The nurse is preparing a discharge plan for an older client who was
recently diagnosed with Alzheimer’s disease. Which intervention should
the nurse suggest to the spouse if the client becomes uncooperative at
home?

A. Restrain the client when agitated or combative


B. Provide explanations of procedures to the client
C. Reinforce reality orientation when the client is forgetful
D. Ensure a calm and predictable environment

68. A client in the third trimester of pregnancy reports that she feels some
“lumpy places” in her breasts and that her nipples sometime leak a
yellowish fluid. She has an appointment with her healthcare provider in
two weeks. What action should the nurse take?

A. Obtain additional data by asking the client if her areolas have become
darker
B. Recommend that the client wear a supportive brassiere to prevent
leaking of fluid
C. Rescheduled the client’s prenatal appointment for the following day
D. Explain that this normal, but can be assessed further at the next
prenatal visit

69. The nurse should instruct the parents of a 2-year-old toddler with
Tetralogy of Fallot to immediately contact their healthcare provider if
their child exhibits which symptom?

A. Assumes knee-chest position


B. Becomes pale and lethargic (cyanosis)
C. Has respiratory rate of 34 breaths/minute
D. Clubbing of the fingers

70. A client with end stage renal disease (ESRD) is undergoing peritoneal
dialysis. What observation made by the nurse during the peritoneal
dialysis treatment warrants immediate intervention?

A. A 2,000 ml amount of dialysate was instilled, and 1,500 ml was


drained
B. A weight loss of 2 pounds since yesterday was recorded after dialysis
C. The client complains of abdominal fullness during the dialysis
treatment
D. The dialysate takes about 20 minutes to drain, and is straw-colored

71. A hospitalized male veteran of a foreign war refuses care from a Middle-
Eastern nurse. The client tells the nurse, “I want an American to take care
of me!” Which action should the charge nurse take?

A. Explain to the client that the nurse is capable of providing competent


care
B. Reassign the client’s care to another nurse….obey veteran
C. Submit a referral to the ethics committee
D. Discuss with the nurse the best methods for addressing the client’s
biases

72. The community health nurse is attempting to address the issue of child
abuse in a large metropolitan area. A primary prevention program for
child abuse might include which program?

A. Foster care programs for placement of abused children


B. High school child development and parenting classes
C. Support groups for abused children
D. Anger management classes for abusive parents

73. A client’s right to give informed consent is based on which ethical


principle?

A. Nonmalfeasance
B. Justice
C. Autonomy
D. Beneficence
74. A client who suffered a stroke and is now on a ventilator receives
nutritional supplements by the feedings three times a day. The nurse
checks the client for a residual volume before administering the next
feeding. Which statement best describes the rationale for this nursing
intervention?

A. Mixing fresh formula with older formula in the client’s stomach often
causes nausea
B. Retention of feeding in the stomach increases the likelihood of
regurgitation and aspiration
C. Aspiration of residual feeding is the best indicator that the tube is in the
stomach
D. The efficiency of gastric digestion should be determined by analyzing
the pH of the residual feeding

75. A client has a history of chronic atrial fibrillation. Which instruction


should the nurse include in the teaching plan for this client?

A. Weigh daily, at the same time, in the same clothes


B. Be sure to take the prescribed daily aspirin
C. Auscultate apical heart rate every morning for a full minute
D. Keep a record of fluid intake and output

76. A client’s telemetry monitor indicates ventricular fibrillation (VF). What


should the nurse do first?

A. Prepare for synchronized cardioversion


B. Administer atropine intravenous bolus
C. Prepare to defibrillate
D. Administer lidocaine intravenous bolus

77. It is most important for the nurse to use an IV pump and/or Buretrol, an
in-line volume-control device, when initiating IV therapy for a client
following which surgical procedure?

A. Colostomy
B. Total hip replacement
C. Femoral-popliteal bypass
D. Craniotomy….as in pure craniotomy, thus Bure craniotomy
78. Which strategy is most important for the nurse to use when assisting a
client with myasthenia gravis to devise a daily routine?(EN LA
MANANA ES CUANDO LOS MUSCULOS ACEPTAN MEJOR LOS
EJERCICIOS)

A. Select a physical diversional activity to promote endurance


B. Protect extremities from injury resulting from decreased sensation
C. Set up daily physical exercise regimen to promote muscle strength
D. Perform necessary physically demanding tasks in the morning

79. The nursing diagnosis, “Altered nutrition: less than body requirements,”
is included in the plan of care for a client with hyperthyroidism. What
primary etiology should the nurse identify when planning care for this
client?

A. Increased metabolic needs


B. Inability to absorb nutrients
C. Pain upon swallowing
D. Disturbed body image

80. The nurse is triaging victims of a tornado that hit a housing area outside
of town. Which client would the nurse issue a black disaster tag to?

A. A 29-year-old female who is 40 weeks gestation and having


contractions
B. A 12-year-old child who has a closed fracture of the right lower leg
C. An 88-year-old male who is complaining of chest pain and dyspnea
D. A 59-year-old female with head injury whose pupils are fixed and
dilated (NEUROLOGI PROBLEM)

81. A male infant born at 30-weeks gestation at an outlying hospital is being


prepared for transport to a level IV neonatal facility. His respirations are
90/min, and his heart rate is 150beats per minute. Which drug is the
transport team most likely to administer to this infant?(PARA PRVENOR
EL COLAPSO PULMONAR ,POR LA INMMADURES DE ESTOS)

A. Ampicillin (Omnipen) 25 mg/kg slow IV push


B. Gentamicin sulfate (Garamycin) 2.5 mg/kg IV
C. Digoxin (Lanoxin) 20 micrograms/kg IV
D. Beractant (Survanta) 100 mg/kg per endotracheal tube….B-baby

82. A 60-year-old female client takes NPH insulin each morning. What
would necessitate holding this client’s usual morning NPH insulin dose?

A. The healthcare provider prescribed a sliding scale of regular insulin


before each meal
B. The client is NPO for surgical debridement of a leg ulcer at 11:00
a.m
C. Breakfast trays will be 30 minutes late
D. The client’s 7:00 a.m blood sugar is 200 mg/dl

83. Two weeks following a fracture, a male client is told by the healthcare
provider that a callus has formed at the fracture site. The client expresses
concern to the nurse about the significance of this information. How
should the nurse respond?

A. Offer reassurance that the healing process is progressing


B. Advise the client that the callus may need to be removed
C. Explain the use of preventive anticoagulants to the client
D. Provide an opportunity for the client to express his anger

84. A 25-year-old female client, a dancer, has just had an ileostomy as a


result of Crohn’s disease. In evaluating the client’s response to this life
change, which behavior would indicate to the nurse that he is coping
effectively? The client

A. Discusses modified costume designs (MODIFICAR LA FORMA


DE VESTIRCE ,,SUELEN HACERLO MUY APRETADO)
B. Call the nurse when her ostomy bag is half-full
C. Asks the nurse to teach her mother how to do ostomy care
D. Notifies her employer that she will return to work within one week of
the surgery

85. The nurse is preparing to insert a saline lock for fluid replacement in a
client with a fluid volume deficit. Which assessment finding is most
relevant to the nurse’s approach to performing the procedure?

A. Altered deep tendon reflexes


B. Flattened veins
C. Thready pulse
D. Prolonged capillary refill

86. The nurse reviews the results of a client’s computerized tomograph scan
(CT), which indicates that a cerebellar infarction is present. Based on this
pathophysiological finding, what nursing diagnosis should the nurse
include in the client’s plan of care?

A. Impaired walking related to loss of balance and coordination-


beacause the cerebellum is for balance and coordination
B. Impaired swallowing related to neuromuscular dysfunction
C. Disturbed sensory perception, visual, related to homonymous
hemianopsia
D. Disturbed body image related to ipsilateral facial paralysis

87. Which finding should the nurse expect a client to exhibit who is newly
diagnosed with fibromyalgia?

A. Unexplained weight gain


B. Itching and rash
C. Recent joint trauma
D. Disruption in sleep patterns

88. A male client with a history of seizures tells the nurse that he obtained a
generic form of his anticonvulsant medication through an online
pharmacy, which was much less expensive than the brand name
medication he has been taking. Which information about the medication
is most important for the nurse to review with the client?

A. Bioequivalency
B. Onset of action
C. Therapeutic index
D. Adverse effects

89. The nurse recognizes that the primary purpose of recommending a


yearly digital rectal examination (DRE) for all men over the age of 40 is
to help detect the early stages of which type of cancer?

A. Cancer of the sigmoid colon


B. Prostate cancer
C. Rectal cancer
D. Bladder cancer

90. The nurse is reviewing the medical history of a client who is scheduled
for a parathyroidectomy. Which disorder in the client’s history is most
likely to be impacted by this surgery?(REDUCE EL CALCIO )

A. Diabetes insipidus
B. Gout
C. Fibromyalgia
D. Osteoporosis

91. A female client reports that she drank ¾ of a liter of a solution to cleanse
her intestines for a colonoscopy. How many ml of fluid intake should the
nurse document?

Answer: 750

92. The nurse observes that a client is receiving oxygen per nasal cannula at
1.5 L/minute as prescribed, but a humidifier is not attached to the
oxygen. What action should the nurse implement?

A. Assess the client’s mucous membranes


B. Call Respiratory Therapy to supply a humidifier
C. Remove the nasal cannula from the client’s nares
D. Teach the client about the need for humidification

93. A 19-year-old male client is brought to the emergency room by a group


of fraternity brothers after a hazing event at the university. The client
arrives with a blood alcohol level (BAL) of 3.8 and a Glasgow Coma
Scale of 3. Which action should the nurse implement first?

A. Insert an indwelling catheter to bedside drainage


B. Administer Narcan 0.4 mg IVP and repeat in 3 minutes
C. Initiate IV access using Lactated Ringer’s solution 1000 ml with
thiamine 100 mg….let it ring on the other fraternity ears
D. Place a nasogastric tube and attach to low continous suction
94. The nurse is preparing to administer vancomycin (Vancocin) 500 mg in
200 ml of DW and based on the manufacturer’s recommendations, the
nurse plans to administer the dosage over 90 minutes. The secondary
infusion pump should be set to administer how many ml/hour?
Answer: 133
95. The nurse is caring for four adults clients: Client A, who has emphysema
and whose oxygen saturation is 94%; Client B, with a postoperative
hemoglobin of 8.7 mg/dl; Client C, newly admitted with a potassium
level of 3.8 mEq/L; and a Client D, scheduled for an appendectomy who
has a white blood cell count of 15,000 mm. What intervention should the
nurse implement? Hgb=M 14-18,F 12-16 K=3.5-5.1,WBC=5-10
MIL,SPO2 95/100

A. Determine if Client B has two units of packed cells available in the


blood bank
B. Ask the dietician to add a banana to Client C’s breakfast tray
C. Inform Client D that surgery is likely to be delayed until the infection
is treated
D. Increase Client A’s oxygen to 4 liters per minute via nasal cannula

96. A client is brought to the emergency room following a massive stab


wound and is bleeding profusely. The nurse knows that this client is most
likely to exhibit what sign?COMO COMPENZACION

A. Tachycardia …….profusely, fast, tachycardia


B. Fatigue
C. Low hemoglobin
D. Hypertension

97. When planning nursing care for immobilized clients, the nurse should
consider which physiological alterations that frequently occur with
immobility? (Select all that apply)

A. Irregular heart rate


B. Venous pooling
C. Bony demineralization
D. Urinary stasis
E. Decreased respiratory capacity
98. A client who has localized eczematous eruptions on both hands is
diagnosed as having contact dermatitis. What instruction should the nurse
include in this client’s discharge teaching plan?PARA LA PICAZON
A. Wear latex gloves whenever outdoors
B. Take prescribed antihistamine near bedtime
C. Soak hands in warm soapy water three times a day
D. Apply an oil-based ointment to the affected areas

99. Before administering a prescribed dose of tetracycline (Achromycin),


what serum lab test should the nurse monitor?M=0.6/1.2,F=0.5/1.1

A. Glucose
B. Uric acid
C. Calcium
D. Creatinine ……cycline and creatinine

100. The nurse identifies a priority diagnosis of “Altered comfort related to


menstrual cramps” for a 25-year-old female client. Which self-care
activity should the nurse emphasize in the client’s teaching plan?

A. Abdominal wall strengthening


B. Pelvic floor exercises
C. Regular aerobic exercise
D. Weight-bearing activities

101. A mother brings her 15-month-old son and 6-year-old daughter to the
clinic for immunizations. Both children are fretful and obviously have
upper respiratory infections. The mother tells the nurse that the younger
child ran a fever of 100.2 F following his last immunization. What plan is
best for the nurse to implement?

A. Rescheduled the older child’s immunization until the upper respiratory


infection subsides
B. Withhold the younger child’s immunization until allergy studies can be
completed
C. Explain that a history of a fever after immunizations may indicate the
need to alter the scheduled of boosters
D. Administer the boosters and instruct the mother to cal the clinic if
either child’s temperature exceeds 101 F
102. The nurse is assessing an unresponsive client who ingested an
unknown number of meperidine (Demerol) 50 mg tablets. Naloxone
(Narcan) 0.4 mg IV is administered, and the client is now responding to
verbal stimuli. Which finding in the next hour requires immediate action
by the nurse?

A. Tachycardia
B. Rounding headache
C. Difficulty in arousing(DESPERTAR)
D. Cold, clammy skin

103. The nurse is caring for a client in the Medical Intensive Care Unit.
What problem is a client probably experiencing who has an easily
obliterated radial pulse and below-normal pressures, including blood
pressure (BP), central venous (CVP), pulmonary artery pressure (PAP),
and pulmonary capillary wedge pressure (PCWP)?

A. An acute myocardial infarction


B. Acute congestive heart failure
C. Hypovolemic shock…..lots of volume shock
D. Cardiogenic shock

104. The nurse is providing preoperative teaching to a client scheduled for


vertical banding gastroplasty. In preparing the client for the immediate
postoperative period, which intervention is most important for the nurse
to implement?(PARA EVITAR VOMITOS)

A. Suggest dietary selections of high protein liquids in the immediate


postoperative period
B. Show the client a nasogastric tube and explain reasons for low
intermittent suction
C. Prepare for monitoring in the intensive care unit during the first
postoperative day
D. Refer for psychological counseling to focus on altered body image and
behavior

105. A male client with cancer is admitted on the oncology unit and tells
the nurse that he is in the hospital for palliative care measures. The nurse
notes that the client’s admission prescriptions include radiation therapy.
What action should the nurse implement?
A. Advise the client that palliative care measures can be implemented by a
hospice in an outpatient setting
B. Reassure the client that radiation treatments can often cure or control
cancer with minimal side effects
C. Consult with the client about his expected goals for his
hospitalizations and current treatment plan
D. Consult with the healthcare provider about the client’s wish to cancel
further radiation treatments

106. The nurse is teaching a client with Addison’s disease about this new
diagnosis. What pathophysilogical explanation should the nurse share
with the client?

A. Adrenal insufficiency is an autoimmune dysfunction that results


from white blood cells damaging the adrenal cortex
B. Pituitary dysfunction, such as diabetes insipidus, can occur after a head
injury or primary tumor that causes increased intracranial pressure
C. End stage renal disease causes hypertension due to decreased renal
perfusion that results in an increased secretion of rennin
D. Hyperthyroidism is an autoimmune disease that causes an increased
secretion of thyroxine resulting in an increased basal metabolic rate

107. The first day after a cesarean section, when being assisted to the
bathroom for the first time, a primipara client experiences a sudden gush
of vaginal blood and notices that several blood clots are in the toilet.
What action should the nurse take?

A. Check fundal consistency and continue to monitor the lochial flow


amount
B. Insert an indwelling catheter to empty the bladder and contract the
fundus
C. Return the client to bed and maintain bedrest until the lochial flow
slows
D. Massage the fundus and avoid direct pressure on the cesarean incision

108. What is the priority nursing diagnosis for a client with restless legs
syndrome?

A. Altered tissue perfusion


B. Impaired mobility
C. Self-care deficit
D. Disturbed sleep patterns (restless disturbs your sleep)

109. A client is receiving an IV of 500 ml NS with 20,000 units of heparin


at 27 ml/hr. The nurse wants to verify that the client is receiving the
prescribed amount of heparin. How many units is the client receiving
every hour?

Answer: 1080

110. A combination multi-drug cocktail is being considered for an


asymptomatic HIV-infected client with a CD4 cell count of 500. What
nursing assessment is most crucial in determining whether therapy should
be initiated?(COMPLACIENTE)

A. Is able and willing to comply with complex drug schedules


B. Understands the effects and side effects of the various drugs
C. Has an adequate social support structure in place
D. Meets qualifications for a prescription assistance program

111. Which client is at the greatest risk for suicide and should be managed
with close observation?

A. A 35-year-old married Hispanic male who recently lost his job


B. A 28-year-old Philipino female who lives with her parents
C. A middle-aged Jewish female whose mother attempted suicide
D. A widowed White male who is a veteran of the Korean War
112. A client with a deep vein thrombosis is receiving a heparin protocol
based on a target partial thromboplastin time (PTT) of 65 to 95 seconds.
The client’s current PTT result is 35 seconds. What action should the
nurse implement?(CUAGULA MUY RAPIDO)

A. Decreasing the rate of the heparin infusion


B. Leaving the heparin infusion at the current rate
C. Increasing the rate of the heparin infusion
D. Discontinuing the heparin infusion

113. A client with allergic rhinitis expresses concern about “giving this
runny nose” to her young children. What nursing action has the highest
priority?

A. Assure the client that her are not contagious


B. Instruct the client to rest and drink plenty of fluids
C. Assess the client’s vital signs and breath sounds
D. Reinforce the need for regular hand washing

114. The nurse observes that a client has received 250 ml of 0.9 % normal
saline through the IV line in the last hour. The client is now tachypneic,
and has a pulse rate of 120 beats/minute, with a pulse volume of +4. In
addition to reporting the assessment findings to the healthcare provider,
what action should the nurse implement?(OVERLOAD FLUID)

A. Increase the rate of the current IV solution


B. Change the IV fluid to 0.45% normal saline at the same rate
C. Discontinue the IV and apply pressure at the site
D. Decrease the saline to a keep-open rate

115. A hospitalized client’s bronchoscopy specimen culture result indicates


the presence of the Mycobacterium tuberculosis organism. Which
intervention is most important for the nurse to implement?

A. Use only disposable drinking glasses and personal care items


B. Don a gown and gloves when handling the client’s soiled linen
C. Don a surgical mask before entering the client’s room
D. Put the client in a room with a negative airflow system

116. Four clients arrive at the labor and delivery nurse’s station at the same
time. Which client should the nurse assess first?

A. A 38-week multigravida with biophysical profile score of 4out of 8


B. A 40-week primigravida who reports contractions occurring every 10
minutes
C. A 39-week multigravida who is scheduled for a repeat cesarean section
today
D. A 37-week primigravida with a prescription for serial blood pressures

117. The industrial health nurse who works in a mobile clinic is developing
an exposure control plan for blood-borne pathogens. Which topics should
be included in this plan? (Select all that apply)

A. Masks for respiratory chemicals and toxins


B. Puncture-resistant needle containers
C. Self-sheathing or needleless medication systems
D. Hepatitis B vaccine series
E. Negative pressure environments

118. The nurse notes that a client is experiencing supra-ventricular


tachycardia (SVT). Which action should the nurse implement?

A. Place a crash-cart at the client’s bedside


B. Call a code and start CPR immediately
C. Assess the client’s heart sounds and vital signs
D. Prepare to administer adenosine, an antidysrhythmic

119. A client diagnosed with acute epididymitis secondary to a sexually


transmitted disease receives a prescription for ceftriaxone (Rocephin).
Prior to administering the prescription, which question should the nurse
ask the client?

A. “How much time to you spend in the sun or using a tanning bed?”
B. “Have you ever had an allergic reaction to any other antibiotic?”
C. “Do you operate dangerous equipment in your job or for recreation?”
D. “Do you drink alcohol and if so what kind and how much a week?”

120. The community health nurse is planning a cardiac rehabilitation


program that will be offered in a neighborhood heavily populated by
African-Americans. In preparing instruction on health promotion topics
for this program, which approach should the nurse use to address these
client’s modifiable risk factors?

A. Offer a cooking class that incorporates low cholesterol recipes


B. Start a support group for African-American males who have had heart
attacks
C. Help clients complete a medical history form that shows which relatives
have hypertension
D. Prepare a poster illustrating the high incidence of heart disease among
African-Americans

121. The nurse is teaching a client newly diagnosed with diabetes mellitus
the signs of hypoglycemia. What symptom should be included in the
description of early signs of hypoglycemia?

A. Polyuria
B. Tremors
C. Difficulty swallowing
D. Bradycardia

122. The nurse identifies the nursing diagnosis of, “Visual


sensory/perceptual alterations related to increased intraocular pressure
(IOP)” for a client with glaucoma. Which nursing intervention should the
nurse include in the plan of care?

A. Encourage compliance with drug therapy to prevent loss of vision


B. Develop pain management strategies associated with ocular nerve
atrophy
C. Recognize that damage to the eye can be reversed until late stages of
the disease
D. Identify coping mechanism related to the eventual loss of peripheral
vision
123. Penicillin G procaine (Wycillin) 135,000 units IM is prescribed for an
infant with a middle ear infection. The drug is available in a vial of
1,200,000 units /2ml. How many ml should the nurse administer?
Answer: 0.23
124. A mother tells the clinic nurse that the healthcare provider wants her
to begin introducing solid foods to her 4-month-old infant. The nurse
should recommend introducing foods in what order?

1. Rice cereal, iron fortified


2. Strained apple sauce
3. Strain green beans
4. Strain pureed chicken

125. The nurse is caring for a client who was admitted two hours ago with
confusion, Kussmaul respirations, and warm, flushed skin. The
healthcare provider determines the client is in acute renal failure (ARF).
Which intervention is most important for the nurse to include in this
client’s plan of care?(DISMINULLE O2 EN SANGRE)

A. Cardiac telemetry
B. Hourly neurological assessments
C. Renal replacement therapy referral
D. Seizure precautions

126. Based on the Braden Risk Assessment Scale, which client is at highest
risk?

A. A 60-year-old male who is lethargic, with dysphasia, and is


experiencing bladder
B. A 30-year-old male who is paraplegic and performs self-catheterization
q6h
C. A 90-year-old female with a history of failing when walking alone at
home
D. A 50-year-old female who has a draining, infected surgical wound and
a fever

127. A client diagnosed with a myxedema coma has assessed vital signs of:
T 99.8F, P 92, R 22, B/P 108/70. Based on this information, what
intervention should the nurse implement first?
A. Assess the client for presence of infection
B. Notify the healthcare provider immediately
C. Encourage the client to use an incentive spirometer
D. Monitor the vital signs q1h for the next 8 hours
128. Following a fracture, a client develops early symptoms of anterior
tibial compartment syndrome. In planning care, the nurse identifies the
prevention of what problem as the priority goal?

A. Infection
B. Embolism
C. Ischemia
D. Ecchymosis

129. The nurse is teaching a client’s caregiver how to cleanse around a


wound drain. What is the best way to explain the proper cleansing
technique?

A. Start at the clean area several inches away from the drain to avoid
contaminating the drain
B. Start at the most inflames area, to protect the tissue and promote healing
C. Start at the area with the most drainage, to avoid infecting other areas
D. Start at the drain site, to avoid bringing skin bacteria toward the wound

130. During discharge teaching the mother asks why her premature infant
should get monthly Synagis (Palivizumab) injections. The nurse’s
response should be based on what information?

A. These injections prevent retinopathy of prematurity caused by high


levels of oxygen
B. Monthly injections promote normal neurological and physical
development
C. This medication provides surfactant, which helps the lungs mature more
quickly
D. This drug protects the premature infant from respiratory syncytial
virus (RSV)
131. The school nurse is planning an anti-smoking program for high school
students. In developing the content for the program, which approach is
likely to be most effective with this group of students?

A. Have fellow students who smoke describe the problems that


smoking causes in their daily lives
B. Have a person who has quit smoking talk to the students about smoking
cessation
C. Show pictures of old men and women who have smoked all their lives
and are now dying of lung cancer
D. Provide public awareness information from the National Institute of
Health

132. The nurse is giving medications to a client who is admitted to the


hospital with a diagnosis of diabetes mellitus. After checking the
fingerstick glucose at 1630, what dose of insulin should the nurse
administer?
Answer: 6

133. The nurse’s assessment of a client admitted with a diagnosis of


diabetic ketoacidosis (DKA) include: scant urinary output, serum
potassium level of 2.5 mEq/L, blood pH of 7.26, temperature 98 F, pulse
128 beats/minute, respirations 36 breaths/minute, and blood pressure
90/52. Which prescription is most important for the nurse to implement?

A. Dopamine IV at 5 mcg/kg/minute
B. Potassium IV at 20 mEq/250 ml over 1 hour
C. Sodium bicarbonate IV at 1 mEq/kg
D. Lasix 20 mg IV push

134. In completing the treatment plan for an 11-year-old who was bipolar
disorder, the nurse plans outcomes for the nursing diagnosis, “Risk for
violence towards peers related to impulsivity.” Which outcome is most
important?

A. Continues taking all medications as prescribed


B. Participates in daily physical exercise to release energy
C. States two reasons for taking mood stabilizing medication
D. Seeks out staff when having thoughts of harming others busca
personal al tener pensamientos de hacer daño a otros

135. Fluids are restricted for a 4-year-old boy with acute poststreptococcal
glomerulonephritis (APSGN). Which nursing intervention makes fluid
restriction less obvious to this child?

A. Pour the full allotment of liquids in a single container and instruct the
child to drink a little at time
B. Give the child crayons and show him how to record intake and output to
help keep him distracted
C. Fill regular cups and glasses half-full and don’t say anything to the
child because it will not be noticed
D. Play a game of tea party and serve the allowed amount of liquids in
small medicine cups

136. A 6-month-old male with brochiolitis is admitted to the hospital. In


monitoring the respiratory status of this child, which symptom indicates
to the nurse that he is experiencing respiratory distress?

A. Abdominal breathing
B. A high pitched cry
C. Respiratory rate of 62 breaths/minute
D. Dry, flushed skin

137. The nurse is caring for a client who has a transcutaneous electrical
nerve stimulator (TENS) unit that was inserted at the incisional site
following a lumbar laminectomy. What information should the nurse
teach the client about the action of this pain modality?

A. A mild electrical stimulus at the skin surface blocks transmission of


pain stimulus
B. The spinal cord is stimulated to release endorphins to help ease the pain
C. A cartridge inside the unit injects an analgesic into the epidural space to
control pain
D. Operation of the unit helps to distract the client from experiencing the
pain
138. An elderly client is admitted with a diagnosis of pneumonia. What
sign or symptom would require immediate intervention by the nurse?

A. Complains of having chills and is febrile


B. Has become agitated, aggressive, and confused (O2 BAJO)
C. Is coughing and expectorating purulent secretions
D. Has egophony and rhonchi upon auscultation of the breath sounds

139. During a family baseball game, an adult male is hit on the head with a
bat, and he is suspected of sustaining an epidural bleed. What is the most
important information for the emergency center nurse to obtain from the
client’s spouse, who witnessed his injury?

A. “Does your husband have advanced directive?”


B. “What time of day did the injury occur?”
C. “Did your husband report having double vision?”
D. “Was your husband knocked out by the blow?” Fue su marido
noqueado por el golpe?"

140. An 86-year-old female client complains to the nurse that she does not
like to eat as much as she used to because things taste differently to her
now that she is older. The nurse’s response should be based on which
fact?

A. Older people often use poor taste sensation as an excuse to avoid eating
foods they do not like
B. Taste sensation decreases in older adults because of diminished gastric
secretions
C. A loss of appetite often occurs in older adults as a result of a
decreased sense of smell
D. Poorly prepared meals and eating alone are the usual causes of a
decreased appetite in older adults

141. A client with a compound fracture of the left ankle is being discharged
with a below-the-knee cast. Before being discharged, the nurse should
provide the client with what instruction?
A. Do not attempt to scratch the skin under the cast No intente arañar la
piel bajo el yeso
B. Apply a cold pack to any “hot spots” on the cast
C. Keep the left leg in dependent position
D. Apply heat to the leg cast

142. The charge nurse is implementing a quality assurance policy and


accompanies a nurse while administering medications. The nurse
identifies a male client by asking him to state his name prior to
administering the medication. Which action should the charge nurse
implement?

A. Provide a medication irregular occurrence form for the nurse to


complete
B. Correct the nurse’s action while administering the medication to the
client
C. Tell the nurse in a private area that the client identification was
incomplete
D. Take no action since the nurse is administering the medication correctly

143. A mother in the well-baby clinic reports that her 3-month-old infant
frequently spits up formula. Based on this complaint, what action should
the nurse take?

A. Ask the mother whether the vomiting is forceful in nature


B. Perform as abdominal examination
C. Assure the mother that this is expected in newborns
D. Ask the mother to describe the infant’s stools

144. While assigned to care for clients on a surgical unit, the nurse receives
a personal phone call about a family emergency that requires the nurse to
leave immediately. What action by the nurse is most important?

A. Maintain confidentiality regarding the situation leading up to the


emergency
B. Complete a personnel request form to document the need for leaving
early
C. Advise the clients that another nurse will assume responsibility for their
care
D. Notify the charge nurse of the situation and of the need to leave
immediately

145. A 50-year-old male client has just been informed that he will require
open heart surgery. He tells the nurse, “This will change my whole life.
Nothing will ever be the same again.” What action should the nurse
implement first?

A. Offer reassurance that most men his age can return to their former
activities
B. Provide client teaching about the postoperative period and rehabilitation
program
C. Invite a client who has recovered from the same surgery to speak with
the client
D. Encourage the client to discuss his perceptions of the changes his
life will undergo

146. The nurse-manager determines that the frequency of medication errors


has remained the same despite the implementation of new policies related
to client identification before administering medications. What action
should the nurse-manager take next?

A. Analyze the frequency with which nursing staff members have


implemented the new policies
B. Collaborate with the hospital pharmacy department to develop revisions
in the existing policy
C. Provide additional education programs regarding the need for
implementing the new policies
D. Determine what additional policies are needed to reduce the frequency
of medication errors

147. The nurse is administering oxygen to a client with pulmonary edema


when a family member asks the nurse why the client needs oxygen.
Which pathophysiological mechanism should the nurse explain to this
family member?
A. Fluid collects in the chest cavity and keeps the lungs from expanding
B. Fluid leaks out of the small blood vessels into the air sacs of the
lungs
C. The blood supply to the lungs is reduced so the lungs overwork
D. The airway fills with frothy, blood-tinged fluid that moves into the
lungs

148. During shift report, the nurse learns that a postoperative client has
atelectasis. What nursing diagnosis should the nurse expect to include in
the client’s plan of care?

A. Disturbed body image


B. Deficient fluid volume
C. High risk for aspiration
D. Impaired gas exchange

149. The nurse knows that the blood urea nitrogen (BUN) can be expected
to change as one ages. Which statement best explains this expected
changes?

A. BUN increases because of a decrease in renal functioning and an


increase in cardiac output
B. BUN increases because of a decrease in renal functioning and a
decrease in cardiac output de BUN aumenta debido a una disminución en el
funcionamiento renal y una disminución en el gasto cardíaco

C. BUN decrease because of a decrease in renal functioning and an


increase in glomerular filtration rate
D. BUN decreases because of an increase in renal functioning and a
decrease in glomerular filtration rate

150. A primigravida at 37-weeks gestation presents to the antenatal unit for


a nonstress test (NST) because she has not felt her baby move in the last
8 hours. The nurse applies an external fetal monitor and provides the
woman with orange juice, but no fetal movement occurs. What should
the nurse do next?

A. Initiate vibro “acoustic” stimulation


B. Perform a vaginal exam
C. Begin a biophysical profile
D. Change the mother’s position

151. The nurse is assessing a 2-week-old breastfeeding infant. To obtain


information about adequate nutrition, which question should the nurse
ask the breastfeeding mother?
A. “How many times does the baby nurse in 24-hour period?”
B. “How many diapers does the infant wet daily?”
C. “How satisfied does the infant seem with each feeding?”
D. “How long does the baby nurse at each feeding?”

152. The nurse is administering sodium polystyrene sulfonate (Kayexalate)


to a client in a acute renal failure. Which normal finding indicates that the
medication has been effective?(DESARROLA EXESO DE
EXCRECION DE K A TRAVES DE LAS ECE FECALES)

A. Hemoglobin level of 14.5 grams/dl


B. Serum potassium level of 4.3 mE /L K=(3.5/5.1)
C. Serum glucose level of 100 mg/dl
D. Serum ammonia level of 30 micrograms/dl

153. In establishing goals for the client’s plan of care, which information is
most important for the nurse to consider?

A. Evaluation strategies
B. Planned interventions
C. Nursing diagnoses
D. Clustered assessment data

154. The nurse is preparing a discharge teaching plan for the parents of an
infant with phenylketonuria (PKU). What dietary instruction should the
nurse provide to the parents?(is the gene for hepatic enzyme )necesarias
para metabolizar los amino acidos.(newborn screening test)two-week

A. Modify the type of amino acids consumed


B. Remove all protein sources from the diet
C. Give apple juice once a day to maintain hydration
D. Supplement evening feeding with rice cereal

155. An infant is admitted to the newborn nursery, and is believed to have


Down syndrome. Which physical finding might the nurse expect to see?
A. Postural hypotonia
B. Maxillary hypoplasia
C. Fusion of cranial sutures
D. Jeneway spots on the palms

156. The healthcare provider prescribes oxytocin synthetic (Pitocin), 10


units/L via IV drip to augment a client’s labor because she is
experiencing a prolonged active phase. Because the client is receiving
Pitocin, the nurse should closely monitor for which complication?

A. Fetal tachycardia
B. Uterine tetany(contraciones uterinas involuntarias o exeso )
C. Hemorrhage
D. Uterine hypostimulation

157. A female client who has been taking the corticosteroid


methylprednisolone (Solu-Medrol) for three weeks reports to the nurse
that she has gained ten pounds since starting the medication, and she
wants to stop taking it. What is the best response by the nurse?

A. Restricting fluid intake can reduce the weight gain caused by


corticosteroids
B. The weight gain is a desired effect, showing that the appetite has
improved
C. The medication must be discontinued gradually, tapering the dose
each day
D. Weight gain is a sign of toxicity, so the medication should be stopped
immediately

158. The healthcare provider hands a newborn to the circulating nurse


during a cesarean delivery. What action should the nurse implement first?

A. Perform a physical assessment


B. Dry the infant under a warming unit ( secar al bebe y ponerlo en la
encubadora)
C. Determine an APGAR score
D. Allow the mother to touch the infant

159. An infant has a medical diagnosis of tracheoesophageal fistula (TFE).


What nursing intervention is indicated for this infant prior to surgical
repair?

A. Feed small frequent meals


B. Prepare the child for a barium enema to correct the condition
C. Keep suction equipment available at all times Mantener un equipo de
succión disponible en todo momento
D. Give isotonic enemas as prescribed

160. The nurse has identified four nursing problems for a 13-year-old
admitted for depression and anxiety. What is the priority problem?

A. Ineffective coping related to post-traumatic stress of fire that killed


siblings
B. Knowledge deficit regarding purpose of medications and side effects
C. Ineffective health maintenance related to substance abuse and unsafe
sex
D. Risk for self-directed violence related to history of self-mutilation

CAT Version 3
1. An adult client being admitted to the psychiatric unit with a diagnosis of
bipolar disorder arrives in an elated state. What is the best room
assignment the nurse can make for this client?

a. A quiet room away from the nurse’s station.

b. A bright-colored room located near the recreation room.

c. A room that contains very little furniture.

d. A room that has at least two other clients assigned to it.


2. The community mental health nurse is planning to visit four clients with
schizophrenia. Which client should the nurse see first?

a. A mother who took her children from school because aliens were
after them.

b. A young man who has a history of substance abuse and had no


telephone.

c. A newly diagnosed client who needs to be evaluated for medication


compliance.

d. A young man recently released from prison who requires an intake


assessment.

3. A female client, the mother of two small children, appears depressed after
learning from her healthcare provider that she has multiple sclerosis. Which
nursing intervention should the nurse implement first?

a. Provide the client with information about the Multiple Sclerosis


Society.

b. Leave the room so the client has privacy to grieve.

c. Encourage the family to be available to the client as much as


possible.

d. Sit quietly with the client and answer questions she may ask.

4. Which statement by a client with emphysema indicates the best


understanding of the purpose of pulmonary function testing?

a. “I won’t pass the test because I smoke and have emphysema.”

b. “It will measure how well my lungs are working.”

c. “I’m afraid I’ll find out that I have lung cancer.”


d. “This test measures how much oxygen I have in my lungs.”

5. An adolescent male client is admitted to the hospital. Based on Erikson’s


theory of psychosocial development, which nursing intervention best
assists this adolescent’s adjustment to his hospital stay?

a. Invite him to participate in the evening group activity.

b. Schedule frequent private phone calls to his parents.

c. Provide access to a variety of video games in his room.

d. Encourage him to learn his way around the hospital.

6. Which individual may legally sign an informed consent?

a. A 16-year-old mother for her newborn.

b. The friend of an 84-year-old married client.

c. A 56-year-old who questions a proposed treatment plan.

d. A 42-year-old client who is sedated.

7. The nurse working on a psychiatric unit is concerned about providing


ethical and legally defensible care for clients on the unit. Which occurrence
is an indication that a client’s civil rights are being violated? A client who

a. attempted suicide recently is not allowed to wear a belt or have


shoelaces.

b. made threatening phone calls is allowed to make phone calls only


under the supervision of a staff member.

c. refused to take an oral psychiatric medication is administered the


same medication as an IM injection.

d. is anorexic is not allowed to go to the bathroom after meals unless


accompanied by a staff member.
8. Which statement by the mother of a toddler girl indicates to the nurse that
scheduled vaccine should not be administered?

a. “Her throat closed up so bad she couldn’t breathe the last time she
got this shot.”

b. “My child has been running a little fever and has a runny nose and
cough.”

c. “Her baby brother has a virus and has had diarrhea for three days
now.”

d. “Her arm gets all red and hurts a lot every time she gets a
vaccination.”

9. Thirty-six hours after delivery, the nurse assesses a client’s fundus just
above the umbilicus and displaced to the right of the midline. What actions
should the nurse take first?

a. Palpate the bladder for distention.

b. Assess the amount of lochia.

c. Ask the client when her last bowel movement occurred.

d. Catheterize the client and record the amount.

10. A hospitalized 81-year-old female client has numerous complaints and uses
her call button often to summon staff to help her with activities that she is
capable of performing for herself. Which plan might be most beneficial in
dealing with this client?

a. Rotate assignment for this client among staff members so that one
nurse is not overworked.

b. Ask the nursing supervisor to move the client to another unit where
the nurse-client ratio is higher.
c. Set up a meeting with the client, her family, and all staff members to
discuss the client’s demands.

d. Check on the client at designated time intervals and let the client
know when the nurse will return.

11. When giving a cooling bath to reduce the fever of a 3-year-old child, which
action should the nurse include?

a. Gently massage the arms and legs with rubbing alcohol.

b. Pour tepid water over the child’s back and chest.

c. Apply an ice pack to the back of the child’s neck.

d. Stroke the child’s abdomen with cooling lotion.

12. The nurse determines that a client has a potential (high risk) problem. What
is important for the nurse to do when planning care?

a. Document the current manifestations of the problem.

b. Limit the number of interventions being planned.

c. Identify the problem as having a low priority.

d. Direct nursing actions toward reducing risk factors.

13. The nurse notes that the influenza immunization rates are much lower for
certain demographic groups than for others. Which intervention is likely to
be most useful in increasing the rates of immunization in these lower
immunization groups?

a. Radio announcements about the availability of the influenza vaccine.

b. Legislative proposal that mandates influenza vaccination for all.

c. Designated clinics conveniently located in target neighborhoods.


d. Reports describe influenza rates during times of greatest prevalence.

14. The first time a male client stands at the bedside following a total hip
replacement, he reports severe pain in his left calf. What intervention
should the nurse take first?

a. Remind the client of the importance of postoperative mobility.

b. Use a pain scale to evaluate the severity of the pain.

c. Return the client to bed and assess the lower extremities.

d. Transfer the client to a chair and elevate the lower extremities.

15. A client is receiving an IV of 5% dextrose in Lactated Ringer’s solution at a


rate of 100 ml/hour. The client tells the nurse, “I don’t need to eat because
I’m getting all the nutrition I need through this IV needle.” What initial
response would be best for the nurse to provide this client?

a. “Only a very small amount of the calories you need are provided by
your IV.”

b. “It is very important to keep eating. Why don’t you want to eat?”

c. “The IV does contain nutrients, but eating is very important.”

d. “This IV is called hypertonic, and is more concentrated than your


blood.”

16. The nurse documents that a male client with paranoid schizophrenia is
delusional. Which statement by the client confirms this assessment?

a. “The nurse at night is trying to poison me with pills.”

b. “The voices are telling me to kill the next person I see.”

c. “The fire is burning my skin away right now.”

d. “The snakes on the wall are going to eat me.”


17. A male client returns to the acute care unit following surgery with
sequential compression devices in place. The nurse observes that the client
dorsiflexes his feet frequently. What action should the nurse implement?

a. Remove the sequential compression devices while the client


exercises his feet.

b. Advise the client to avoid flexing his feet while wearing the
compression devices.

c. Offer to massage the client’s feet and legs while assisting him with
personal care.

d. Encourage the client to perform foot exercises regularly while his


mobility is limited.

18. Which statement by a 16-year-old male client with acute osteomyelitis in


his leg indicates the best understanding of the appropriate activity level for
his disorder?

a. “During this illness, I need to keep my leg as immobile as possible.”

b. “As long as I don’t re-injure my leg, I can resume my normal


activities.”

c. “I need to exercise my leg as much as possible to maintain muscle


tone.”

d. “I will not be able to participate in contact sports ever again.”

19. The nurse has conducted a cancer prevention community education


program. In evaluating the participant’s understanding of the carcinogens,
what statement indicates an accurate understanding?

a. Carcinogens are the environment and cannot be avoided.

b. Carcinogens are substances that contain cancerous cells.


c. Environmental factors such as sunlight and chemicals can cause
cancer to spread.

d. Substances that change a cell so that it becomes cancerous are


potential cause of cancer

20. A 25-year-old female client is diagnosed with endometriosis by her


healthcare provider. Which nursing diagnosis has the highest priority?

a. Anxiety related to the fear of infertility.

b. Low self-esteem related to potential infertility.

c. Anxiety related to the possibility of surgical intervention.

d. Pain related to extrauterine tissue inflammation.

21. The charge nurse is supervising a newly licensed practical nurse (PN) who is
administering medications. The PN notes that a client with exophthalmus is
scheduled to receive artificial tear drops. What action should the charge
nurse implement?

a. Change staff assignments so an RN can administer medications to


this client.

b. Advise the PN that the charge nurse will administer the medication
to this client.

c. Instruct the PN to hold the medication until the healthcare provider


is contacted.

d. Remind the PN to evaluate the effectiveness of the medication after


administration.

22. The charge nurse is assessing the morning lab work on four clients. Which
client’s laboratory findings should prompts the charge nurse to contact the
healthcare provider immediately?
a. A 50-year-old diagnosed with myocardial infarction who has an
elevated CPK-MB on serial cardiac isoenzymes.

b. A 35-year-old diagnosed with pneumonia having a white blood cell


(WBC) of 13,000 mm

c. A 29-year-old diagnosed with ulcerative colitis having a serum


potassium level of 3.1 mEq/L.

d. A 74-year-old diagnosed with COPD who has ABGs at pH 7.35, PaCO2


49, PaO2 74, HCO3 26.

23. When assessing a client with Raynaud’s disease, which symptoms should
the nurse expect the client to exhibit?

a. Headache, epistaxis and papilledema.

b. Coldness, pain and pallor of fingertips, toes, and tip of nose.

c. Complaints of cramps in the feet or legs after exercise.

d. Feeling of heart beating in the abdomen when lying down.

24. A client is currently receiving an infusion of 25,000 units of Heparin in 500


ml of normal saline at 10 ml/hour. A prescription is received to change the
rate of the infusion to 700 units of Heparin per hour. The nurse should set
the infusion pump to deliver how many ml/hour? (Enter numeric value
only.)

Answer: 14

25. An elderly client is suspected of having a cardiac dysrhythmia. The nurse


knows that it is most important to obtain which information when
assessing this client?

a. Changes in mentation, personality, and behavior.

b. Blood glucose level.


c. Nutritional history, focusing on fat content of the diet.

d. Ability to perform range of motion exercises.

26. A client has developed drug toxicity after receiving a high dose of a
medication with a prolonged half-life. After consulting with the health care
provider, the nurse expects to administer which treatment plan?

a. Several dose of antagonistic medication with a short half life


administered at frequent time intervals.

b. A high dose of an antagonistic medication with a short half life


administered rapidly as a one time dose.

c. Continuous IV infusion of a synergistic drug for the duration of the


toxic drugs half life.

d. Hold the administration of any fluids or medications until the toxic


drug is cleared from the client’s body.

27. The nurse is assigned to care for a group of clients. Based on priority of
need, arrange in the order in which the nurse should assess these four
clients, with the first client the nurse should assess on top and the last
client the nurse should assess on the bottom. The client who has

A T6 spinal cord injury with a blood pressure of 180/98.

Type 1 diabetes mellitus with a blood sugar of 55 mg/dl.

End stage renal disease with a serum creatinine of 10 m.

A total hip prosthesis with a 12 g hemoglobin.

28. A 22-year-old female client calls the public health clinic because her breasts
are tender and she felt several small lumps during a breast self-examination
(SBE). What information should the nurse elicit from the client first?
a. History of familial breast cancer.

b. Technique used to examine the breasts.

c. Daily intake of foods and fluids containing caffeine.

d. Date of last menstrual period.

29. An adolescent female who has been a lacto-ovo-vegetarian for six months
tells the nurse she is experience increased fatigue. What dietary
recommendation should the nurse provide?

a. Eat spinach three times a weeks.

b. Increase intake of fruit to 6 servings per day.

c. Decrease saturated fat intake.

d. Eliminate carbonated soft drinks from the diet.

30. A client who has been admitted to the Emergency Department following a
sexual assault tells the nurse that she wants to use emergency
contraception. Pending the results of a pregnancy test, what information is
most important for the nurse to obtain from this client?

a. Length of time since the rape occurred.

b. Ability to access and pay for emergency contraception.

c. Date of last menstrual period.

d. Understanding of the emergency contraception regimen.

31. A neonate who has congenital adrenal hyperplasia (CAH) presents with
ambiguous genitalia. What is the primary nursing consideration when
supporting the parents of a child with this anomaly?

a. Offer information about ultrasonography and genotyping to


determine sex assignment.
b. Explain that corrective surgical procedures consistent with sex
assignment can be delayed.

c. Discuss the need for cortisol and aldosterone replacement therapy


after discharge.

d. Support the parent in their decision to assign sex of their child


according to their preference.

32. A nurse is assessing a client who as an arteriovenous (AV) graft in the right
forearm for hemodialysis access. The nurse auscultates a bruit over the
graft area. What intervention should the nurse implement?

a. Elevate the extremity.

b. Apply gentle pressure.

c. Assess the client’s temperature.

d. Document the findings.

33. What assessment finding places a client at risk for problems associated with
impaired skin integrity?

a. Capillary refill 5 seconds.

b. Smooth nail texture.

c. Scattered macula on the face.

d. Absence of skin tenting.

34. The nurse is evaluating discharge teaching of an adolescent who had a long
leg cast applied in the emergency department. Which statement by the
adolescent indicates an understanding of cast care?

a. “If my toes are tingling I will elevate my leg above my heart, on


several pillows.”
b. “I should wrap a cloth around a stick before using it to scratch under
my cast.”

c. “I will not be able to take a shower until the cast is removed from my
leg.”

d. “I will put adhesive tape around the edges of the cast if they become
sharp.”

35. What instructions should the nurse include in the discharge teaching plan
of a client who has recently been diagnosed with Parkinson’s disease?

a. How to care for hand splints and prevent skin irritation associated
with their use.

b. Increasing fluid intake to compensate for the chronic diarrhea


associated with Parkinson’s.

c. Side effects to anticipate with the regular use of NSAIDs (non-


steroidal anti-inflammatory drugs)

d. A progressive program of daily exercise to increase muscle strength.

36. Which type of therapeutic bath should the nurse recommend to a client
who is complaining of pruritis?

a. A Betadine bath.

b. An emollient bath.

c. A colloidal bath.

d. An antibacterial bath.

37. A child is to receive vancomycin (Vanconin) 40 mg/kg IV one hour before a


scheduled procedure. The child weighs 44 pounds. How many mg of the
medication should the nurse administer? (Enter numeric value only.)

Answer: 800
38. While interviewing an elderly client, the nurse observes that the client’s
hands tremble uncontrollably while reaching for a glass of water. How
should the nurse document this finding?

a. Transient ischemic attack.

b. Intention tremor.

c. Sensory dysfunction.

d. Muscle flaccidity.

39. A 60-year-old male client is admitted to the hospital with the complaint of
right knee pain for the past week. His right knee and calf are warm and
edematous. He has a history of diabetes and arthritis. Which neurological
assessment action should the nurse perform for this client?

a. Glasgow coma scale.

b. Pulses, paresthesia, paralysis distal to the right knee.

c. Pulses, paresthesia, paralysis proximal to the right knee.

d. Optic nerve using an ophthalmoscope.

40. The parents of a 4-week-old infant phone the pediatric clinic to report that
their infant eats well but vomits after each feeding. To differentiate
between normal regurgitation and pyloric stenosis, which information is
most important for the nurse to obtain?

a. Degree of forcefulness of vomiting episodes.

b. Level of infant’s distress after vomiting.

c. Position of the infant when vomiting occurs.

d. Odor and texture associated with emesis.

41. Proper nutrition is crucial for a client who is in acute renal failure. What is
the recommended diet for this client?
a. High protein, low carbohydrates, low sodium, low potassium.

b. Low protein, high carbohydrates, low sodium, low potassium.

c. Low protein, high carbohydrates, low sodium, high potassium.

d. High protein, low carbohydrates, low sodium, high potassium.

42. During a home visit, the nurse learns that a client is taking calcium
polycarbophil (FiberCon), an over-the-counter laxative. A single tablet
contains 500 mg and the maximum daily dose is 6 grams. What is the
maximum number of tablets the client should take in one day? (Enter the
numeric value only)

Answer: 12

43. A 9-year-old girl is diagnosed with nontropical sprue (celiac diasease), and
the nurse is evaluating teaching about a gluten-free diet. What action
indicates that the teaching is successful?

a. Family members state that they will read the brochures about the
diet.

b. The girl’s mother tells the nurse that the entire family will adhere to
the diet.

c. The entire family plans to attend a topic-related support group.

d. The girl chooses a rice cake instead of a whole-wheat toast for


breakfast.

44. What intervention is most important to include in the nursing care plan of a
client who is receiving chemotherapy and has a platelet count of
30,000/mm?

a. Place the client in reverse isolation.

b. Frequently assess the client’s blood pressure.


c. Observe for signs of dehydration.

d. Assess the client for abnormal bleeding.

45. When conducting a postpartum assessment, the nurse notes that the client
has positive Homan’s sign. Base on this finding, what action should the
nurse take?

a. Assess the client’s bladder and bowel functioning.

b. Tell the client to remain in bed and notify the healthcare provider.

c. Chart the finding and assess the client’s hemoglobin.

d. Assess the client’s temperature and respirations.

46. When is the best time for the nurse to assess the client for residual urine?

a. When the client’s bladder is distended.

b. Immediately after the client voids.

c. Just prior to the client voiding.

d. After draining the urinary catheter bag.

47. The nurse determines that the serum lithium level of a client who is
admitted in an acute manic episode is 0.9 mEq/L. what action should the
nurse implement?

a. Hold the next dose of lithium.

b. Give the next dose of lithium.

c. Observe the client for lithium toxicity.

d. Collect a specimen for serum sodium.

48. The hospital nurse-educator is planning prenatal education class for


adolescents in their third trimester of pregnancy. Which teaching technique
should the nurse use to meet the needs of this group of clients?
a. Practice bathing and diapering dolls that look and feel like newborn
infants.

b. Invite new adult mothers and fathers to help present course


materials.

c. Emphasize how labor preparation will affect the client rather than
the baby.

d. Limit attendance at the classes to the client and one other


designated person.

49. It would be a greatest benefit for the client with which problem related to
diabetes mellitus to change from the use of insulin syringes to using an
insulin pen for medication administration.

a. Diminished dexterity due to finger paresthesias.

b. Blindness secondary to diabetic retinopathy.

c. Lipodystrophy from continuous use of one injection site.

d. Hyperglycemia due to noncompliance with diet.

50. A client’s case is being reviewed by the hospital’s multi-disciplinary ethics


committee. What information could the nurse provide to the committee
regarding this case?

a. Descriptions of client behavior during the hospitalization that


indicate ineffective coping.

b. Information about the treatment alternatives that offer the great


chance of recovery.

c. Counsel on how to legally document the client’s wishes to have the


living will enacted.

d. Advice about handling a spiritual conflict a client may experience as a


result of an ethical crisis.
51. Which turning schedule demonstrates the best positioning routine for a
client on prescribed bedrest?

a. Alternating right and left lateral positions.

b. Right lateral to supine to left lateral positions.

c. Supine to Fowler’s to lateral positions.

d. Supine to Semi-Fowler’s positions.

52. The nurse manager of a pediatric unit needs to assign a room to a 6-month-
old diagnosed with respiratory syncytial virus (RSV). Which room
assignment is best for this child?

a. Single room with a sink inside the room.

b. Double room with a 3-month-old who has RSV.

c. Double room with a 6-month-old who has fifth disease.

d. Single room with negative air pressure.

53. When caring for a client who had a craniotomy yesterday for removal of a
pituitary tumor, which finding indicates to the nurse that further
information is needed?

a. Suture line is slightly reddened and swollen.

b. Glasgow coma scale (GCS) score is 14.

c. Urine output for 8 hours is 2,000 ml with a specific gravity of 1.001.

d. White blood cells (WBC) are 11000/mm and glucose is 138 mg/dl.

54. A 59-year-old female client who has diabetes is receiving 25 units of NPH-
100 insulin each morning. What assessment finding indicates that the
amount of insulin is inadequate to meet this client’s current needs?
a. A wound on the client’s ankle starts to drain and she complains of
pain.

b. Client complains of blurred vision and an inability to focus her eyes,


especially during the evening hours.

c. Client complains of being slightly nauseated in the morning, but was


able to eat her meal the night before.

d. The client’s serum glucose reading has been over 260 mg/dl for the
past two evenings.

55. In assessing a client four hours after a total knee replacement, the nurse
observes that the amount of drainage in the client’s auto transfusion
collection container has increased from 50 ml/hour to 200 ml/hour. What is
the priority nursing action?

a. Increase the amount of suction.

b. Assess the client’s level of pain.

c. Test the drainage for occult blood.

d. Notify the surgeon immediately.

56. A client with a C-7 spinal cord injury is experiencing autonomic dysreflexia.
The nurse should first assess the client for which precipitating factor?

a. A distended bladder.

b. A severe pounding headache.

c. Misalignment of the skeletal traction.

d. Profuse diaphoresis on the forehead.

57. The healthcare provider prescribed 3 liters of D5W to infuse in 24 hours.


The IV administration set delivers 10gtt/ml. The nurse should program the
infusion pump to deliver how many ml per hour? (Enter numeric value
only.)

Answer: 125

58. An adult male client, two days postoperative knee surgery, is diaphoretic
and experiencing visual hallucinations. He has been using a PCA pump with
morphine for pain control since surgery. On admission, he described a daily
intake of six cans of beer nightly. What is the priority nursing intervention?

a. Notify the healthcare provider immediately.

b. Discontinue the PCA pump.

c. Obtain the client’s vital signs.

d. Assess the amount of morphine used in the last 8 hours.

59. The nurse notes that a female client with a T-tube excreted a total of 300
ml of greenish-brown drainage in the sixteen ours since her
cholecystectomy. What nursing action has the highest priority at this time?

a. Document findings in the client record.

b. Notify the healthcare provider of the findings.

c. Place the client in Fowler’s position.

d. Begin clamping the tube for one hour during meals.

60. A 3-month-old with developmental dysplasia of the hip (DDH) is being


discharged with a Pavlik harness. Which information should the nurse
provide the parents about the use of harness?

a. The harness is worn continuously until the hip is clinically and


radiographically stable, about 3 to 5 months.

b. The harness should be worn for 6months during the day and at night
while the child is doubled-diapered.
c. The harness is worn for 4—to 6 weeks and then a hip spica cast is
applied for the remainder of the treatment.

d. To avoid interfering with normal movement, the harness is worn for


one year while the child is sleeping.

61. Which information should the nurse offer family members to support the
administration of vitamin K to a newborn infant?

a. State law requires that all newborns receive an injection of Vitamin


K.

b. Healthcare providers routinely recommend and prescribe Vitamin K


for newborns.

c. Vitamin K is administered to stimulate the production of clotting


factors.

d. Newborns do not ingest adequate amounts of fat soluble vitamins


from feedings.

62. A client two days postoperative after receiving a coronary artery bypass
graft is suspected of having a pulmonary embolus (PE). Which assessment
finding should the nurse recognize as characteristic of PE?

a. Heart rate of 52 beats/minute

b. Respiratory rate of 34 breaths/minute

c. Blood pressure of 170/90

d. Urine output of 200 ml every hour.

63. While assessing a client’s blood pressure using an aneroid


sphygmomanometer, the nurse inflates the cuff to an initial reading of 160
mm calibration. Upon release of the air valve, the nurse immediately hears
loud Korotkoff sounds. What action should the nurse implement next?
a. Continue the blood pressure assessment until the last Korotkoff
sound is heard.

b. Release the air and reinflate the cuff to 30 mm Hg above the client’s
previous systolic reading.

c. Reposition the stethoscope in the antecubital fossae over the


palpable branchial pulse point.

d. Inflate the cuff quickly to a higher mm of Hg reading than the


previously auscultated systolic sound.

64. A male client tells the home health nurse that he has stated taking
magnesium hydroxide and aluminum hydroxide (Malox) to treat occasional
heartburn. It is most important for the nurse to review the client’s medical
history regarding the presence of which disorder?

a. Renal disease.

b. Diabetes mellitus.

c. Chronic bronchitis.

d. Deep vein thrombosis.

65. The nurse assigns an unlicensed assistive personnel (UAP) to take the vital
signs of the client whi is positive for Human Immunodeficiency Virus (HIV).
What protective apparel should the nurse counsel the UAP to wear when
carrying out this assignment?

a. None.

b. Gown, gloves, mask.

c. Gloves only.

d. Gloves and mask.


66. A 4-year-old is admitted to intensive care following heart surgery for
Tertrology of Fallot and the nurse is planning immediate postoperative
care. Which intervention should the nurse include in this child’s plan of
care?

a. Elevate the head of the bed to 30 degrees.

b. Continued use of hypothermia blanket.

c. Increase fluids to maintain the blood pressure.

d. Turn, cough, and deep breathe every shift.

67. The nurse administers acetylcysteine (Mucomyst) to a client with thick


mucous and a nonproductive cough. To evaluate the effectiveness of the
medication, what question should the nurse ask the client?

a. “How much phlegm are you coughing up?”

b. “Are you still coughing?”

c. “What color is the mucous?”

d. “Are you experiencing any pain when you cough?”

68. The nurse is preparing to administer 1,000 ml of dextrose 25% total


parental nutrition (TPN) to a client with ulcerative colitis. Which
intervention is most important for the nurse to implement?

a. Administer the TPN through a central line.

b. Review the client’s intake and output.

c. Evaluate the client’s nutritional history.

d. Assess vital signs prior to administration.

69. A high-school girl asks the school nurse what to do about her fingernails
that look “so awful” since she had her artificial nails removed 6 weeks ago.
On inspection, the nurse finds the girl’s nails are thickened, cracked, and
yellowing. What instruction should the nurse provide?

a. Do not use manicure products that dry the nails.

b. Use a prescribed systemic antifungal medication.

c. Keep nails short and trimmed straight across.

d. Avoid harsh chemicals and abrasives on the nails.

70. The nurse is preparing to administer an injection for a 5-year-old boy, and
he asks the nurse if its going to hurt. What response is best for the nurse to
provide?

a. Reassure the child that the injection will not hurt.

b. Instruct the child to look the other way during the injection.

c. Tell the child that the injection will hurt some.

d. Instruct the mother to hold the child securely.

71. When assessing a male client who is receiving a unit of packed red blood
cells (PRBCs), the nurse notes that the infusion was started 30 minutes ago,
and 50 ml of blood is left to be infused. The client’s vital signs are within
normal limits. He reports feeling “out of breath” but denies any other
complains. What action should the nurse take at this time?

a. Administer a PRN prescription for diphenhydramine (Benadryl).

b. Start the normal saline attached to the Y-tubing at the same rate.

c. Decrease the intravenous flow rate of the PRBC transfusion.

d. Ask the respiratory therapist to administer PRN albuterol (Ventolin).

72. The nurse is evaluating a client’s central venous pressure (CVP) readings
that are trending upward. What nursing diagnosis is supported by the
pathophysiologic mechanism most likely causing the increases?
a. Fluid volume deficit.

b. Inadequate airway clearance.

c. Impaired cardiac output.

d. Ineffective tissue perfusion.

73. The unlicensed assistive personnel (UAP) helping the nurse with the care of
a client with a Clostridium difficile infection has obtained a box of
disposable gowns and placed them in the client’s room. What action should
the nurse implement?

a. Determine why the UAP felt that gowns were needed for the client.

b. Remind the UAP to obtain face masks and goggles as well as gowns.

c. Inform the UAP that gowns will not be needed in caring for this
client.

d. Advise the UAP to place the box of gowns outside the client’s room.

74. A client comes to the prenatal clinic on April 20th and her estimates date of
birth (EDB) is July 15th. Identify the location where the nurse expects to
locate the fundus. (Click the chosen location. To change, click on the new
location.)

Answer: See Picture

75. Which statement would the nurse consider characteristic of a client


diagnosed with diabetes insipidus (DI)?

a. “I am always thirsty and crave cold water.”

b. “I stay hungry all of the time.”

c. “I have gained a lot of weight.”

d. “I urinate about 4 times a day, and it is straw-colored.”


76. Which discharge instruction has the greatest priority for the mother of a 6-
year-old who was diagnosed with type 1 diabetes mellitus one week ago?

a. “Measure blood sugar at least four times daily.”

b. “Always carry a quick source of sugar.”

c. “Eat a well-balanced diet.”

d. “Draw regular insulin into a syringe before NPH insulin.”

77. To assess for the presence of lower extremity paresthesia, what action
should the nurse take?

a. Apply touch and pain stimuli to the client’s legs.

b. Tap the client’s knee tendons with a reflex hammer.

c. Palpate the client’s call muscles while at rest.

d. Observe the client’s gait while ambulating.

78. A client in the first trimester of pregnancy calls the prenatal clinic to report
she is nauseated and her stools are black and thick since she started taking
iron supplements last week. How should the nurse respond? (Select all that
apply)

a. Come to the clinic today.

b. Take the iron with breakfast.

c. Increase the consumption of milk while taking iron.

d. Drink full glass of tea with each iron tablet.

e. Changes in color and consistency of stool are normal.

f. Take the iron supplement at bedtime.

79. An infant admitted to the neonatal intensive care unit is tachypneic,


tachycardiac, and has bounding brachial pulses. The healthcare provider
suspects that the infant has coarctation of the aorta. Which intervention is
most important for the nurse to include in the infant’s plan of care?

a. Correct respiratory alkalosis related to tachypnea.

b. Monitor for congestive heart failure.

c. Assess for centralized cyanosis 4 times daily.

d. Auscultate for a diastolic murmur daily.

80. A client with which problem requires the most immediate intervention by
the nurse.

a. Finger paresthesias related to carpal tunnel syndrome.

b. Increasing sharp pain related to compartment syndrome.

c. Increasing burning pain related to a Morton’s neuroma.

d. Increasing sharp pain related to plantar fasciitis.

81. A client at 40-weeks gestation is admitted to labor and delivery. Her


obstetrical history includes 3 live births at 39, 38, and 35 weeks gestation, 2
miscarriages at 6 and 8 weeks gestation, and a fetal demise at 33-weeks
gestation. Which is an accurate summary of this client’s obstetrical history?

a. Gravida 7 Term 1 Preterm 3 Abortion 2 Living 4.

b. Gravida 6 Term 3 Preterm 1 Abortion 2 Living 2.

c. Gravida 6 Term 2 Preterm 2 Abortion 2 Living 4.

d. Gravida 7 Term 2 Preterm 2 Abortion 2 Living 3.

82. A father watching the admission of his newborn to the nursery notices that
eye ointment is placed in the infants eyes. He asks the nurse what the
purpose of this ointment is. The nurse would be correct in stating that the
purpose for using the ointment is to
a. prevent eye infections.

b. dilate the pupil so the red reflex can be visualized.

c. clear the infant’s vision.

d. prevent herpes infection.

83. Which client situation requires the most immediate intervention by the
nurse?

a. A four centimeter area of dehiscence is observed on a client’s


abdominal incision one day after surgery.

b. A stage IV pressure ulcer has a five centimeter area of necrosis


surrounded by pale pink tissue.

c. A stage II pressure ulcer located on a client’s sacrum is draining a


moderate amount of purulent drainage.

d. A six centimeter area of reactive hyperemia is observed over the left


trochanter of a bedfast client.

84. A client is having trouble breathing while lying in a dorsal recumbent


position. What action should the nurse implement first?

a. Elevate the head of the bed.

b. Obtain a pulse oximeter.

c. Document the presence of orthopnea.

d. Assess the client’s vital signs.

85. When assessing the oral temperature of an adult client at 6:00 p.m., the
nurse notes that the client’s temperature at 6:00 a.m. was 97.2 F and now
is 98.8 F. What intervention should the nurse implement?

a. Administer a PRN dose of medication to reduce the fever.


b. Document the intermittent fever in the nurse’s notes.

C. Notify the healthcare provider of the increase in temperature.

d. Document this temperature variation on the graphic sheet.

86. The nurse is planning to provide mouth care for an unconscious client.
Which statement is accurate in regard to implementing mouth care for this
client?

a. Brushing an unconscious client’s teeth should be avoided because of


his/her inability to cooperate with the procedure.

b. Unconscious clients need less frequent mouth care than conscious


clients because they are not eating.

c. Positioning the unconscious client flat with the head turned to the
side is the key to providing safety during mouth care.

d. Cleaning the inner cheeks and outer gum surfaces with glycerin
swabs is the best method of providing mouth care fir an unconscious
client.

87. In monitoring a client receiving antibiotics for acute glomerulonephritis,


which laboratory test result requires the most immediate intervention by
the nurse?

a. Serum potassium of 6 mEq/L.

b. Serum sodium of 150 mEq/L.

c. Presence of protein in the urine.

d. White blood cell count of 14,000 mm.

88. A client is in the emergency center following a thoracic injury. Assessment


data include: a 50 palpable systolic blood pressure, an increase in the right
arterial pressure (RAP) from 6 to 25, distended neck veins, and very distant
heart sound. What intervention should the nurse anticipate based on these
findings?

a. Pericardiocentesis.

b. Endotracheal intubation.

c. Emergency tracheostomy.

d. Chest tube insertion.

89. The nurse assesses that a client has nailbed clubbing. What additional
information is consistent with this finding?cianosis

a. Oxygen saturation of 85%.

b. +3 peripheral dependent edema.

c. Capillary refill <3 seconds.

D. Absent deep tendon reflexes.

90. What finding would warrant further investigation to determine if a client is


septic shock was experiencing the serious complication of disseminated
intravascular coagulation (DIC)?

a. There is a marked increase in atelectasis visible on x-ray.

b. The jugular catheter is oozing blood at the insertion site.

c. The urine is frothy and is dark amber in color.

d. The mucous membranes are pink but appear dry and cracked.

91. While conducting an interview to obtain a health history, the nurse notices
that the client pauses frequently and looks at the nurse expectantly. Which
response is best for the nurse to provide?

a. Sit quietly to allow the client to respond comfortably.

b. Reassure the client that there are no wrong answers.


c. Tell the client to return later for another interview.

d. Continue to ask questions until the client responds.

92. During an evening shift on a medical unit, the only nurse on the unit is busy
with an unstable client. The unit clerk, who is also both a certified
medication aide and an unlicensed assistive to prescribe a PRN dose of an
oral over the counter laxative for a client who is…….

a. Tell the healthcare provider that RN will return the phone call as
soon as possible.

b. Remain with this client and monitor the vital sings while the RN takes
the call.

c. Ask the healthcare provider to remain on “hold” until the RN can


confirm the prescription.

d. Be sure to write down what is prescribed and then repeat it back to


the healthcare provider.

93. In assigning care on a telemetry unit, it is most important for the charge
nurse to assign which client to an RN rather than an LPN?

a. A middle-aged client with hyperlipidemia who just received a new


prescription for lovastatin (Mevacor).

b. An older adult with dyspnea and edema due to heart failure who is
receiving nesiritide (Natrecor).

c. A 62-year old with coronary artery disease who is receiving


clopidogrel (Plavix) for transient ischemic attacks (TIA).

d. A 55-year-old with orthostatic hypertension who is receiving low-


molecular-weight heparin (Lovenox).

94. Following two defibrillation shocks, the client’s ECG continues to indicate
ventricular fibrillation (VF). Which intervention should the nurse implement
next?
a. Perform the third defibrillation shock.

b. Obtain an arterial blood gas sample.

c. Administer an IV bolus of epinephrine.

d. Resume CPR immediately.

95. The nurse is triaging a 50 year-old male client in the emergency


department. He is complaining of sever mid-chest pain of sudden onset.
What other information should the nurse obtain during the initial
assessment? (Select all that apply.)

a. Has he previously experienced a similar type of pain?

b. On a 10-point scale, how does he rate the pain he is currently


experiencing?

c. Does anything make his pain worse?

d. Does he think he is having a heart attack?

e. Does the pain radiate to any particular area of the body?

96. Despite repeated instructions, an 80-year-old client with Parkinson’s


disease is unable to instill ophthalmic medication without assistance
because of hand tremors. What action is best for the nurse to take?

a. Obtain a prescription for a visiting nurse to instill the medication


twice a day.

b. Continue to reinforce the instructions to enhance the client’s self-


confidence.

c. Determine if a family member is available and willing to instill the


medication.

d. Document the client’s inability to instill the medication without


assistance.
97. After assessing a client, the nurse identifies three nursing diagnoses. When
developing the client’s plan of care, which action should the nurse take
next?

a. Collaborate with client to establish goals.

b. Cluster supportive client data.

c. Identify client care interventions.

d. Prioritize the identified nursing diagnoses.

98. What instruction should the nurse include in the teaching plan for the
family of a school-aged child with AIDS?

a. Avoid exposure to chickenpox.

b. Keep the child away from other children and begin a home school
program.

c. Obtain an injection of penicillin G 1000 units weekly.

d. Obtain a booster for all immunization as soon as possible.

99. A male client admitted the morning of same day surgery states he drank a
glass of water during the night. What intervention should the nurse
implement first?

a. Reassure the client that a small amount of water is not harmful.

b. Determine the amount of water and exact time it was taken.

c. Assess the client for active bowel sounds and ability to urinate.

d. Notify the healthcare provider of the client’s fluid intake.

100. The nurse is assessing a client on a ventilator. The endotracheal tube (ET)
lip line measurement is 24 cm. Four hours ago the lip line measurement
was 20 cm. Which intervention should the nurse implement first?
a. Reposition the ET back to the 20 cm mark.

b. Assess the client’s bilateral breath sounds.

c. Request a STAT portable chest x-ray.

d. Monitor the client’s arterial blood gases.

101. A client is ambulating with a two-wheeled walker by rolling the walker


forward and then moving each foot forward. The nurse notes that the
client’s elbows are slightly flexed when grasping the hand bar. After the
client returns to the chair, what action should the nurse implement?

a. Explain the need to remove the wheels from the walker.

b. Offer to adjust the height of the walker.

c. Demonstrate more coordinated movement of the legs and walker.

d. Encourage the client to continue using the walker as observed.

102. A postoperative client is receiving meperidine hydrochloride (Demerol) 60


mg and hydroxyzine (Vistaril) 35 mg IM q3h PRN pain. Demerol available in
a prefilled syringe labeled 100 mg/ml, and the vial of Vistaril is labeled 50
mg/ml. What is the total volume, in ml, that the nurse should administer?
(Enter the numeric value only)

Answer: 1.3

103. A male client with an HIV infection is placed on a “drug cocktail” consisting
of three antiretroviral agents and a protease inhibitor. He asks the nurse
why he must take so many drugs at once. Which response by the nurse best
addresses this client’s question?

a. “The drug will interact with each other and shorten your overall drug
therapy.”

b. “A multi-drug approach decreases the risks for developing drug


resistance.”
c. “The drug combination decreases the side effects of the
medications.”

d. “Taking several drugs at once decreases your viral load more


quickly.”

104. The nurse notices that the catheter of a client who had a transurethral
resection of the prostate (TURP) 2 days ago is not draining and his bladder
is distended. What action should the nurse take initially?

a. Irrigate the catheter.

b. Discontinue the catheter.

c. Milk the catheter tubing.

d. Change the catheter.

105. What is the priority nursing diagnosis when caring for a client with a
Jackson-Pratt drain and a surgical wound that is healing by secondary
intention?

a. Risk for impaired skin integrity related to wound drainage.

b. Body image disturbance related to draining wound.

c. Risk for infection related to open wound.

d. Knowledge deficit related to inadequate information about drains.

106. A 17-year-old male who was arrested last month for gang-related activities
has a court order to attend weekly group therapy sessions at the mental
health clinic. Today his mother calls the clinic nurse to report that her son
became angry last night and put his fist through a window. Which
intervention is most import for the nurse to implement?

a. Reinforce the need for the adolescent to attend group therapy


sessions.
b. Refer the mother for psychiatric evaluation for anxiety and
depression.

c. Tell the mother to describe her feelings of helplessness to her son.

d. Advise the mother to call the police if violent behavior occurs again.

107. An 8-year-old girl is brought to the clinic by her mother who reports that
her daughter has had a severe sore throat for the last three days and
suddenly began drooling. The child’s tympanic temperature is 103 F and
she is struggling to breathe. What initial action should the nurse take?

a. Notify the healthcare provider immediately.

b. Ask the child to cough several times.

c. Use a tongue blade to inspect the throat.

d. Review immunization records for influenza vaccine.

108. respuesta de un cuadro con la historia del pt (change)

109. A newly-delivered infant with which condition should be assessed by the


nurse first?

a. Caput succedaneum.

b. A fetal scalp pH of 7.05 before birth.(asidosos )

c. Born to a mother who was in labor for 14 hours.

d. Delivered by caesarean section for breech presentation.

110. The nurse-manager observes that the staff nurse has used wrist restraints
to help secure an elderly female in her wheelchair. The client is pleading for
the nurse to release her arms. The nurse explains to the nurse manager
that the client needs to be restrained in the wheelchair so that the nurse
can ………….
a. Advise the staff nurse to remove the restraints from the client’s
wrists.

b. Determine if the client has a PRN prescription for an anxiety agent.

c. Contact the healthcare provider to ensure that a prescription for


restraints was written.

d. Close the door to the room to avoid disturbing other clients in nearby
rooms.

111. While preparing a client with suspected appendicitis for an abdominal


ultrasound, the nurse notes that the client is experiencing localized
rebound tenderness in the right lower quadrant of the abdomen. Based on
this information, what is the most important nursing intervention?

a. Continue to prepare the client for the ultrasound.

b. Hold the ultrasound until the healthcare provider is notified.

c. Obtain a chest x-ray in preparation for surgery.

d. Administer the oral analgesic prescribed for PRN use.

112. The American Diabetes Association recommends diabetic screening every


three years for those over the age of 45. This is an example of which type of
public service?

a. Primary prevention.

b. Secondary prevention.

c. Initial screening.

d. Tertiary prevention.

113. The nurse is monitoring a 6-month-old infant with a closed head injury.
Which assessment finding is the earliest indication of neurological
deterioration?
a. Sluggish, unequal pupillary response.

b. Projectile vomiting after eating.

c. Irritable and unable to rest.

d. Decorticate posturing.

114. While the nurse is inserting a nasogastric tube, the client becomes cyanotic.
What intervention should the nurse implement?

a. Notify the healthcare provider.

b. Initiate oxygen per face mask.

c. Slowly continue to insert the tube.

d. Withdraw the nasogastric tube.

115. In reviewing the medical record, the nurse notes that a client’s last eye
examination revealed an intraocular pressure (IOP) of 28 mmHg. What
information should the nurse ask the client?(10-21)

a. Length of time the client has been wearing prescription lenses.

b. Recent experience of seeing light flashes or floaters.

c. Complaints of any blind spots in the client’s field of vision.

d. Use of prescribed eyes drops since last exam by ophthalmologist.

116. While the nurse is bathing a bedfast client with generalized weakness, the
client develops labored respirations and an audible pharyngeal rattles. The
nurse auscultates coarse rattles in the upper lung fields. What action should
the nurse implement first?

a. Performing oropharyngeal suctioning.

b. Encourage increased fluid intake.

c. Assess the client’s temperature.


d. Complete the client’s personal care.

117. An older man with a history of multiple falls at home tells the clinic nurse
that his son, who was incarcerated last year for an assault and battery
conviction, has become increasingly abusive since his release from prison
six week ago. What intervention is most important for the nurse to
implement?

a. Discuss becoming a survivor rather than a victim of abuse.

b. Refer to a program for victims of domestic violence.

c. Examine the client for evidence of physical abuse.

d. Assist the client in developing an emergency safety plan.

118. A female client presents to the emergency department in the early evening
complaining of abdominal cramping, watery diarrhea, and vomiting. She
tells the nurse that she was at a picnic and ate barbeque that afternoon.
What question is most important for the triage nurse to ask this client?

a. “Is anyone else sick who was also at the picnic?”

b. “How high was your temperature when you returned home?”

c. “Have you taken any medication to treat this problem?”

d. “Have you recently traveled outside the United States?”

119. A client has severe bradycardia following the administration of metoprolol


(Toprol XL). What medication should the nurse anticipate administering?

a. Naloxone (Narcan).

b. Digoxin (Lanoxin).

c. Diltiazem (Cardizem)

d. Atropine sulfate
120. The first day postoperative, a client’s vital signs are: temperature 99 F oraly,
respiration 29 breaths/minute, blood pressure 120/74 mm Hg, heart rate
88 beats/minute. Based on these findings, what nursing action should the
nurse implement first?

a. Auscultate the lung sound.

b. Retake temperature rectally.

c. Administer an antipyretic.

d. Confirm heart rate with an ECG.

121. The nurse is assessing a client with hypothyroidism and knows that these
clients are at risk for myxedema coma. What symptoms indicate that the
client is developing this condition?

a. Weight loss, sinus tachycardia, and exophtalmus.

b. Chest pain, dyspnea, and temperature above 37.7 C.

c. Hypothermia, decreased cardiac output, and decreased respiratory


functioning.

d. Hair loss, brittle nails, numbness and tingling of the fingers.

122. A client received a stent following angioplasty and is being monitored


postprocedure in a cardiac unit. Which discharge prescription, written by
the cardiology resident, should the nurse question?

a. Schedule MRI of the head to visualize carotids within one week after
angiography.

b. Take clopidogrel (Plavix) 75 mg and aspirin 325 mg PO daily for the


next six months.

c. Do not lift heavy objects or vigorously exercise for two weeks


following the procedure.
d. Carry a patient identification card at all times indicating that a stent
was placed.

123. A school-age child with asthma is intubated and placed on a mechanical


ventilator. The parents of the child are pale, holding onto each other, and
have tears in their eyes. What statement by the nurse is most therapeutic
when first interacting with these parents?

a. “The ventilator is making sure you child is getting enough oxygen.”

b. “Your child is resting comfortably at the present time.”

c. “It must be difficult for you to see your child go through this.”

d. “Your child is in good hands. Everything is going to be okay.”

124. A male client who is diagnosed with schizophrenia and takes clozapine
(Clozaril), tells the nurse that he does not understand the reason why he
must have his blood drawn. What is the most important reason for drawing
blood levels? Because Clozaril

a. can cause agranulocytosis, so the white blood cells count need to be


monitored.

b. can cause hyperglycemia, so serum glucose levels should be


evaluated periodically.

c. drug blood levels need to be assessed to monitor for toxicity.

d. can cause live damage, so liver function tests are necessary.

125. A 27-year-old gravida 2, at 40-weeks gestation, is experiencing firm


contractions occurring 4 to 5 minutes apart. Three hours after initiation of
labor, she begins moaning and crying, is restless, belches, and states that
she needs to have a bowel movement. Which intervention should the nurse
implement?

a. Assist the client onto a bedpan.


b. Assess the client’s cervical dilation.

c. Have the client turn to her left side.

d. Tell the client to bear down with the next contraction.

126. A gravida 3 para 3 who is Rh-negative delivers a full-term infant at home


with the assistance of a nurse-midwife. Two days later, the client calls the
clinic to ask if it is necessary to see the healthcare provider since the infant
is healthy, and she is not having any complications. The woman’s history
indicates that both previously born infants were Rh-negative. Which
response should the nurse provide?

a. The newborn’s blood type should be teste to determine the need for
RhoGAM.

b. It is likely that the husband is Rh negative, and if so RhoGAM is not


needed.

c. RhoGAM injections must be administered within 24 hours after


delivery.

d. RhoGAMis not indicated since both previous babies were Rh-


negative.

127. The healthcare provider prescribes the antibiotic erythromycin (Erythrocin)


for a client with pneumonia. Before administering the medication which
intervention should the nurse implement?

a. Make sure a sputum culture has been obtained.

b. Assess the serum electrolyte levels.

c. Obtain a sterile urine specimen.

d. Take the client’s apical pulse.


128. A client is admitted to the emergency center with a flail chest after an
automobile accident. Which set of arterial blood gases should the nurse
report to the healthcare provider immediately?

a. PaO2 90 mm Hg, SaO2 90%, pH 7.45, PaCO 42 mm Hg, HCO 23.

b. PaO2 80 mm Hg, SaO2 95%, pH 7.30, PaCO 55 mm Hg, HCO 25.

c. PaO2 84 mm Hg, SaO2 93%, pH 7.35, PaCO 35 mm Hg, HCO 20.

d. PaO2 85 mm Hg, SaO2 90%, pH 7.49, PaCO 40 mm Hg, HCO 28.

129. Pursed-lip breathing is a controlled breathing technique helpful to many


clients with emphysema. What instruction given by the nurse is the most
accurate description of the pursed-lip breathing technique for clients with
emphysema?

a. “Inhale through the nose and exhale through pursed lips.”

b. “Inhale through the nose, exhale through the nose, purse the lips and
hold your breath for 30 seconds.”

c. “Hold your breath for 10 to 15 seconds, purse the lips and inhale
through your mouth, then exhale through pursed lips.”

d. “Purse the lips, inhale through the mouth and exhale through the
mouth.”

130. An older client with chronic liver failure and metastatic cancer is admitted
with bilateral subdural hematomas. The healthcare provider discontinues
the client’s dialysis treatments, stating that death is inevitable, but the
client is unconscious, and there is no DNR directive. What is the priority
nursing action?

a. Notify the healthcare provider that the DNR prescription cannot be


implemented.

b. Determine if a family member has the client’s legal power of


attorney.
c. Ask the nurse-manager to request as ethics committee decision.

d. Encourage the family to request hospice care for their relative.

131. A female college student presents to the health center complaining of


anxiety related to her fear that she has contracted genital herpes. The coed
tells the nurse that she has vaginal irritation and is embarrassed by the
possibility of having acquired a sexually transmitted disease. What response
is best for the nurse to provide?

a. “We need to conduct some tests to validate that you have herpes.”

b. “Are you more worried about what others think that your own well-
being?”

c. “What makes you think that you might have herpes?”

d. “Illness like herpes can be embarrassing, but ther are also serious.”

132. What adverse effect(s) of chemotherapy place the client with cancer at
highest risk for sepsis-induced distributive shock?

a. Bone marrow depression.

b. Oral candidiasis.

c. Nausea and diarrhea.

d. Fatigue and weakness.

133. When obtaining subjective data from a client, what intervention should the
nurse implement first?

a. Clarify inferences.

b. Listen attentively.

c. Establish rapport.

d. List problems.
134. A client with reflux esophagitis reports relief of symptoms. The nurse
instructs the client that symptom relief is most likely the result of which of
the client’s prescriptions?

a. Clarithromycin (Biaxin), an antibiotic.

b. Celecoxib (Celebrex), a COX-2 inhibitor.

c. Promethazine hydrochloride (Phenergan), an antiemetic.

d. Lansoprazole (Prevecid), a proton pump inhibitor.

135. The healthcare provider prescribes Cyanocabalmin Injection, USP 100 mcg
IM every 3 days for a client with pernicious anemia. The vial is labeled, “1
mg/ml.” How many ml should the nurse administer? (Enter numeric value
only. If rounding is required, round to the nearest tenth.)

Answer: 0.1

136. A client is admitted with a diagnosis of right lower lobe pneumonia. Which
breath sound is the nurse most likely to auscultate over the right lower
lobe?

a. Coarse crackles.

b. Audible rhonchi.

c. Wheezing.

d. Friction rub.

137. A 48-year-old female client who has been treated for metastasized breast
cancer for the past year is told by her healthcare provider that
chemotherapy is not producing the desired remission. The next morning
client is crying and asks the nurse, “Who will care for my children?” Which
response is best for the nurse to provide?

a. “Your husband will have to be there for your children.”


b. “Have you talked to your family about who will be responsible for
your children?”

c. “What would you like to see happened with your children?”

d. “Try to think about getting well. Someone will care for your children.”

138. A male client with a fungal infection of the toenail reports to the nurse that
he has been applying an over-the-counter triple antibiotic ointment to the
infection daily for two weeks without any improvement. What action
should the nurse take?

a. Tell the client that treatment of fungus-infected toenails often takes


several

b. Instruct the client to obtain a prescription for oral terbinafine


(Lamisil).

c. Advise the client to obtain a prescription-strength formulation of the


ointment.

d. Suggest that the client use the ointment twice a day to be more
effective.

139. A male client who had a lumbar laminectomy has a prescription to be


turned every two hours. One hour after being positioned on the right side,
the client reports he is uncomfortable and requests to be moved. Which
action should the nurse take?

a. Inform the client that he will be turned in one hour.

b. Instruct the client to flex his toes to help his discomfort.

c. Logroll the client to the left side with two personnel.

d. Offer a PRN analgesic for the client’s discomfort.


140. A client is discussing feelings related to a recent loss with the nurse. The
nurse remains silent when the client says, “I don’t know how I will go on.”
What is the most likely reason for the nurse’s behavior?

a. The nurse is stating disapproval of the statement.

b. The nurse is respecting the client’s loss.

c. Silence is reflecting the client’s sadness.

d. Silence allows the client to reflect on what was said.

141. The nurse is planning to flush a heparin-loc on a central venous catheter


with a total of 300 units of heparin flush solution. Which concentration of
solution should the nurse select?

a. 100 units heparin/ml.

b. 1000 units heparin/ml.

c. 0.1 units heparin/ml.

d. 10 units heparin/ml.

142. The nurse finds a female client crying quietly in her room. What action
should the nurse take first?

a. Ask the client why she is crying.

b. Provide the client privacy and quietly close the door.

c. Review the client’s record before attempting to intervene.

d. Pull up a chair and sit beside the client.

143. A client is taking a comolyn sodium (Intal) inhaler for chronic asthma.
Which statement indicates the client understands the medication teaching?

a. “I will not discontinue taking this medication abruptly.”

b. “I will have my liver enzymes checked monthly.”


c. “I should keep my inhaler with me at all times.”

d. “It is important to take this medication with food.”

144. What information in a client’s history indicates the highest risk factor for
hepatitis C?

a. Monogamous sexual activity.

b. Intravenous drug abuse.

c. Eating contaminated shellfish.

d. Recent travel to an underdeveloped country.

145. When providing diet teaching for a client who is experiencing


thrombocytopenia as the result of chemotherapy, which instruction is most
important for the nurse to include?

a. Avoid hard foods.

b. Floss teeth after meals.

c. Warm all foods.

d. Chew food completely.

146. An elderly client seems confused and reports the onset of nausea, dysuria,
and urgency with incontinence. Which action should the nurse implement?

a. Auscultate for renal bruits.

b. Obtain a clean catch mid-stream specimen.

c. Use a dipstick to measure for urinary ketones.

d. Begin to strain the client’s urine.

147. A male client receiving fentanyl (Duragesic) via transdermal patch reports
to the nurse that he is experiencing abdominal discomfort. The nurse’s
assessment indicates abdominal distention with decreased bowel sounds.
Which intervention should the nurse initiate?

a. Remove the transdermal patch.

b. Administer a prescribed PRN laxative.

c. Perform a digital exam for impaction.

d. Notify the healthcare provider of the symptoms.

148. The father of an 11-year-old boy tells the nurse that he feels unsure about
talking to his son about nocturnal emissions. How should the nurse address
this issue with this parent?

a. Inform the father that it is most important to let the son know that
nocturnal emissions are normal.

b. Reassure the father that he does not need to have this discussion
with his son unless his son asks about nocturnal emissions.

c. Refer the father and son for counseling with a therapist that
specializes in sexual dysfunction.

d. Tell the father to begin discussion of this issue if his son seems
embarrassed by the occurrence of nocturnal emissions.

149. Which gastrointestinal findings should the nurse be concerned about in a


client at 28 weeks gestation?

a. Pica.

b. Pyrosis.

c. Ptyalism.

d. Decreased peristalsis.

150. A client was admitted with an exacerbation of congestive heart failure


secondary to COPD. Which observation(s) by the nurse require immediate
intervention to reduce the likelihood of harm to this client? (Select all that
apply)

a. A peripheral IV is saline-locked.

b. The client is resting supine in bed.

c. Oxygen is flowing at 5 L/min. via mask.

d. The call light cord is wrapped on the siderail.

e. Two glasses of water are on the bedside table.

f. A bedside commode is located near the bed.

151. The pediatric unit is extremely busy when the admission office notifies the
charge nurse that a child who has acute lymphocytic leukemia (ALL) needs
to be admitted to the unit. The parents brought prescriptions from the
healthcare provider for their child to have a chest x-ray and blood work
drawn on admission. What action should the charge nurse take?

a. Ask the family to wait in the visiting area until a nurse can assist them
with the admission process.

b. Tell the admission clerk to bring the child immediately to the unit and
places the child in a private room.

c. Ask the radiology department to complete the chest x-ray before


admitting the child to the unit.

d. Have hospital transportation take the child to the central laboratory


for blood work before admission.

152. A female student nurse is paired with a staff nurse as a learning experience.
The student nurse has an offensive body odor, her uniform is wrinkled, and
she is acting sluggish and tired. What action should the staff nurse
implement?
a. Inform the charge nurse that the student is unprepared for the
clinical area.

b. Tell the student to refresh up before allowing her to see clients.

c. Ask the student to wait in the lounge until her faculty member is
contacted.

d. Complete an incident report and refer the student for counseling.

153. Following a traumatic delivery, an infant receives an initial Apgar score of 3.


What intervention is most important for the nurse to implement?

a. Continue resuscitative efforts.

b. Inform the parents of the infant’s condition.

c. Repeat the Apgar assessment in 5 minutes.

d. Page the pediatrician STAT.

154. The nurse is caring for a 10-year-old who is diagnosed with acute
glomerulonephritis. Which outcome is the priority for this child?

a. Adequate nutritional status as evidenced by no weight gain or loss.

b. Fluid balance maintained as evidenced by a urine output of 1 to 2


ml/kg/hr.

c. No signs of skin breakdown as evidenced by intact skin and no


redness.

d. Activity tolerance as evidenced by performing appropriate age-level


activities.

155. A female client who is scheduled to be discharge within several hours


shows the nurse a knife on her tray and states that she could use it to cut
herself. What intervention is most important for the nurse to implement?

a. Inform the healthcare provider who discharged the client.


b. Call dietary and ask them to send plastic utensils.

c. Include guidelines for coping in the discharge plan.

d. Inform family of the client’s statement.

156. An older client is transferred to the rehabilitation unit with the diagnosis of
cerebrovascular accident with left side hemiplegia. The nurse addresses the
client from the right side, and the client points to the left leg and states,
“There is someone’s leg in my bed!” What is the best response by the
nurse?

a. “Your stroke has impaired your ability to recognize that it is your


leg.”

b. “Can you tell me your name? Do you know where you are?”

c. “Push against my hands at the same time using both of your feet.”

d. “Where are your glasses? Can you see clearly?”

157. A client returns to the acute care unit following surgery with 0.9% normal
saline infusing at 45 drops/minute through tubing with a drop factor of 60
drops per ml. The postoperative prescription include 0.9% normal saline at
75 ml/hour to alternate with Lactated Ringer’s solution at 75 ml/hour. An
intravenous infusion pump is not available. What action should the nurse
implement?

a. Change the normal saline to keep open rate until an infusion pump is
available.

b. Increase the rate of the present normal saline infusion to 75 drops


per minute.

c. Switch the saline to lactated ringer’s solution infusing at 75 drops per


minute.

d. Leave the normal saline at the current rate until an infusion pump is
available.
158. When conducting a physical examination, the nurse is assessing a client’s
abdomen and identifies a centrally localized distention that is pulsating.
This finding should direct the nurse to consider what pathology?

a. Aneurysm.

b. Appendicitis.

c. Tympany.

d. Hernia.

159. Which of these women, all of whom have recently discovered a new breast
lump, is at greatest risk for a diagnosis of breast cancer?

a. A 51-year-old whose mother had breast cancer and describes the


lump as non-tender.

b. A 55-year-old whose weight is normal for her height, and had one
child at age 31.

c. A 45-year-old who is taking estrogen therapy and has had four


children before the age of 28.

d. A 22-year-old who has fibrocystic breast disease and describes the


lump as painful.

160. A clinic nurse is attempting to obtain an initial health history from an East
Indian woman. The woman’s East Indian husband answers the nurse’s
questions while the wife sits quietly. What action should the nurse
implement?

a. Carefully observe the wife for any indication of physical abuse.

b. Continue the interview with the established communication pattern.

c. Advise the couple that the wife must answer the interview questions.
d. Escort the husband to the family room until the interview is
complete.

Вам также может понравиться